Final APUSH SBMC Review

Ace your homework & exams now with Quizwiz!

"The Student Nonviolent Coordinating Committee now states its opposition to United States involvement in Vietnam on these grounds: "We believe the United States government has been deceptive in its claims of concern for the freedom of the Vietnamese people, just as the government has been deceptive in claiming concern for the freedom of colored people in such other countries as the Dominican Republic, the Congo, South Africa, Rhodesia, and the United States itself. "We . . . have been involved in the black people's struggle for liberation and self-determination in this country for the past five years. Our work, particularly in the South, has taught us that the United States government has never guaranteed the freedom of oppressed citizens, and is not yet truly determined to end the rule of terror and oppression within its own borders. . . . "We are in sympathy with, and support, the men in this country who are unwilling to respond to a military draft which would compel them to contribute their lives to United States aggression in Vietnam in the name of the 'freedom' we find so false in this country." Student Nonviolent Coordinating Committee (SNCC), position paper on Vietnam, 1966 Which of the following historical processes most directly contributed to reactions like those depicted in the excerpt? A Efforts to contain the spread of communism following the Second World War B Attempts to secure United States oil supplies from the Middle East C Rejection of Great Society programs meant to combat poverty D Decline in public confidence in government following political scandals

A Efforts to contain the spread of communism following the Second World War

"Both the phrase 'Great Society' and the planning for it dated to May 1964, when [President Lyndon] Johnson addressed the graduating class of the University of Michigan. 'We have the opportunity,' he proclaimed, 'to move not only toward the rich society and the powerful society, but upward to the Great Society.' . . . Starting in that summer [of 1964] he also established the first of what ultimately became 135 'task forces' to study a wide range of social problems. . . . Much of what he requested aimed to go beyond . . . the New Deal in order to create a Great Society that would be qualitatively better and that would guarantee 'rights' and government entitlements." James T. Patterson, Grand Expectations: The United States, 1945-1974, published in 1996 The principles of the Great Society as expressed in the excerpt are best explained in the context of which of the following twentieth-century ideologies? A Liberalism B Conservatism C Feminism D Isolationism

A Liberalism

"In the mass movement into suburban areas a new kind of community was produced, which caricatured both the historic city and the archetypal suburban refuge: a multitude of uniform, unidentifiable houses, lined up inflexibly, at uniform distances, on uniform roads, in a treeless communal waste, inhabited by people of the same class, the same income, the same age group, witnessing the same television performances, eating the same tasteless pre-fabricated foods, from the same freezers, conforming in every outward and inward respect to a common mold, manufactured in the central metropolis. Thus the ultimate effect of the suburban escape in our time is, ironically, a low-grade uniform environment from which escape is impossible." Lewis Mumford, historian, The City in History: Its Origins, Its Transformations, and Its Prospects, 1961 Which of the following best explains a limitation in Mumford's critique of postwar suburbanization? A Many families moved to the new suburbs to find affordable homes. B The federal government allocated many new subdivisions for public housing. C Many African American and Latino American families moved to the new suburbs. D The federal government recognized suburban sprawl as an environmental concern.

A Many families moved to the new suburbs to find affordable homes.

"This morning the mob again gathered in front of the Central High School of Little Rock, obviously for the purpose of again preventing the carrying out of the Court's order relating to the admission of Negro children to the school. "Whenever normal agencies prove inadequate to the task and it becomes necessary for the Executive Branch of the Federal Government to use its powers and authority to uphold Federal Courts, the President's responsibility is inescapable. "In accordance with that responsibility, I have today issued an Executive Order directing the use of troops under Federal authority to aid in the execution of Federal law at Little Rock, Arkansas." President Dwight Eisenhower, national television and radio address, 1957 President Eisenhower's actions in the excerpt were most similar to which of the following earlier actions? A The ending of segregation in the military in the 1940s B The internment of Japanese Americans during the Second World War C The removal of American Indians to reservations in the 1830s D The issuing of the Emancipation Proclamation during the Civil War

A The ending of segregation in the military in the 1940s

"I have today signed an Executive Order providing for the establishment of a Peace Corps. . . . This Corps will be a pool of trained American men and women sent overseas by the U.S. Government or through private institutions and organizations to help foreign countries meet their urgent needs for skilled manpower. . . . "In establishing our Peace Corps we intend to make full use of the resources and talents of private institutions and groups. Universities, voluntary agencies, labor unions and industry will be asked to share in this effort—contributing diverse sources of energy and imagination—making it clear that the responsibility for peace is the responsibility of our entire society. ". . . . Our Peace Corps is not designed as an instrument of diplomacy or propaganda or ideological conflict. It is designed to permit our people to exercise more fully their responsibilities in the great common cause of world development. "Life in the Peace Corps will not be easy. There will be no salary and allowances will be at a level sufficient only to maintain health and meet basic needs. Men and women will be expected to work and live alongside the nationals of the country in which they are stationed—doing the same work, eating the same food, talking the same language. ". . . Every young American who participates in the Peace Corps—who works in a foreign land—will know that he or she is sharing in the great common task of bringing to man that decent way of life which is the foundation of freedom and a condition of peace." President John F. Kennedy, statement upon signing the order establishing the Peace Corps, 1961 Which of the following postwar developments most directly contributed to the ideas in the excerpt? A The foreign policy strategy of supporting developing nations as a means to prevent the spread of communism B The popularization of liberal policies characterized by movements for rights and equality C The ascendance of a youth counterculture and its role in leading social and cultural change D The decline of Cold War military strategies in the 1960s

A The foreign policy strategy of supporting developing nations as a means to prevent the spread of communism

In the 1960s, conservatives most likely had which of the following goals in challenging laws that they perceived as being liberal, such as Lyndon Johnson's Great Society programs? A They sought to limit the role of the federal government in people's lives. B They sought to restrain the growth of the military. C They sought to expand religious freedoms. D They sought to fight pollution and protect natural resources.

A They sought to limit the role of the federal government in people's lives.

"I have today signed an Executive Order providing for the establishment of a Peace Corps. . . . This Corps will be a pool of trained American men and women sent overseas by the U.S. Government or through private institutions and organizations to help foreign countries meet their urgent needs for skilled manpower. . . . "In establishing our Peace Corps we intend to make full use of the resources and talents of private institutions and groups. Universities, voluntary agencies, labor unions and industry will be asked to share in this effort—contributing diverse sources of energy and imagination—making it clear that the responsibility for peace is the responsibility of our entire society. ". . . . Our Peace Corps is not designed as an instrument of diplomacy or propaganda or ideological conflict. It is designed to permit our people to exercise more fully their responsibilities in the great common cause of world development. "Life in the Peace Corps will not be easy. There will be no salary and allowances will be at a level sufficient only to maintain health and meet basic needs. Men and women will be expected to work and live alongside the nationals of the country in which they are stationed—doing the same work, eating the same food, talking the same language. ". . . Every young American who participates in the Peace Corps—who works in a foreign land—will know that he or she is sharing in the great common task of bringing to man that decent way of life which is the foundation of freedom and a condition of peace." President John F. Kennedy, statement upon signing the order establishing the Peace Corps, 1961 The point of view expressed in the excerpt most directly illuminates which of the following debates within United States foreign policy during the post-Second World War period? A Whether to use direct or indirect tactics to challenge the influence of the Soviet Union B Whether to join mutual defense organizations such as the North Atlantic Treaty Organization (NATO) C Whether to support European nations' efforts to keep control over their overseas colonies D Whether to give the executive branch increased power over foreign and military policy

A Whether to use direct or indirect tactics to challenge the influence of the Soviet Union

"[The United States federal government in] Washington had a mixed response to Asian decolonization. On the one hand, it was not unhappy to see the European empires dissolved. Washington regarded these empires, which functioned as restricted trading blocs, as obstacles to economic integration and as incubators of communism and anti-Western revolution. On the other hand, Washington recognized that Europe's economic and political stability often depended upon income generated in the colonies. Whether the United States supported or opposed a particular nationalist movement often depended on its relationship to communism. . . . Washington only endorsed nationalist movements, such as those in Indonesia and the Philippines, that promised to preserve Western access after independence. It was willing to abolish formal empire, as long as the relations of informal empire continued uninterrupted." Christina Klein, Cold War Orientalism: Asia in the Middlebrow Imagination, 1945-1961, published in 2003 "Shortly after the outbreak of war between the Vietnamese and the French, Ho Chi Minh's Democratic Republic of [North] Vietnam (DRV) launched a four-month diplomatic initiative in the spring and summer of 1947 designed to secure the support of the [President Harry] Truman administration. . . . [The DRV's] agenda included calls for recognition of the DRV and mediation of the war with the French, requests for rehabilitation loans and promises of economic concessions to U.S. businesses, and appeals for technical assistance and cultural exchange. . . . ". . . With Soviet diplomacy focused on Europe and the Chinese communists preoccupied by civil war, the DRV also faced almost complete isolation from the communist world. . . . [But United States] fears of Vietnamese subservience to Moscow that first had emerged in 1946 intensified with the escalation of Soviet-American tensions in Europe. . . . The commitment of the United States to maintain French political and economic stability in Western Europe complicated its abilities to challenge French policies in Vietnam directly." Mark Philip Bradley, Imagining Vietnam and America: The Making of Post-Colonial Vietnam, 1919-1950, published in 2000 Bradley's argument differs from Klein's in that Bradley claims that A some communist governments actually sought assistance from the United States B decolonization occurred in areas where the United States was given access to trade C the United States actively opposed colonialism in Asia but not in Africa D the Soviet Union supported decolonization so that it could limit the influence of the United States

A some communist governments actually sought assistance from the United States

"[The United States federal government in] Washington had a mixed response to Asian decolonization. On the one hand, it was not unhappy to see the European empires dissolved. Washington regarded these empires, which functioned as restricted trading blocs, as obstacles to economic integration and as incubators of communism and anti-Western revolution. On the other hand, Washington recognized that Europe's economic and political stability often depended upon income generated in the colonies. Whether the United States supported or opposed a particular nationalist movement often depended on its relationship to communism. . . . Washington only endorsed nationalist movements, such as those in Indonesia and the Philippines, that promised to preserve Western access after independence. It was willing to abolish formal empire, as long as the relations of informal empire continued uninterrupted." Christina Klein, Cold War Orientalism: Asia in the Middlebrow Imagination, 1945-1961, published in 2003 "Shortly after the outbreak of war between the Vietnamese and the French, Ho Chi Minh's Democratic Republic of [North] Vietnam (DRV) launched a four-month diplomatic initiative in the spring and summer of 1947 designed to secure the support of the [President Harry] Truman administration. . . . [The DRV's] agenda included calls for recognition of the DRV and mediation of the war with the French, requests for rehabilitation loans and promises of economic concessions to U.S. businesses, and appeals for technical assistance and cultural exchange. . . . ". . . With Soviet diplomacy focused on Europe and the Chinese communists preoccupied by civil war, the DRV also faced almost complete isolation from the communist world. . . . [But United States] fears of Vietnamese subservience to Moscow that first had emerged in 1946 intensified with the escalation of Soviet-American tensions in Europe. . . . The commitment of the United States to maintain French political and economic stability in Western Europe complicated its abilities to challenge French policies in Vietnam directly." Mark Philip Bradley, Imagining Vietnam and America: The Making of Post-Colonial Vietnam, 1919-1950, published in 2000 Which of the following arguments about United States policy toward decolonization do the excerpts best support? A It sought to encourage decolonization by withholding economic support to European nations. B It was more interested in maintaining stability in regions than in backing decolonization. C It supported decolonization efforts by providing weapons and money to nationalists. D It rejected decolonization in favor of replacing European control with American control.

B It was more interested in maintaining stability in regions than in backing decolonization.

"The Student Nonviolent Coordinating Committee now states its opposition to United States involvement in Vietnam on these grounds: "We believe the United States government has been deceptive in its claims of concern for the freedom of the Vietnamese people, just as the government has been deceptive in claiming concern for the freedom of colored people in such other countries as the Dominican Republic, the Congo, South Africa, Rhodesia, and the United States itself. "We . . . have been involved in the black people's struggle for liberation and self-determination in this country for the past five years. Our work, particularly in the South, has taught us that the United States government has never guaranteed the freedom of oppressed citizens, and is not yet truly determined to end the rule of terror and oppression within its own borders. . . . "We are in sympathy with, and support, the men in this country who are unwilling to respond to a military draft which would compel them to contribute their lives to United States aggression in Vietnam in the name of the 'freedom' we find so false in this country." Student Nonviolent Coordinating Committee (SNCC), position paper on Vietnam, 1966 The activities of young people like those in SNCC exemplify which of the following broader changes in United States society during the 1960s? A Many embraced an idealized suburban lifestyle. B Many rejected the values of previous generations. C Many opposed the women's rights movement D Many supported for conservative political policies.

B Many rejected the values of previous generations.

"This morning the mob again gathered in front of the Central High School of Little Rock, obviously for the purpose of again preventing the carrying out of the Court's order relating to the admission of Negro children to the school. "Whenever normal agencies prove inadequate to the task and it becomes necessary for the Executive Branch of the Federal Government to use its powers and authority to uphold Federal Courts, the President's responsibility is inescapable. "In accordance with that responsibility, I have today issued an Executive Order directing the use of troops under Federal authority to aid in the execution of Federal law at Little Rock, Arkansas." President Dwight Eisenhower, national television and radio address, 1957 Which of the following contexts best explains the origins of the modern African American Civil Rights movement? A Evangelical Christians became increasingly active in national politics. B The federal government continued to fail to live up to its promises of guaranteeing the equality of citizens. C The United States increased military spending and foreign aid to limit the expansion of communism. D A conservative movement developed that criticized the role of court decisions in society.

B The federal government continued to fail to live up to its promises of guaranteeing the equality of citizens.

"What is important is the claim of the Communists themselves that for every party member there are 10 others ready, willing, and able to do the party's work. Herein lies the greatest menace of communism. . . . ". . . It might be of interest to observe that in 1917 when the Communists overthrew the Russian Government there was one Communist for every 2,277 persons in Russia. In the United States today there is one Communist for every 1,814 persons in the country. "One who accepts the aims, principles, and program of the party, who attends meetings, who reads the party press and literature, who pays dues, and who is active on behalf of the party 'shall be considered a member.' The open, avowed Communist who carries a card and pays dues is no different from a security standpoint than the person who does the party's work but pays no dues, carries no card, and is not on the party rolls. In fact, the latter is a greater menace because of his opportunity to work in stealth." J. Edgar Hoover, director of the Federal Bureau of Investigation (FBI), testimony before the House Un-American Activities Committee (HUAC), 1947 Which of the following best explains how the Red Scare following the Second World War reflected the larger historical context? A The ideas reveal strategies by the United States to gain support of other nations against Soviet expansion. B The ideas demonstrate efforts by the federal government to ensure domestic security. C The ideas represent a consensus among policy makers in favor of expanding the social welfare state. D The ideas show how conservatives mobilized to promote isolationism from international affairs.

B The ideas demonstrate efforts by the federal government to ensure domestic security.

The fact that Virginia prohibits only interracial marriages involving white persons demonstrates that the racial classifications must stand on their own justification, as measures designed to maintain White Supremacy. We have consistently denied the constitutionality of measures which restrict the rights of citizens on account of race. There can be no doubt that restricting the freedom to marry solely because of racial classifications violates the central meaning of the Equal Protection Clause. "These statutes also deprive the Lovings of liberty without due process of law in violation of the Due Process Clause of the Fourteenth Amendment. The freedom to marry has long been recognized as one of the vital personal rights essential to the orderly pursuit of happiness by free men. "Marriage is one of the 'basic civil rights of man,' fundamental to our very existence and survival. . . . To deny this fundamental freedom on so unsupportable a basis as the racial classifications embodied in these statutes . . . is surely to deprive all the State's citizens of liberty without due process of law. The Fourteenth Amendment requires that the freedom of choice to marry not be restricted by invidious racial discriminations. Under our Constitution, the freedom to marry, or not marry, a person of another race resides with the individual, and cannot be infringed by the State." United States Supreme Court, ruling in Loving v. Virginia, 1967 The ruling described in the excerpt is most similar to other civil rights achievements in the 1960s in that it A reflected feminist demands for changes within the Civil Rights movement B represented responses by the federal government to calls for the expansion of civil rights C represented debates among civil rights activists over the best strategies to pursue D reflected the adoption of violent protest tactics among civil rights activists in the 1960s

B represented responses by the federal government to calls for the expansion of civil rights

"Wall Street owns the country. It is no longer a government of the people, by the people, and for the people, but a government of Wall Street, by Wall Street, and for Wall Street. ... The [political] parties lie to us and the political speakers mislead us. We were told two years ago to go to work and raise a big crop, that was all we needed. We went to work and plowed and planted; the rains fell, the sun shone, nature smiled, and we raised the big crop that they told us to; and what came of it? Eight-cent corn, ten-cent oats, two-cent beef and no price at all for butter and eggs-that's what came of it. The politicians said we suffered from overproduction. Overproduction, when 10,000 little children, so statistics tell us, starve to death every year in the United States..." Mary Elizabeth Lease 1890. "Wall Street Owns The Country" Which group below would be the most supportive of the sentiments expressed in the passage above in the 1890s? a Republicans b Populists c Democrats d Socialists

B) Populists

Which of the following best explains the change in United States military combat deaths in Vietnam between 1964 and 1968 as depicted in the graph? A Growing opposition to the Vietnam War by university students in the United States B The end of the alliance between the Soviet Union and the United States C Increasing commitment by politicians in the United States to combat the spread of global communism D The direct intervention of Chinese military forces on the side of the North Vietnamese government

C Increasing commitment by politicians in the United States to combat the spread of global communism

"Both the phrase 'Great Society' and the planning for it dated to May 1964, when [President Lyndon] Johnson addressed the graduating class of the University of Michigan. 'We have the opportunity,' he proclaimed, 'to move not only toward the rich society and the powerful society, but upward to the Great Society.' . . . Starting in that summer [of 1964] he also established the first of what ultimately became 135 'task forces' to study a wide range of social problems. . . . Much of what he requested aimed to go beyond . . . the New Deal in order to create a Great Society that would be qualitatively better and that would guarantee 'rights' and government entitlements." James T. Patterson, Grand Expectations: The United States, 1945-1974, published in 1996 The social policies advocated by which of following earlier groups were most similar to the policies of the Great Society? A Supporters of Social Darwinism in the 1890s B Opponents of imperialism in the early 1900s C Progressives in the 1910s D Republican politicians in the 1920s

C Progressives in the 1910s

A sense of optimism in the 1950s and 1960s most likely emerged in response to which of the following contexts? A Changing foreign policies resulted in political debates. B Social changes brought about conservative reactions. C Rapid economic growth led to greater affluence. D Concerns about communist influence contributed to the Red Scare.

C Rapid economic growth led to greater affluence.

"In the mass movement into suburban areas a new kind of community was produced, which caricatured both the historic city and the archetypal suburban refuge: a multitude of uniform, unidentifiable houses, lined up inflexibly, at uniform distances, on uniform roads, in a treeless communal waste, inhabited by people of the same class, the same income, the same age group, witnessing the same television performances, eating the same tasteless pre-fabricated foods, from the same freezers, conforming in every outward and inward respect to a common mold, manufactured in the central metropolis. Thus the ultimate effect of the suburban escape in our time is, ironically, a low-grade uniform environment from which escape is impossible." Lewis Mumford, historian, The City in History: Its Origins, Its Transformations, and Its Prospects, 1961 Mumford overlooks which of the following broader historical contexts that best explains why many Americans might have been attracted to the consistency and conformity of the suburbs? A The rise of youth rebellion against middle class values helped expand a new consumer niche for products marketed directly to teenagers. B Many Americans developed a more cosmopolitan outlook after experiencing cultures different from their own during the Second World War. C Recent periods of economic depression and war encouraged many families to seek stability and security. D The technological innovations available in new homes, such as air-conditioning and dishwashers, fostered a sense of social isolation for many women.

C Recent periods of economic depression and war encouraged many families to seek stability and security.

Which of the following was the primary reason behind the change in United States military forces in Vietnam after 1969 as depicted in the graph? A Following his election President Richard Nixon began an immediate de-escalation of the conflict and North and South Vietnam eventually signed a peace treaty. B By shifting to a strategy more reliant on air raids and espionage, President Nixon was able to win the war with fewer soldiers. C Responding to decreasing public support for the Vietnam War, President Nixon eventually withdrew United States troops without eliminating communism or uniting Vietnam. D As part of his negotiation with China, President Nixon brought United States involvement in the Vietnam War to an end through diplomacy.

C Responding to decreasing public support for the Vietnam War, President Nixon eventually withdrew United States troops without eliminating communism or uniting Vietnam.

"The fact that Virginia prohibits only interracial marriages involving white persons demonstrates that the racial classifications must stand on their own justification, as measures designed to maintain White Supremacy. We have consistently denied the constitutionality of measures which restrict the rights of citizens on account of race. There can be no doubt that restricting the freedom to marry solely because of racial classifications violates the central meaning of the Equal Protection Clause. "These statutes also deprive the Lovings of liberty without due process of law in violation of the Due Process Clause of the Fourteenth Amendment. The freedom to marry has long been recognized as one of the vital personal rights essential to the orderly pursuit of happiness by free men. "Marriage is one of the 'basic civil rights of man,' fundamental to our very existence and survival. . . . To deny this fundamental freedom on so unsupportable a basis as the racial classifications embodied in these statutes . . . is surely to deprive all the State's citizens of liberty without due process of law. The Fourteenth Amendment requires that the freedom of choice to marry not be restricted by invidious racial discriminations. Under our Constitution, the freedom to marry, or not marry, a person of another race resides with the individual, and cannot be infringed by the State." United States Supreme Court, ruling in Loving v. Virginia, 1967 The ruling described in the excerpt is most similar to which of the following earlier political changes? A The enactment of universal voting rights for women during the Progressive Era B Court rulings during the Gilded Age that "separate but equal" public facilities were constitutional C The extension of citizenship rights to formerly enslaved people following the Civil War D The gradual shift of African American support to the Democratic Party during the New Deal

C The extension of citizenship rights to formerly enslaved people following the Civil War

"I have today signed an Executive Order providing for the establishment of a Peace Corps. . . . This Corps will be a pool of trained American men and women sent overseas by the U.S. Government or through private institutions and organizations to help foreign countries meet their urgent needs for skilled manpower. . . . "In establishing our Peace Corps we intend to make full use of the resources and talents of private institutions and groups. Universities, voluntary agencies, labor unions and industry will be asked to share in this effort—contributing diverse sources of energy and imagination—making it clear that the responsibility for peace is the responsibility of our entire society. ". . . . Our Peace Corps is not designed as an instrument of diplomacy or propaganda or ideological conflict. It is designed to permit our people to exercise more fully their responsibilities in the great common cause of world development. "Life in the Peace Corps will not be easy. There will be no salary and allowances will be at a level sufficient only to maintain health and meet basic needs. Men and women will be expected to work and live alongside the nationals of the country in which they are stationed—doing the same work, eating the same food, talking the same language. ". . . Every young American who participates in the Peace Corps—who works in a foreign land—will know that he or she is sharing in the great common task of bringing to man that decent way of life which is the foundation of freedom and a condition of peace." President John F. Kennedy, statement upon signing the order establishing the Peace Corps, 1961 President Kennedy's primary goal in enacting the program described in the excerpt was most likely to A increase the civic engagement and global activism of United States youth B train a more determined and persistent United States workforce through challenging overseas service C promote a free-market global economy through international aid to other nations D encourage the adoption of the foreign policy of détente

C promote a free-market global economy through international aid to other nations

"What is important is the claim of the Communists themselves that for every party member there are 10 others ready, willing, and able to do the party's work. Herein lies the greatest menace of communism. . . . ". . . It might be of interest to observe that in 1917 when the Communists overthrew the Russian Government there was one Communist for every 2,277 persons in Russia. In the United States today there is one Communist for every 1,814 persons in the country. "One who accepts the aims, principles, and program of the party, who attends meetings, who reads the party press and literature, who pays dues, and who is active on behalf of the party 'shall be considered a member.' The open, avowed Communist who carries a card and pays dues is no different from a security standpoint than the person who does the party's work but pays no dues, carries no card, and is not on the party rolls. In fact, the latter is a greater menace because of his opportunity to work in stealth." J. Edgar Hoover, director of the Federal Bureau of Investigation (FBI), testimony before the House Un-American Activities Committee (HUAC), 1947 At the time, Hoover's purpose in his testimony would have most likely been interpreted as attempting to convince Americans of the A need to expand United States nuclear defenses B danger exhibited by Soviet expansion in Eastern Europe C threat posed by suspected communists within the United States D links between domestic security and a prosperous consumer economy

C threat posed by suspected communists within the United States

"The Student Nonviolent Coordinating Committee now states its opposition to United States involvement in Vietnam on these grounds: "We believe the United States government has been deceptive in its claims of concern for the freedom of the Vietnamese people, just as the government has been deceptive in claiming concern for the freedom of colored people in such other countries as the Dominican Republic, the Congo, South Africa, Rhodesia, and the United States itself. "We . . . have been involved in the black people's struggle for liberation and self-determination in this country for the past five years. Our work, particularly in the South, has taught us that the United States government has never guaranteed the freedom of oppressed citizens, and is not yet truly determined to end the rule of terror and oppression within its own borders. . . . "We are in sympathy with, and support, the men in this country who are unwilling to respond to a military draft which would compel them to contribute their lives to United States aggression in Vietnam in the name of the 'freedom' we find so false in this country." Student Nonviolent Coordinating Committee (SNCC), position paper on Vietnam, 1966 Political statements such as the excerpt most directly culminated in which of the following? A The emergence of evangelical Christians as a political force in the United States B The creation of a national energy policy to limit the influence of oil-producing countries on the United States C The establishment of political institutions meant to reduce environmental pollution D The growth in opposition to United States foreign policies by many groups on the political left

D The growth in opposition to United States foreign policies by many groups on the political left

"In the mass movement into suburban areas a new kind of community was produced, which caricatured both the historic city and the archetypal suburban refuge: a multitude of uniform, unidentifiable houses, lined up inflexibly, at uniform distances, on uniform roads, in a treeless communal waste, inhabited by people of the same class, the same income, the same age group, witnessing the same television performances, eating the same tasteless pre-fabricated foods, from the same freezers, conforming in every outward and inward respect to a common mold, manufactured in the central metropolis. Thus the ultimate effect of the suburban escape in our time is, ironically, a low-grade uniform environment from which escape is impossible." Lewis Mumford, historian, The City in History: Its Origins, Its Transformations, and Its Prospects, 1961 Mumford's argument in the excerpt does not account for which of the following historical factors that most directly explains the rise of the suburbs in the United States? A Growth of public transportation in most major urban areas B Increased access to higher education opportunities C The government's recognition of labor unions in many industries D The necessity of having an automobile to travel to work

D The necessity of having an automobile to travel to work

"Both the phrase 'Great Society' and the planning for it dated to May 1964, when [President Lyndon] Johnson addressed the graduating class of the University of Michigan. 'We have the opportunity,' he proclaimed, 'to move not only toward the rich society and the powerful society, but upward to the Great Society.' . . . Starting in that summer [of 1964] he also established the first of what ultimately became 135 'task forces' to study a wide range of social problems. . . . Much of what he requested aimed to go beyond . . . the New Deal in order to create a Great Society that would be qualitatively better and that would guarantee 'rights' and government entitlements." James T. Patterson, Grand Expectations: The United States, 1945-1974, published in 1996 The expansion of African American rights during the Great Society was most similar to which of the following earlier situations? A The federal support for unionization of workers in the 1930s B The new laws protecting consumer rights in the 1900s C The end of property requirements for voting during the Jacksonian era D The passage of constitutional amendments during Reconstruction

D The passage of constitutional amendments during Reconstruction

"What is important is the claim of the Communists themselves that for every party member there are 10 others ready, willing, and able to do the party's work. Herein lies the greatest menace of communism. . . . ". . . It might be of interest to observe that in 1917 when the Communists overthrew the Russian Government there was one Communist for every 2,277 persons in Russia. In the United States today there is one Communist for every 1,814 persons in the country. "One who accepts the aims, principles, and program of the party, who attends meetings, who reads the party press and literature, who pays dues, and who is active on behalf of the party 'shall be considered a member.' The open, avowed Communist who carries a card and pays dues is no different from a security standpoint than the person who does the party's work but pays no dues, carries no card, and is not on the party rolls. In fact, the latter is a greater menace because of his opportunity to work in stealth." J. Edgar Hoover, director of the Federal Bureau of Investigation (FBI), testimony before the House Un-American Activities Committee (HUAC), 1947 The House Un-American Activity Committee (HUAC)'s interpretation of Hoover's testimony most likely influenced the committee to A approve significant increases in the FBI's budget B disregard Hoover's testimony as unreliable C criticize Hoover for political bias and inaccurate information gathering D investigate communism in labor unions and industries such as filmmaking

D investigate communism in labor unions and industries such as filmmaking

"The fact that Virginia prohibits only interracial marriages involving white persons demonstrates that the racial classifications must stand on their own justification, as measures designed to maintain White Supremacy. We have consistently denied the constitutionality of measures which restrict the rights of citizens on account of race. There can be no doubt that restricting the freedom to marry solely because of racial classifications violates the central meaning of the Equal Protection Clause. "These statutes also deprive the Lovings of liberty without due process of law in violation of the Due Process Clause of the Fourteenth Amendment. The freedom to marry has long been recognized as one of the vital personal rights essential to the orderly pursuit of happiness by free men. "Marriage is one of the 'basic civil rights of man,' fundamental to our very existence and survival. . . . To deny this fundamental freedom on so unsupportable a basis as the racial classifications embodied in these statutes . . . is surely to deprive all the State's citizens of liberty without due process of law. The Fourteenth Amendment requires that the freedom of choice to marry not be restricted by invidious racial discriminations. Under our Constitution, the freedom to marry, or not marry, a person of another race resides with the individual, and cannot be infringed by the State." United States Supreme Court, ruling in Loving v. Virginia, 1967 By the late 1960s, critics within the Civil Rights movement were dissatisfied with the tactics illustrated by the excerpt. These critics were increasingly likely to argue that A a constitutional amendment was needed to prevent the resegregation of public spaces B anticommunism should be a major component for gaining popular support of the movement C organizations needed to ally with other rights movements to gain more influence D nonviolent methods should be abandoned because they were ineffective

D nonviolent methods should be abandoned because they were ineffective

The activism for lesbian and gay rights most closely mirrored the activism for African American civil rights in that both groups A gained widespread political support for their causes B accepted the moral values of their parents' generation C avoided challenging discriminatory laws D used public protests to call for legal protections

D used public protests to call for legal protections

"This morning the mob again gathered in front of the Central High School of Little Rock, obviously for the purpose of again preventing the carrying out of the Court's order relating to the admission of Negro children to the school. "Whenever normal agencies prove inadequate to the task and it becomes necessary for the Executive Branch of the Federal Government to use its powers and authority to uphold Federal Courts, the President's responsibility is inescapable. "In accordance with that responsibility, I have today issued an Executive Order directing the use of troops under Federal authority to aid in the execution of Federal law at Little Rock, Arkansas." President Dwight Eisenhower, national television and radio address, 1957 Opponents of Eisenhower's decision in the excerpt most likely held views similar to which of the following earlier groups? A Radical Republicans in Congress in the 1870s B Southern state leaders in the 1880s and 1890s C New Deal Democrats in the North in the 1930s and 1940s D Immigrants in urban ethnic neighborhoods in the early 1900s

Southern state leaders in the 1880s and 1890s

Presidential Election of 1860 Which of the following was the most immediate result of the election outcome shown on the map? a A national draft law was passed by the North to help build up its military in preparation for the inevitable coming of war. b Congressional leaders called for constitutional amendments which would reestablish the Missouri Compromise line. c Southern states began seceding from the union. d Slaveholding states of the upper South, such as Maryland, began to free their slaves.

Southern states began seceding from the union.

"It is now fourteen weeks since the revolution of government here. Future consequences we are ignorant of, yet we know that, at present we are eased of the great oppressions that we groaned under...making the arbitrary commission of Sir Edmund Andros null and void in the law;...although some could not advise to [recommend] the enterprise, yet [all] are hopeful that we shall not be greatly blamed, but shall have a pardon granted for any error the law will charge us with in this matter. ... Nature has taught us self-preservation....Our great remoteness from England denies us the opportunity of direction for the regulation of ourselves in all emergencies, nor have we the means to know the laws and customs of our nation....We have always endeavored to prove ourselves loyal to the Crown of England...and we are not without hopes but that we shall receive from Their Royal Majesties the confirmation of our charter, with such addition of privileges as may advance the revenue of the Crown, and be an encouragement to Their Majesties' subjects here." Letter from Thomas Danforth of Massachusetts to a colleague in London, 1689 The excerpt above best demonstrates which of the following about how colonials perceived British imperial rule. a. Colonists had developed a sense of independence through colonial traditions of self-government but were still loyal to the crown. b. Colonists resisted attempts by Great Britain to assert tighter control over the colonists by rebelling against the crown. c. By 1689, colonials sought independnce from Great Britain by deposing Sir Edmund Andros. d. By 1689, influence from the Enlightenment and First Great Awakening fostered the American Revolution.

a. Colonists had developed a sense of independence through colonial traditions of self-government but were still loyal to the crown.

"Their world, quite literally, changed before the Indians' eyes as European colonists transformed the forest into farmland. . . . In the Southeast, hogs ran wild. Sheep and goats became permanent parts of the economy and culture of Pueblo and Navajo peoples in the Southwest. Horses transformed the lives and cultures of Indian peoples on the plains. Europeans also brought honeybees, black rats, cats, and cockroaches to America.' — Colin G. Calloway, historian, First Peoples: A Documentary Survey of American Indian History, 2012 The excerpt describes effects of the a. Columbian Exchange b. European Enlightenment c. Middle Passage d. Great Awakening

a. Columbian Exchange

"Col. MASON. Slavery...bring[s] the judgement of heaven on a Country. Mr. ELSWORTH. Let us not intermeddle. As population increases poor laborers will be so plenty as to render slaves useless. Slavery in time will not be a speck in our Country... abolition has already taken place in Massachusetts. General PINKNEY. S. Carolina and Georgia cannot do without slaves. The more slaves, the more produce to emply the carrying trade; the more consumption also, and the more of this, the more of revenue for the common treasury. He...should consider a rejection of the clause as an exclusion of S. Carola. from the Union." James Madison, Notes from the Constitutional Convention, August 22, 1787 Which of the following best describes the impact of the decisions made by the Constitutional Convention regarding slavery following the debates excerpted above? a. Conflict over the issues was postponed via compromise, rather than resolved. b. Fears about the size of government led the convention to forbid the national government from preventing the importation of slaves. c. Strong abolitionist from northern states led to the creation of civil rights protections. d. The southern economy's viability was challenged, setting the stage for sectional conflict.

a. Conflict over the issues was postponed via compromise, rather than resolved.

*bill maulden's The Strategists cartoon* The situation depicted in the cartoon came into existence as a result of the pursuit of which of the following policy goals? a. Containment of communism b. Decolonization in Asia c. Détente with China and the Soviet Union d. Nuclear supremacy

a. Containment of communism

"It is for us the living, rather, to be dedicated here to the unfinished work which they who fought here have thus far so nobly advanced. It is rather for us to be here dedicated to the great task remaining before us—that from these honored dead we take increased devotion to that cause for which they gave the last full measure of devotion—that we here highly resolve that these dead shall not have died in vain—that this nation, under God, shall have a new birth of freedom—and that government of the people, by the people, for the people, shall not perish from the earth." Abraham Lincoln, Gettysburg Address, November 1863 Which of the following most directly contributed to the conflict referred to in the excerpt? a Disagreements over whether to allow slavery in new territories b Debates about the role of religion in society and government c Tensions between isolationism and international engagement d Disputes over taxation and representation

a. Disagreements over whether to allow slavery in new territories

"There was with General Washington, during most of the summer, a Seneca chief, called The Great Tree, who, on leaving the head-quarters of the Commander-in-chief, professed the strongest friendship for the American cause, and his first object, after his return to his own people, was to inspire them with his own friendly sentiments. While passing through the Oneida nation on his way home, he professed the strongest confidence in his ability to keep his own tribe bound in the chain of friendship, and pledged himself, in the event of his failure, to come down with his friends and adherents and join the Oneidas. The Oneida messengers were farther told that all the Indians west of their own tribe, including, of course, the Onondagas, together with the Indian settlements on the Susquehanna and its branches, were to join them. They were to rendezvous somewhere on the Tioga, and make a descent either upon the Pennsylvania or New Jersey frontier." -- William E. Stone, Life of Joseph Brant - Thayendanegea: including the Border Wars of the American Revolution, published in 1838 The nature of the relationships amongst the various Native American tribes revealed in the excerpt best supports which statement? a. Divisions among the tribes contributed to the Native American loss in their fight to limit migration of white settlers. b. Native American tribes successfully maintained a strong confederacy in their common goal to stop further European settlement of Native American lands. c. Indecision and chaos amongst the Native American tribes weakened their resolve to fight for protection of their lands. d. Warfare between the tribes did not contribute to the defeat of Native Americans in their fight to save their lands.

a. Divisions among the tribes contributed to the Native American loss in their fight to limit migration of white settlers.

*graph showing general increase in government spending in various sectors (national defense, social service)* The pattern in spending for national defense shown in the table most directly reflects which of the following? a. Efforts to contain communism in Southeast Asia b. Increased public confidence in the United States government c. Democratic Party dominance in national politics after 1968 d. A decisive loss for the United States in the Korean War

a. Efforts to contain communism in Southeast Asia

*Picture of graph showing declining native population* Which of the following was the most direct effect of the changes shown in the graph? a. European settlers were able to gain control over Native American lands. b. Trade between European settlers and Native Americans expanded. c. Conflicts among Native American groups became more intense. d. European settlers relied mainly on indentured servants and enslaved Africans for labor.

a. European settlers were able to gain control over Native American lands.

"Unlike some anticommunists. . . I have always believed that we can and must communicate and, when possible, negotiate with Communist nations. . . . "There were, however, a few things in our favor. The most important and interesting was the Soviet split with China. . . . "It was often said that the key to a Vietnam settlement lay in Moscow and Peking rather than in Hanoi. . . . Aside from wanting to keep Hanoi from going over to Peking, Moscow had little stake in the outcome of the North Vietnamese cause, especially as it increasingly worked against Moscow's own major interests vis-à-vis the United States. While I understood that the Soviets were not entirely free agents where their support for North Vietnam was concerned, I nonetheless planned to bring maximum pressure to bear on them in this area." —Richard Nixon, RN: Memoirs of Richard Nixon, 1978 Which of the following best explains why Nixon's foreign policy was a departure from the previous administrations'? a. He exploited that Communism was not a unified world movement b. He was the first president willing to negotiate with communist leaders c. He was willing to use massive bombing to force an issue d. He turned over his foreign policy to his national security advisor

a. He exploited that Communism was not a unified world movement

The "women's lib" movement is not an honest effort to secure better jobs for women whowant or need to work outside the home. This is just the superficial sweet-talk to win broad support for a radical "movement." Women's lib is a total assault on the role of the American woman as wife and mother, and on the family as the basic unit of society. Women's libbers are trying to make wives and mothers unhappy with their career, make them feel that they are "second-class citizens" and "abject slaves." Women's libbers are promoting free sex instead of the "slavery" of marriage. They are promoting Federal "day-care centers" for babies instead of homes. They are promoting abortions instead of families. - Phyllis Schlafly, What's Wrong with 'Equal Rights' for Women?, February 1972 All of the following were acts passed during the 1960s-1970s in an attempt to extend some equal rights to women EXCEPT a. Higher Education Act of 1965 b. Equal Employment Opportunity Commission c. Equal Pay Act of 1963 d. Title IX of the Education Amendments of 1972

a. Higher Education Act of 1965

"In the mass movement into suburban areas a new kind of community was produced, which caricatured both the historic city and the archetypal suburban refuge: a multitude of uniform, unidentifiable houses, lined up inflexibly, at uniform distances, on uniform roads, in a treeless communal waste, inhabited by people of the same class, the same income, the same age group, witnessing the same television performances, eating the same tasteless pre-fabricated foods, from the same freezers, conforming in every outward and inward respect to a common mold, manufactured in the central metropolis. Thus the ultimate effect of the suburban escape in our time is, ironically, a low-grade uniform environment from which escape is impossible. What has happened to the suburban exodus in the United States now threatens, through the same mechanical instrumentalities, to take place, at an equally accelerating rate, everywhere else—unless the most vigorous countermeasures are taken..." Lewis Mumford on Suburbanization Which of the following could best serve as evidence to support Mumford's contentions in the excerpt? a. Levittown's refusal to sell homes to African Americans b. the explosive growth of television and television sales c. personalization of suburban homes through paint color and structural changes d. the growth of organizations like Little League and the PTA

a. Levittown's refusal to sell homes to African Americans

"The reason why we find ourselves in a position of impotency is not because our only powerful potential enemy has sent men to invade our shores ... but rather because of the traitorous actions of those who have been treated so well by this Nation. It has not been the less fortunate, or members of minority groups who have been traiorous to this Nation, but rather those who have had the benefits that the wealthiest Nation on earth has had to offer ... the finest homes, the finest college education, and the finest jobs in government we can give. This is glaringly true in the State Department. There the bright young men who are born with silver spoons in their mouths are the onces who have been most trairorous." Senator Joseph McCarthy, speech in Wheeling, West Virginia, 1950. The claims McCarthy made in the speech heightened debates over which of the following issues? a. Policies and methods to root out possible communists within the United States b. United States intervention in the Korean War c. The expansion of the United States nuclear arsenal d. The emergence of a military-industrial complex in the United States

a. Policies and methods to root out possible communists within the United States

The image most directly reflects the growing belief after World War II that a. The United States would have to take on a more global role of leadership. b. Communism should be allowed to spread to all of Europe. c. The United States should become isolationist to prevent involvement in another war. d. Europe was the dominant power in the world and would direct future economic efforts worldwide.

a. The United States would have to take on a more global role of leadership.

"It is proposed that humble application be made for an act of Parliament of Great Britain, by virtue of which one general government may be formed in America, including all the said colonies, within and under which government each colony may retain its present constitution, except in the particulars wherein a change may be directed by the said act as hereafter follows....That they make such laws as they judge necessary for regulating all Indian trade....That they raise and pay soldiers, and build forts for the defence of any of the Colonies, and equip vessels of force to guard the coasts and protect the trade on the oceans, lakes, or great rivers; but shall not impress men in any Colony, without the consent of the Legislature. That for these purposes they have power to make laws and lay and leavy such general duties, imposts, or taxes as to them shall appear most equal and just...and such as may be collected with the least inconvenience to the people..." Albany Plan of Union, 1754 Documents Illustrative of the Formation of the Union of the American States, ed. Charles C. Tansill. (Washington, D.C.: Government Printing Office, 1927) House Document No. 398. The excerpt above is indicative of which of the following? a. The colonists having grown accustomed to a large measure of autonomy b. Growing mistrust on both sides of the Atlantic c. An ideology critical of perceived corruption in the imperial system d. The colonial production of commodities that were valued in Europe

a. The colonists having grown accustomed to a large measure of autonomy

"I. Resolved, that the several States composing the United States of America... delegated to [the national] government certain definite powers, reserving each State to itself, the residuary mass of right to their own self government.... II....therefore the act of Congress, passed on the 14th day of July, 1798 ... [is] altogether void... and that the power to create, define, and punish such other crimes is reserved solely and exclusively to the respective States.... VIII....that every State has a natural right in cases not within the compact, to nullify of their own authority all assumptions of power by others within their limits." - Thomas Jefferson, Kentucky Resolutions, 1798 The conflict revealed in the passage led most directly to which of the following? a. The creation of the first political parties b. George Washington's Farewell Address c. New legislation for the government of the western territories d. the dissolution of the Articles of the Confederation

a. The creation of the first political parties

"In 1680 Pueblo leaders united most of their communities against the European intruders. . . . In a matter of weeks, the Pueblos had eliminated Spaniards from New Mexico above El Paso. The natives had killed over 400 of the province's 2,500 foreigners, destroyed or sacked every Spanish building, and laid waste to the Spaniards' fields. There could be no mistaking the deep animosity that some natives, men as well as their influential wives and mothers, held toward their former oppressors. . . . Some Pueblo leaders. . . urgedanend to all things Spanish as well as Christian. After the fighting subsided, they counselled against speaking Castilian or planting crops introduced by the Europeans." - David J. Weber, historian, The Spanish Frontier in North America, 1992 Which of the following most shaped the events described in the excerpt? a. The demands of the encomienda system in the Spanish colonies b. The importance of Old World crops to the subsistence of Spanish colonial subjects c. The establishment of African slavery in the Spanish colonies d. The amount of Spanish intermarriage with Native Americans

a. The demands of the encomienda system in the Spanish colonies

*pic of levittown advertisement* Americans whose lifestyle paralleled the image above most likely opposed which aspect of post-World War II society? a. The growing counterculture b. Military actions to contain communism c. Cultural conformity d. The continued growth in federal spending

a. The growing counterculture

"Wee must be knitt together in this worke as one man, wee must entertaine each other in brotherly Affeccion... wee must delight in eache other, make others Condicions our owne rejoyce together, mourne together, labour, and suffer together... soe that wee shall see much more of his wisdome power goodnes and truthe then formerly wee have beene acquainted with, wee shall finde that the God of Israell is among us, when tenn of us shall be able to resist a thousand of our enemies, when hee shall make us a prayse and glory, that men shall say of succeeding plantacions: the lord make it like that of New England: for wee must Consider that wee shall be as a Citty upon a Hill, the eies of all people are uppon us; soe that if wee shall deale falsely with our god in this worke wee have undertaken and soe cause him to withdrawe his present help from us, wee shall be made a story and a byword through the world, wee shall open the mouthes of enemies to speake evill of the wayes of god and all professours for Gods sake." John Winthrop, A Model of Christian Charity, 1630 Winthrop's ideas on Puritan settlement were largely based on which of the following? a. The idea that the Massachusetts Bay Colony was a colony of religious believers b. Fears of Spanish territorial settlement in New England c. The prohibition of slavery in New England in this period d. The ethnic diversity of colonists in New England

a. The idea that the Massachusetts Bay Colony was a colony of religious believers

"I said everything to them I could to divert them from their idolatries, and draw them to a knowledge of God our Lord. Moctezuma replied, the others assenting to what he said, that they had already informed me they were not the aborigines of the country, but that their ancestors had emigrated to it many years ago; and they fully believed that after so long an absence from their native land, they might have fallen into some errors; that I having more recently arrived must know better than themselves what they ought to believe; and that if I would instruct them in these matters, and make them understand the true faith, they would follow my directions, as being for the best. Afterwards, Moctezuma and many of the principal citizens remained with me until I had removed the idols, purified the chapels, and placed the images in them, manifesting apparent pleasure." - Letter from Hernán Cortés to Charles V, Holy Roman Emperor and King of Spain, on his interaction with the Mexica (Aztecs), 1520 Moctezuma's statement that the Mexica "were not the aborigines of the country" most likely refers to which of the following developments? a. The presence of different and complex societies before European contact b. Widespread deadly epidemics before the arrival of the Spanish c. The development of African slavery in the Spanish colonies d. Use of the encomienda system by the Mexica before the arrival of the Spanish

a. The presence of different and complex societies before European contact

"An act for the better regulating the government of the province of the Massachusetts's Bay, in New England. I. ...Be it therefore enacted by the King's most excellent Majesty ... That the charter, granted ... to the inhabitants of the said province of the Massachusetts's Bay... be revoked, and is hereby revoked and made void and of none effect; ...And that ... the council, or court of assistants of the said province ... shall be thereunto nominated and appointed by his Majesty, III. And be it further enacted by the authority aforesaid, ... it shall and may be lawful for his Majesty's governor ... to nominate and appoint, ... and also to remove, without the consent of the council, all judges of the inferior courts of common pleas, commissioners of Oyer and Terminer, the attorney general, provosts, marshals, justices of the peace, and other officers to the council or courts of justice belonging ..." -- Massachusetts Government Act (one of the so-called Intolerable or Coercive Acts) Which of the following best describes the effect of laws such as that above on how a significant number of American colonists' came to view themselves in relation to the British? a. They moved many colonists to define themselves in opposition to the British government. b. They undermined revolutionary sentiments of colonists by showing the futility of rebellion. c. They lessened regional and class divisions as colonists saw themselves as united Americans. d. They reinforced colonists' status as subjects to the authority of the monarch.

a. They moved many colonists to define themselves in opposition to the British government.

"The national dignity and justice require that the arms of the Union should be called forth in order to chastise the Creek nation of Indians, for refusing to treat with the United States on reasonable terms, and for their hostile invasion of the State of Georgia....The most effectual mode of reducing the Creeks to submit to the will of the United States...would be by an adequate army, to be raised and continued until the objects of the war should be accomplished....But, in future, the obligations of policy, humanity, and justice, together with that respect which every nation sacredly owes to its own reputation, unite in requiring a noble, liberal, and disinterested administration of Indian affairs....In the administration of the Indians, every proper expedient that can be devised to gain their affections, and attach them to the interest of the Union, should be adopted....Missionaries, of excellent moral character, should be appointed to reside in their nation....They should be their friends and fathers." - From Henry Knox to George Washington, July 7, 1789, in American State Papers, Class II: Indian Affairs During the late 1700s, which of the following was the most difficult challenge for most American Indian groups with the United States? a. Treaty disputes and the seizure of Indian lands b. Gaining free navigation of the Mississippi River c. Navigating frontier conflicts between England, Spain, and the United States d. Unwanted attempts to assimilate Indian groups into American society

a. Treaty disputes and the seizure of Indian lands

"Their world, quite literally, changed before the Indians' eyes as European colonists transformed the forest into farmland. . . . In the Southeast, hogs ran wild. Sheep and goats became permanent parts of the economy and culture of Pueblo and Navajo peoples in the Southwest. Horses transformed the lives and cultures of Indian peoples on the plains. Europeans also brought honeybees, black rats, cats, and cockroaches to America.' — Colin G. Calloway, historian, First Peoples: A Documentary Survey of American Indian History, 2012 Europeans most typically explained the process described in the excerpt in which of the following ways? a. Viewing it as proof of the higher level of civilization among Europeans b. Interpreting it as evidence of a need to protect the rights of artisans and farmers c. Asserting that all people were equal and deserving of the same rights d. Denying that such changes had taken place

a. Viewing it as proof of the higher level of civilization among Europeans

*bill mauldin's the Strategists's cartoon* The issues depicted in the cartoon most closely resemble which of the following issues facing the United States in the nineteenth century? a. Whether to acquire new territory in the Western Hemisphere and the Pacific b. Whether to allow slavery to expand to newly acquired territories c. Whether to limit migration from other countries d. Whether to compromise with Southern leaders to avoid civil war

a. Whether to acquire new territory in the Western Hemisphere and the Pacific

Whereas it is... essential to our Interest, and the Security of our Colonies, that the several Nations or Tribes of Indians ...who live under our Protection, should not be molested or disturbed ...--We do therefore,...declare it to be our Royal Will and Pleasure... for the use of the said Indians, [to be reserved] all the Lands and Territories ...lying to the Westward of the Sources of the Rivers [along the Appalachian Mountains] which fall into the Sea....And We do hereby strictly forbid... all our loving Subjects from... taking Possession of any of the Lands above reserved..." The Royal Proclamation, King George III, October 1763 British Proclamations like this one directly led to a. a growing sense of American identity in the face of perceived injustices. b. open warfare between the British colonists and their French counterparts. c. the development of a series of alliances between the British colonists and neighboring Native American tribes. d. an increase in the economic profitability of the British colonies in North America.

a. a growing sense of American identity in the face of perceived injustices.

"I congratulate you on the successes of our two allies. Those of the Hollanders are new, and therefore pleasing. It proves there is a god in heaven, and the he will not slumber without end on the iniquities of tyrants, or would-be tyrants, as their Stadtholders [political leaders]. This ball of liberty, I believe most piously, is now so well in motion that it will roll round the globe, at least the enlightened part of it, for light & liberty go together. It is our glory that we first put it into motion, & our happiness that being foremost we had not bad examples to follow. What a tremendous obstacle to the future attempts at liberty will be the atrocities of Robespierre!" Letter from Thomas Jefferson to Tench Coxe, June 1, 1795 According to the above excerpt, the American Declaration of Independence and subsequent victory in the war of independence most directly led to a. a movement towards political self-determination in several other b. the overthrow of absolute monarchical systems of government throughout Europe. c. the United States being seen as a power nation with the most coveted form of government. d. the spread of Enlightenment ideals across the trans-atlantic World.

a. a movement towards political self-determination in several other

"Thus, fellow citizens, have I pointed out what I thought necessary to be amended in our Federal Constitution. I beg you to call to mind our glorious Declaration of Independence, read it, and compare it with the Federal Constitution; what a degree of apostacy will you not then discover. Therefore, guard against all encroachments upon your liberties so dearly purchased with the costly expense of blood and treasure." A Georgian, Gazette of the State of Georgia, November 15, 1787 The opinion expressed in the excerpt would most likely have been held by a. an Anti-Federalist b. an advocate of republican motherhood c. an abolitionist d. a Loyalist

a. an Anti-Federalist

"[S]ince a report had been made to the king on the fertility of the soil by [Sieur de Monts] and by me on the feasibility of discovering the passage to China,... his Majesty directed Sieur de Monts to make a new outfit, and send men to continue what he had commenced He was also influenced by the hope of greater advantages in case of settling in the interior, where the people are civilized,. than along the sea-shore, where the [natives] generally dwell. From this course, he believed the king would derive an inestimable profit; for it is easy to suppose that Europeans will seek out this advantage rather than those of a jealous and intractable disposition to be found on the shores." Samuel de Champlain, French explorer, 1604 The French most differed from the Spanish in relations with American Indians in that the French a. developed stronger alliances with American Indians b. converted more American Indians to Catholicism c. more frequently enslaved American Indians d. did not intermarry with American Indians

a. developed stronger alliances with American Indians

"The Secretary of the Treasury having perused with attention the papers containing the opinions of the Secretary of State and Attorney General concerning the constitutionality of the bill for establishing a National Bank proceeds according to the order of the President [George Washington] to submit the reasons which have induced him to entertain a different opinion...the objections of the Secretary of State and Attorney General are founded on a general denial of the authority of the United States to erect corporations...It is not denied, that there are implied, as well as express powers...They are "to make all laws, necessary & proper for carrying into execution the foregoing powers & all other powers vested by the constitution in the government of the United States, or in any department or officer thereof." ...A bank has a natural relation to the power of collecting taxes; to that of borrowing money; to that of regulating trade; to that of providing for the common defence...it brings the case within the provision of the clause of the constitution which immediately respects the property of the United States." Alexander Hamilton, "An Opinion on the Constitutionality of an Act to Establish a Bank," 1791 The split in opinion over Hamilton's economic policies most directly led to the a. emergence of two political parties, the Federalists and the Democratic Republicans. b. diplomatic incident known as the XYZ Affair. c. Quasi-War between the United States and France. d. outbreak of hostilities between the Indians and the farmers on the Western frontier.

a. emergence of two political parties, the Federalists and the Democratic Republicans.

"The Americas were discovered in 1492, and the first Christian settlements established by the Spanish the following year.... [I]t would seem... that the Almighty selected this part of the world as home to the greater part of the human race.... [T]heir delicate constitutions make them unable to withstand hard work or suffering and render them liable to succumb to almost any illness, no matter how mild....It was upon these gentle lambs... that, from the very first day they clapped eyes on them, the Spanish fell like ravening wolves upon the fold, or like tigers and savage lions who have not eaten meat for days....The native population, which once numbered some five hundred thousand, was wiped out by forcible expatriation to the island of Hispaniola." Bartolome de Las Casas, 1552 An implication of Las Casas' argument is that a major cause of the decline of the native populations in the Americas after 1492 was the a. epidemics brought to the Americas by Europeans b. importation of European and African labor wildlife to the Americas c. large-scale clashes between native armies Americans and the Spanish d. resistance of indigenous groups to military religious conversion

a. epidemics brought to the Americas by Europeans

"What began as a protest movement is being challenged to translate itself into a political movement. It is now concerned not merely with removing the barriers to full opportunity but with achieving the fact of equality. From sit-ins and freedom rides we have gone into rent strikes, boycotts, community organization, and political action. As a consequence of this natural evolution, the Negro today finds himself stymied by obstacles of far greater magnitude than the legal barriers he was attacking before: automation, urban decay, de facto school segregation." — Bayard Rustin, "From Protest to Politics," 1965 The Civil Rights movement's shift in focus described in the excerpt most directly contributed to a. increased divisions among activists over strategies and goals b. growing cooperation with feminist groups c. more concentration on problems in the South d. greater use of nonviolent demonstrations as a protest tactic

a. increased divisions among activists over strategies and goals

"For the increase of shipping . . . from thencefoquestion 33rward, no goods or commodities whatsoever shall be imported into or exported out of any lands, islands, plantations, or territories to his Majesty belonging . . . but in ships or vessels as do . . . belong only to the people of England . . . and whereof the master and three-fourths of the mariners at least are English. . . . "And it is further enacted . . . that . . . no sugars, tobacco, cottonwool, indigos, ginger, fustic, or other dyeing wood, of the growth, production, or manufacture of any English plantations in America, Asia, or Africa, shall be . . . transported from any of the said English plantations [colonies] to any land . . . other than to such other English plantations as do belong to his Majesty." English Parliament, Navigation Act of 1660 The policies stated in the above law can best be seen as an example of a. mercantilism b. Enlightenment thinking. c. capitalism. d. communism

a. mercantilism

Resolutions proposed by Ms. Randolph in Convention May 29, 1787. 2. Resolved that the rights of suffrage in the National Legislature ought to be proportioned ... to the number of free inhabitants. 3. Resolved that the National Legislature ought to consist of two branches. 4. Resolved that the memers of hte first branch of the National Legislature ought to be elected by the people of the several States... 5. Resolved that the members of the second branch of the National Legislature ought to be elected by those of the first ... nominated by the individual Legislatures. 7. Resolved that a National Executive be instituted; ... [and granted] a general authority to execute the National laws. 9. Resolved that a National Judiciary be established to consist of one or more supreme [courts] ...to hold their offices during good behavior. Notes from the Constitutional Convention as recorded by James Madison, May 29, 1787. The debates at the Constitutional Convention deadlocked over the issue of a. representation in the National Legislature b. issue of slavery c. separation of powers d. powers of the Executive branch

a. representation in the National Legislature

"Had these principles been adhered to, we should not have been brought to this alarming transition, from a confederacy to a consolidated government...Here is a resolution as radical as that which separated us from Great Britain. It is radical in this transition; our rights and privileges are endangered, and the sovereignty of the states will be relinquished: and cannot we plainly see that this is actually the case? The rights of conscience, trial by jury, liberty of the press, all your immunities and franchises, all pretensions to human rights and privileges are rendered insecure, if not lost, by this change, so loudly talked of by some, and inconsiderately by others. Is this tame relinquishment of rights worthy of freemen?...Guard with jealous attention the public liberty" Patrick Henry, "Speech against the Federal Constitution," June 5, 1788 In the excerpt, Henry's fear of a "consolidated government" most directly reflects which of the following? a. the belief that a strong national government will trample on the rights of individuals b. the concern that state legislatures will be dissolved by the new government c. the suspicion that an anti-slavery coalition will dominate the government d. the perception that America will develop into a monarchy

a. the belief that a strong national government will trample on the rights of individuals

*Picture displaying the number of slaves going to each part of the America's* A significant long-term result of the major pattern depicted on the map was a. the development of a strict racial system in British colonial societies b. cooperation between European countries over colonization in the America c. an increase in the destructiveness of American Indian warfare d. frequent intermarriage between people of African and British descent

a. the development of a strict racial system in British colonial societies

"For more than four centuries, the communities formed by escaped slaves dotted the fringes of plantion America...Maroons and their communities were... a widespread and embarrassingly visible part of these systems. The very nature of plantation slavery engendered violence and resistence, and the wilderness setting of early New World plantations allowed marronage. ... Throughout Afro-America, such communities stood out as an heroic challenge to white authority, and as living proof of a slave consciousness that refused to be limited by the whites' definition and manipulation of it." Richard Price, "Maroons: Rebel Slaves in the Americas," 1992. The existence of communities like those described by Richard Price in the passage above most clearly demonstrates a. the strength of attempts by Africans in the Americas to maintain cultural autonomy b. Europeans' challenges in developing economically viable colonies c. the devastating impact that disease and abuse would have on the culture of transplanted Africans d. the lack of options for most enslaved Africans once under the pressure of the plantation system

a. the strength of attempts by Africans in the Americas to maintain cultural autonomy

" Let us, then, fellow-citizens, unite with one heart and one mind. Let us restore to social intercourse that harmony and affection without which liberty and even life itself are but dreary things. And let us reflect that, having banished from our land that religious intolerance under which mankind so long bled and suffered, we have yet gained little if we countenance a political intolerance as despotic, as wicked, and capable of as bitter and bloody persecutions...We have called by different names brethren of the same principle. We are all Republicans, we are all Federalists. If there be any among us who would wish to dissolve this Union or to change its republican form, let them stand undisturbed as monuments of the safety with which error of opinion may be tolerated where reason is left free to combat it." Thomas Jefferson, first inaugural address, 1801 The excerpt best reflects which of the following? a. Fear that the United States would be overtaken by a foreign power b. Conflicts over how the Constitution should be implemented and interpreted c. Disagreement over the consequences of the French Revolution for the United States d. Secessionist pressures coming from slaveholders in the South

b

"I must confine myself to few examples, but am ready to furnish other and more complete details, if required. If my pictures are displeasing, coarse, and severe, my subjects must be recollected, offer no tranquil, refined, or composing features. The condition of human beings, reduced to the extremest states of degradation and misery, cannot be exhibited in softened language, or adorn a polished page. I proceed, gentlemen, briefly to call your attention to the present state of insane persons, confined within this Commonwealth, in cages, closets, cellars, stalls, pens! Chained, naked, beaten with rods, and lashed into obedience." Dorothea Dix, 1843 Dorothea Dix a. fought to improve conditions for American factory workers. b. worked for better treatment for the mentally ill. c. worked in support of the "cult of domesticity". d. encouraged church membership for everyone.

b

"If any one proposition could command the universal assent of mankind, we might expect it would be this: that the government of the Union, though limited in its powers, is supreme within its sphere of action. This would seem to result necessarily from its nature. It is the government of all; its powers are delegated by all; it represents all, and acts for all. Though any one State may be willing to control its operations, no State is willing to allow others to control them. The nation, on those subjects on which it can act, must necessarily bind its component parts.... Although, among the enumerated powers of government, we do not find the word 'bank' or 'incorporation,' we find the great powers to lay and collect taxes; to borrow money; to regulate commerce; to declare and conduct a war; and to raise and support armies and navies...a government, intrusted with such ample powers...must also be instructed with ample means for their execution....We are unanimously of opinion, that the law passed by the legislature of Maryland, imposing a tax on the Bank of the United States, is unconstitutional and void." Chief Justice John Marshall, McCullough v. Maryland, 1819 The ideas expressed in the excerpt above can best be understood in the context of debates over a. how to match democratic political ideals to political institutions. b. the relationship between the federal government and state governments. c. the authority of different branches of the federal government. d. the scope of the federal government's role in the economy.

b

"The question, whether an act, repugnant to the constitution, can become the law of the land, is a question deeply interesting to the United States, but, happily, not of an intimacy proportioned to its interest. It seems only necessary to recognize certain principles, supposed to have been long and well established to decide it... Thus, the particular phraseology of the constitution of the United States confirms and strengthens the principle, supposed to be essential to all written constitutions, that a law repugnant to the constitution is void; and that courts, as well as other departments, are bound by that instrument..." Chief Justice John Marshall, 1803 Which of the following judicial concepts is the above passage referring to? a. Separation of powers b. Judicial review c. The necessary and proper clause d. The three branches of government

b

"This momentous question like a fire-bell in the night, awakened and filled me with terror. I considered it at once as the knell of the Union. It is hushed, indeed, for the moment. But this is a reprieve only, not a final sentence. A geographical line, coinciding with a marked principle, moral and political, once conceived and held up to the angry passions of men, will never be obliterated; and every new irritation will mark it deeper and deeper....But as it is, we have the wolf by the ears, and we can neither hold him, nor safely let him go. Justice is in one scale, and self-preservation in the other." Thomas Jefferson, Letter to John Holmes, 1820Thomas Jefferson Randolph, ed., Memoirs, Correspondence, and Private Papers of Thomas Jefferson (London: Henry Colburn and Richard Bentley, 1829), 4:332. The concerns expressed in the letter above can best be understood in the context of a. competing ideas about geographical boundaries. b. debates over the extension of slavery into the western territories. c. concerns over the rights and responsibilities of individual citizens. d. federal efforts to control American Indian populations.

b

"[I promise]...to demonstrate in the course of...my Appeal...that we Coloured People of these United States, are, the most wretched, degraded and abject set of beings that ever lived since the world began, down to the present day, and that the white Christians of America, who hold us in slavery, (or, more properly speaking, pretenders to Christianity,) treat us more cruel and barbarous than any Heathen nation did any people whom it had subjected, or reduced to the same condition....I advance it therefore to you...as an unshaken and forever immoveable fact, that your full glory and happiness, as well as all other coloured people under Heaven, shall never be fully consummated, but with the entire emancipation of your enslaved brethren all over the world." David Walker, Appeal to the Coloured Citizens of the World, 1829David Walker, Walker's Appeal in Four Articles; Together with a Preamble, To the Coloured Citizens of the World, but in Particular, and Very Expressly, to Those of the United States of America Written in Boston, State of Massachusetts, September 28, 1829 (Boston: Revised and Published by David Walker, 1830). During the antebellum era, which of the following groups shared the most similar experience to that described in the excerpt above? a. European migrants b. American Indians c. White women d. Abolitionists

b

"[T]he condition of the African race throughout all the States where the ancient relation between the two [races] has been retained enjoys a degree of health and comfort which may well compare with that of the laboring population of any country in Christendom; and, it may be added that in no other condition, or in any other age or country, has the Negro race ever attained so high an elevation in morals, intelligence, or civilization." - John C. Calhoun, political leader, 1844 Which of the following most directly undermines Calhoun's assertions? a. Many slaves adopted elements of Christianity. b. Many slaves engaged in forms of resistance to slavery. c. A majority of White Southerners were not slaveholders. d. Abolitionist societies encountered difficulty organizing in Southern states.

b

All of the following groups would have been likely to agree with the image's depiction of Andrew Jackson EXCEPT a. South Carolinians who opposed federal tariffs b. western farmers who opposed the national bank c. American Indians such as Cherokees d. advocates of federally funded internal improvement projects

b

The change depicted on the maps most directly contributed to which of the following? a. The drawing of a boundary line to discourage settlers from moving west of the Appalachians b. The creation of more interconnected and efficient markets for consumer goods c. The calls for the United States annexation of French and Spanish territories in the Mississippi River valley d. The establishment of a national bank to promote more stability in financial transactions

b

The creator of the image would most likely agree with which of the following statements? a. Jackson was celebrated by the people because his foreign policies benefited the nation. b. Jackson's political actions were unconstitutional. c. Jackson's natural leadership skills made him an ideal president. d. Jackson would have accepted internal improvements if Congress had consulted him.

b

The maps above most clearly demonstrate which of the following antebellum-era historical processes? a. The rise of abolitionist and other voluntary reform organizations b. The free and forced migration of peoples across the continent c. The acceleration of a national and international market economy d. The attempts of the United States to dominate the North American continent

b

"Our...destiny [is] to overspread the continent allotted by Providence for the free development of our yearly multiplying millions...The Anglo-Saxon foot is already on [California's] borders. Already the advance guard of the irresistible army of the Anglo-Saxon emigration has begun to pour down upon it, armed with the [plow] and the rifle, and marking its trail with schools and colleges, courts and representative halls, mills, and meeting houses. A population will soon be in actual occupation of California...Their right to independence will be the natural right of self-government belonging to any community strong enough to maintain it." - John L. O'Sullivan, 1845 The ideas expressed in the passage above most clearly show the influence of which of the following? a Concerns about foreign alliances expressed in George Washington's Farewell Address b Concepts of republican democracy founded in the Declaration of Independence. c Beliefs in separation of powers articulated in the United States Constitution d models of limited government inherent in the Articles of Confederation

b Concepts of republican democracy founded in the Declaration of Independence.

"Give us a business man [to] introduce business methods into politics and government"... yet politics remains corrupt... The commercial spirit is the spirit of profit, not patriotism; of credit, not honor; of individual gain, not national prosperity; of trade and dickering, not principle... "Business is business" is not a political sentiment, but our politician has caught it. He takes essentially the same view of the bribe... But there is hope...in the commercialism of our politics. If our political leaders are to be always a lot of political merchants, they will supply any demand we may create. All we have to do is to establish a steady demand for good government." Lincoln Steffens, The Shame of the Cities, 1904 Which of the following methods did reformers like Lincoln Steffens use to accomplish their goals? a Organizing labor unions to oppose business interests b Creating new organizations to address the problems they saw c Calling for restricted immigration to reduce corruption d Campaigning against further government intervention in the people's lives

b Creating new organizations to address the problems they saw

American farming is growing less profitable and less encouraging. In a country possessing so many faculties of cheap production this discouraging aspect of agriculture must be and is the result of other than natural causes. The annual additions of wealth under the enlightened system of agriculture are enormous, but from the unequal divisions of the profits of labor and the unjust discriminations made against it, the enlistments of property show that the farmers of the United States are not prospering...The farmers of America have on all occasions shown themselves to be a patient and enduring people..." 13th National Meeting of the Grange, 1868 Which of the following reasons led to the formation of the Grange? a The rise of industrialization b Discontentment with the railroads c The New South d Indians encroaching upon the land

b Discontentment with the railroads

"The Cold War is now behind us. Let us not wrangle over who won it. It is in the common interest of our two countries and nations not to fight this trend toward cooperation but rather to promote it." Soviet Premier Mikhail Gorbachev, 1990 Mikhail S. Gorbechev, "Address at Stanford University," New York Times, 1990. Which U.S. foreign policy was most consistent with the goal articulated in the statement above? a The containment of communism in the 1950s b Détente in the 1970s c Isolationism in the 1920s and 1930s d Increased defense spending in the 1980s

b Détente in the 1970s

The nineteenth-century temperance movement laid the foundation for which of the following? a Twenty-second and Twenty-third Amendments b Eighteenth and Nineteenth Amendments c Twentieth and Twenty-first Amendments d Sixteenth and Seventeenth Amendments

b Eighteenth and Nineteenth Amendment

"Since the days when the fleet of Columbus sailed into the waters of the New World, America has been another name for opportunity, and the people of the United States have taken their tone from the incessant expansion which has not only been open but has even been forced upon them. ...Movement has been its dominant fact, and, unless this training has no effect upon a people, the American energy will continually demand a wider field for its exercise. But never again will such gifts of free land offer themselves. ...each frontier did indeed furnish a new field of opportunity, a gate of escape from the bondage of the past; and freshness, and confidence, and scorn of older society, impatience of its restraints and its ideas, and indifference to its lessons, have accompanied the frontier. What the Mediterranean Sea was to the Greeks, breaking the bond of custom, offering new experiences, calling out new institutions and activities, that, and more, the ever retreating frontier has been to the United States directly, and to the nations of Europe more remotely. And now, four centuries from the discovery of America, at the end of a hundred years of life under the Constitution, the frontier has gone, and with its going has closed the first period of American history." --Frederick Jackson Turner, historian, essay titled "The Significance of the Frontier in American History," first presented in 1893 and later collected in a book of essays by Turner titled The Frontier in American History, ©1920 Conclusions about the American frontier, as conveyed by Turner, most directly prompted what general concern in America towards the turn of the 20th century? a Fears of an economic depression b Fears of the dwindling of America's natural resources c Fears of political radicalism in the U.S. d Fears over the stability of the currency based on gold

b Fears of the dwindling of America's natural resources

*Can't find chart, but it shows the economic status of both the North and the South during the Civil War* Which of the following was a significant cause of the trend illustrated in the chart? a Growing urban centers in the South b Generations of industrial development in the North c A more diversified economic structure in the South d Failure to develop agriculture in the North

b Generations of industrial development in the North

"On the first of February we intend to begin submarine warfare unrestricted. In spite of this, it is our intention to endeavor to keep neutral the United States of America. "If this attempt is not successful, we propose an alliance on the following basis with Mexico: That we shall make war together and together make peace. We shall give general financial support, and it is understood that Mexico is to reconquer the lost territory in New Mexico, Texas, and Arizona. The details are left to you for settlement." Arthur Zimmermann, German Foreign Minister, January 19, 1917 Which of the following does this excerpt support as the primary cause of the U.S. declaration of war in April 1917? a Pro-British intelligence and propaganda b Germany's violation of U.S. neutral rights c Mexico's plan to invade the United States d Germany's violation of the Monroe Doctrine

b Germany's violation of U.S. neutral rights

"The Vigilance Committee of Boston inform you that the MOCK TRIAL of the poor Fugutuve Slave has been further postponed. ... Come down, then, Sons of the Puritans: for even if the poop victim is to be carried off by the brute force of arms, and delivered over to Slavery, you should at least be present to witnessss the sacrifice, and you should follow him in sad procession with your tears and prayers, and then go home and take such action as your manhood and your patriotism may suggest. Come, then, by the early trains on MONDAY, and rally....Come with courage and resolution in your hearts; but, this time, with only such arms as God gave you." Proclamation addressed "To the Yeomanry of New England," Boston, 1854 The issuing of documents such as the proclamation generally had which of the following effects? a Improving relations between native born residents and recent Catholic immigrants b Increasing the visibility of organized opposition to slavery c Encouraging armed rebellions by enslaved people in the South d Accumulating support for the theory of slavery as a positive good

b Increasing the visibility of organized opposition to slavery

*map showing the creation of independent states in Eastern Europe after the collapse of the Soviet Union* The map above best supports which of the following contentions about the end of the Cold War? a It prompted the United States to embark on a policy of isolationism. b It created new political and military challenges for the United States. c It undermined the process of economic globalization. d It stoked widespread public support for enlarging the U.S. nuclear arsenal.

b It created new political and military challenges for the United States.

It has been settled by the decisions of the highest court in Missouri, that by the laws of that State, a slave does not become entitled to his freedom, where the owner takes him to reside in a State where slavery is not permitted, and afterwards brings him back to Missouri. Conclusion. It follows that it is apparent upon the record that the court below erred in its judgment on the plea in abatement and also erred in giving judgment for the defendant, when the exception shows that the plaintiff was not a citizen of the United States. And as the Circuit Court had no jurisdiction, either in the case stated in the plea in abatement, or in the one stated in the exception, its judgment in favor of the defendant is erroneous, and must be reversed. - Dred Scott v. Sandford, 1857 What effect did the above ruling have on previous American legislation regarding slavery? a It invalidated the Fugitive Slave Act b It declared the provisions of Missouri Compromise unconstitutional c It validated the Black Codes of the South d It strengthened the provisions of the Northwest Compromise

b It declared the provisions of Missouri Compromise unconstitutional

Let us know the modern city in its weakness and wickedness, and then seek to rectify and purify it until it shall be free at least from the grosser temptations which now beset the young people who are living in its tenement houses and working in its factories." - Excerpt from Jane Addams, The Spirit of Youth and the City Streets, 1909. Which of the following groups did the most to improve the lives of recent immigrants through settlement houses? a Unionized factory workers b Middle class women c Wealthy philantropists d Urban political machines

b Middle class women

The majority of immigrants who arrived in the United States between 1821 and 1880 settled in the a South and Midwest b Midwest and Northeast c South and Northeast d West and Midwest

b Midwest and Northeast

Which of the following was NOT a reason for the Union's ultimate success in the war? a The Anaconda Plan b Military initiative and superior leadership of George McClellan c Greater resources of food and military supplies d Destruction of Southern infrastructure and supply lines

b Military initiative and superior leadership of George McClellan

Which of the following groups or movements most opposed the process illustrated above? a Trusts b Populists c Social Darwinists d Republicans

b Populists

"There was never the least attention paid to what was cut up for sausage; there would come all the way back from Europe old sausage that had been rejected, and that was moldy and white--it would be dosed with borax and glycerine, and dumped into the hoppers, and made over again for home consumption. There would be meat that had tumbled out on the floor, in the dirt and sawdust, where the workers had tramped and spit uncounted billions of consumption germs. There would be meat stored in great piles in rooms; and the water from leaky roofs would drip over it, and thousands of rats would race about on it. It was too dark in these storage places to see well, but a man could run his hand over these piles of meat and sweep off handfuls of the dried dung of rats. These rats were nuisances, and the packers would put poisoned bread out for them; they would die, and then rats, bread, and meat would go into the hoppers together." The Jungle, Upton Sinclair 1906 The practices described in the excerpt above most directly led to a violent strikes in the meatpacking industry as workers for more sanitary working conditions b Progressive urban and middle-class Americans advocating a greater role for government in regulating industry in the United States c the rise of the "People's Party" to fight corporate greed in the meatpacking industry d greater acceptance of muckraking journalists like Upton Sinclair in mainstream media.

b Progressive urban and middle-class Americans advocating a greater role for government in regulating industry in the United States

*picture of a man planting a tree for the WPA during the depression* The roots of FDR's New Deal policies can be traced to which political group in American history? a Jeffersonians b Progressives c Bull-Moose Party d Radical Republicans

b Progressives

"The era of big government is over but we can't go back to a time when our citizens were just left to fend for themselves. We will meet them by going forward as one America, by working together in our communities, our schools, our churches and synagogues, our workplaces across the entire spectrum of our civic life." - President Bill Clinton, radio address to the nation, 1996 The ideas expressed by Clinton in the excerpt were most similar to those of which twentieth-century president? a Lyndon Johnson b Ronald Reagan c Woodrow Wilson d Franklin Roosevelt

b Ronald Reagan (i think)

Which of the following groups would be most likely to support the Populist Party? a Industrialists b Sharecroppers c Immigrants d Bankers

b Sharecropper

"The Republican electoral triumph in 2004 was the culmination of a half-century of struggle by the Right to achieve political power in the United States. In the immediate aftermath of the Second World War, a small band of intellectuals launched a movement to stop what they saw as the advance of the collectivist state embodied in modern liberalism and the New Deal political order. They were joined by anti-Communist activists across grassroots America....In their struggle against the dominant liberal state, conservatives gained control of the Republican party by defeating its liberal eastern wing." Donald T. Critchlow, historian, The Conservative Ascendancy: How the GOP Right Made Political History, 2007 Which of the following best supports Critchlow's assertion about the Republican Party? a The continued expansion of Great Society programs b The 1980 election of Ronald Reagan as president c The end of the Cold War d The political scandals under Richard Nixon's administration in the 1970s

b The 1980 election of Ronald Reagan as president

"...They buried him in the midst of a marble quarry—they cut through solid marble to make his grave—and yet a little tombstone they put above him was from Vermont. They buried him in the heart of a pine forest, and yet the pine coffin was imported from Cincinnati...the iron in the shovel that dug his grave was imported from Pittsburg. They buried him by the side of the best sheep-grazing country on earth...and the coffin bands were brought from the North. The South didn't furnish a thing on earth for that funeral but the corpse and the hole in the ground..." Henry Grady, 1889 Which is true of the point of view expressed by the author in the excerpt? a A person does not think of all the resources and where they come from when burying someone. b The South needs to industrialize and make its own products. c Labor unions need to be created in the South to encourage industrialization. d Farmers need to join together to prevent further abuses of sharecropping and tenant farming

b The South needs to industrialize and make its own products.

*picture of General Motors poster, says "Cars on Credit"* Many historians criticize the economy that developed during the 1920s. Which of the following statements best supports that point of view? a The growth of the auto industry badly hurt the railroads b The boom was based on speculation and borrowed money c Advertising was based on gaining status and popularity d Consumerism weakened the moral character of the nation

b The boom was based on speculation and borrowed money

*picture of lucky strike cigarette advertisement* Which of the following historical developments was most likely responsible for increasing the effectiveness of the advertisement above? a The cultural conflict of urban versus rural b The continued development of the mass media c Greater personal mobility d Progressive attempts to regulate the abuses of the economy

b The continued development of the mass media

*graph showing the difference between the highest income's high shares of income and the other income groups* Which of the following factors contributed most to the major trend depicted in the graph? a The construction of the interstate highway system b The declining number of manufacturing jobs in the United States c The expansion of the social safety net through programs like Social Security d The increase in United States trade with eastern Europe after the Cold War

b The declining number of manufacturing jobs in the United States

"Americans faced an overwhelming task after the Civil War and emancipation: how to understand the tangled relationship between two profound ideas—healing and justice. . . . [T]hese two aims never developed in historical balance. One might conclude that this imbalance between outcomes of sectional healing and racial justice was simply America's inevitable historical condition. . . . But theories of inevitability . . . are rarely satisfying. . . . The sectional reunion after so horrible a civil war was a political triumph by the late nineteenth century, but it could not have been achieved without the resubjugation of many of those people whom the war had freed from centuries of bondage. This is the tragedy lingering on the margins and infesting the heart of American history from Appomattox to World War I." David W. Blight, historian, Race and Reunion: The Civil War in American Memory, 2001 Which of the following best characterizes the "sectional reunion" Blight describes? a New political alliances united northern and southern members of the Democratic Party to win control of both houses in Congress. b The federal government removed troops from the South and eliminated aid for former slaves. c Gilded Age financial policies encouraged economic growth in the North and the South. d White laborers in the North and African American farmers in the South joined together in the Populist movement.

b The federal government removed troops from the South and eliminated aid for former slaves.

It has been settled by the decisions of the highest court in Missouri, that by the laws of that State, a slave does not become entitled to his freedom, where the owner takes him to reside in a State where slavery is not permitted, and afterwards brings him back to Missouri. Conclusion. It follows that it is apparent upon the record that the court below erred in its judgment on the plea in abatement and also erred in giving judgment for the defendant, when the exception shows that the plaintiff was not a citizen of the United States. And as the Circuit Court had no jurisdiction, either in the case stated in the plea in abatement, or in the one stated in the exception, its judgment in favor of the defendant is erroneous, and must be reversed. - Dred Scott v. Sandford, 1857 The Court's ruling in the excerpt is most indicative of which of the following developments in American society during the years leading up to the Civil War? a The increasing influence of abolitionist literature in the South b The growing sectionalism in the United States c The growth of Nativist parties and anti-immigrant feelings d The decrease in power of the established political parties

b The growing sectionalism in the United States

"It was in suburbs such as Garden Grove, Orange County [California] . . . that small groups of middle-class men and women met in their new tract homes, seeking to turn the tide of liberal dominance. Recruiting the like-minded, they organized study groups, opened 'Freedom Forum' bookstores, filled the rolls of the John Birch Society, entered school board races, and worked within the Republican Party, all in an urgent struggle to safeguard their particular vision of freedom and the American heritage. In doing so, they became the ground forces of a conservative revival—one that transformed conservatism from a marginal force preoccupied with communism in the early 1960s into a viable electoral contender by the decade's end." — Lisa McGirr, historian, Suburban Warriors: The Origins of the New American Right, 2001 Which of the following historical developments could best be used as evidence to support McGirr's argument in the excerpt? a The passage of the Civil Rights Act of 1964 b The new public defenses of the traditional family as an ideal c The rise of the Black Power movement d The emergence of a counterculture

b The new public defenses of the traditional family as an ideal

"It's been quite a journey this decade, and we held together through some stormy seas. And at the end, together, we're reaching our destination. The fact is, from Grenada to the Washington and Moscow summits, from the recession of '81 to '82, to the expansion that began in late '82 and continues to this day, we've made a difference. The way I see it, there were two great triumphs, two things that I'm proudest of. One is the economic recovery, in which the people of America created—and filled—nineteen million new jobs. The other is the recovery of our morale. America is respected again in the world and looked to for leadership." President Ronald Reagan, Farewell Address to the Nation, January 1989 Reagan's reference to "reaching our destination" most directly reflects which of the following political changes? a The expanded influence of young people in politics b The rise of a new conservatism that challenged liberal views about the role of government c Increased political mobilization of labor unions d Bipartisan support for reform of the United States financial system

b The rise of a new conservatism that challenged liberal views about the role of government

"The peace-loving nations must make a concerted effort in opposition to those violations of treaties and those ignorings of humane instincts which today are creating a state of international anarchy and instability from which there is no escape through mere isolation or neutrality. "Those who cherish their freedom and recognize and respect the equal right of their neighbors to be free and live in peace, must work together for the triumph of law and moral principles in order that peace, justice and confidence may prevail in the world. There must be a return to a belief in the pledged word, in the value of a signed treaty. There must be recognition of the fact that national morality is as vital as private morality." - President Franklin Roosevelt, Quarantine Speech, 1937 Which of the following best represents continuity in the years after 1945 with the ideas that Roosevelt expressed in the excerpt? a The conviction and execution of suspected Soviet spies in the United States b United States membership in an international peacekeeping body c The rise of peace organizations opposed to the buildup and use of nuclear weapons d United States military commitment to countries battling communist insurgencies

b United States membership in an international peacekeeping body (maybe)

*picture w/ pie graph with ethnicities of immigrants in 1860-1900 and 1900-1920* Which of the following best describes the direct effect of the immigration patterns of the late 1800s and early 1900s as shown in the graphs on the development of urban centers in America during this period? a Rural southerners shunned the industrial cities of the North. b Urban centers saw the growth of ethnic communities and neighborhoods. c Urban parks and public spaces were deliberately created to ease the congestion of the cities. d The more affluent moved increasingly to the suburbs.

b Urban centers saw the growth of ethnic communities and neighborhoods.

"Our nation will continue to be steadfast, and patient and persistent in the pursuit of two great objectives. First, we will shut down terrorist camps, disrupt terrorist plans and bring terrorists to justice. And second, we must prevent the terrorists and regimes who seek chemical, biological or nuclear weapons from threatening the United States and the world....Our second goal is to prevent regimes that sponsor terror from threatening America or our friends and allies with weapons of mass destruction....States like these [North Korea, Iran, and Iraq], and their terrorist allies, constitute an axis of evil, arming to threaten the peace of the world. By seeking weapons of mass destruction, these regimes pose a grave and growing danger....And all nations should know: America will do what is necessary to ensure our nation's security." President George W. Bush, State of the Union Address, January 2002 Which of the following groups would most likely have supported the point of view expressed in the above passage? a Anti-war opponents during the Mexican-American War b Western, pro-war expansionists during the War of 1812 c Anti-imperialists during the Spanish-American War d Pro-British loyalists during the Revolutionary War

b Western, pro-war expansionists during the War of 1812 (i think)

"The Negro race, like all races, is going to be saved by its exceptional men. The problem of education, then, among Negroes must first of all deal with the Talented Tenth; it is the problem of developing the Best of this race that they may guide the Mass away from the contamination and death of theWorst, in their own and other races. Now the training of men is a difficult and intricate task. Its technique is a matter for educational experts, but its object is for the vision of seers. If we make money the object of man-training, we shall develop money-makers but not necessarily men; if we make technical skill the object of education, we may possess artisans but not, in nature, men. Men we shall have only as we make manhood the object of the work of the schools —intelligence, broad sympathy, knowledge of the world that was and is, and of the relation of men to it." W. E. B. Du Bois, "The Talented Tenth," 1903 The perspective expressed in the excerpt most directly supported the national expansion of a exams for civil service jobs b access to higher education c vocational job training d financial aid for college

b access to higher education

"Article 2: [T]he United States now solemnly agrees that no persons . . . shall ever be permitted to pass over, settle upon, or reside in . . . this reservation for the use of said Indians. "Article 6: If any individual belonging to said tribes of Indians, or legally incorporated with them, being the head of a family, shall desire to commence farming, he shall have the privilege to select . . . a tract of land within said reservation, not exceeding three hundred and twenty acres in extent. "Article 11: [T]he tribes who are parties to this agreement hereby stipulate that they will relinquish all right to occupy permanently the territory outside their reservations . . . but yet reserve the right to hunt on any lands north of North Platte, and on the Republican Fork of the Smoky Hill river, so long as the buffalo may range thereon in such numbers as to justify the chase. . . . They will withdraw all opposition to the construction of the railroads now being built on the plains. . . . They will not attack any persons at home, or traveling, nor molest or disturb any wagon trains, coaches, mules, or cattle belonging to the people of the United States." Second Treaty of Fort Laramie, agreed between the United States government and various bands of the Sioux nation, 1868 the conflict between the Sioux nation and the United States was primarily driven by differing a forms of government b claims to land c family structures d styles of farming

b claims to land

"There remains, then, only one mode of using great fortunes; but in this we have the true antidote for the temporary unequal distribution of wealth, the reconciliation of the rich and the poor—a reign of harmony. . . .Under its sway we shall have an ideal state, in which the surplus wealth of the few will become, in the best sense, the property of the many, because administered for the common good; and this wealth, passing through the hands of the few, can be made a much more potent force for the elevation of our race than if it had been distributed in small sums to the people themselves. Even the poorest can be made to see this, and to agree that great sums gathered by some of their fellow-citizens and spent for public purposes, from which the masses reap the principal benefit, are more valuable to them than if scattered among them through the course of many years in trifling amounts. Andrew Carnegie, "Wealth," 1889 "The temporary unequal distribution of wealth" that Carnegie refers to in the excerpt resulted most directly from the a efforts by workers to organize local and national unions b consolidation of corporations into trusts and holding companies c government policy of reducing tariffs to promote free trade d growth of cities in both size and number

b consolidation of corporations into trusts and holding companies

the image above most clearly epitomized the Gilded Age focus on a labor reform. b consumption. c utopianism. d business consolidation.

b consumption.

"The President of the United States...hereby is authorized, whenever in his opinion any reservation or any part thereof...is advantageous for agricultural and grazing purposes...to allot the lands in said reservation in severalty to any Indian located thereon in quantities as follows: To each head of family, one-quarter of a section; To each single person over eighteen years of age, one eighth of a section; To each single orphan child under eighteen years of age, one eighth of a section... Every Indian born within the territorial limits of the United States to whom allotments shall have been made...who has voluntarily taken up, within said limits his residence separate and apart from any tribe of Indians therein, and has adopted that habits of civilized life, is hereby declared a citizen of the United States." The Dawes Severalty Act, 1887 The primary goal of the government policy cited above was to a promote a larger reservation system. b end tribal identities. c respond to military resistance by Indians. d launch new treaties with Indians.

b end tribal identities.

The situation depicted in the image best serves as evidence of the a decline of an agrarian economy b expansion of federal power c institutionalization of racial segregation d increase in sectional divisions

b expansion of federal power

"The Republican electoral triumph in 2004 was the culmination of a half-century of struggle by the Right to achieve political power in the United States. In the immediate aftermath of the Second World War, a small band of intellectuals launched a movement to stop what they saw as the advance of the collectivist state embodied in modern liberalism and the New Deal political order. They were joined by anti-Communist activists across grassroots America....In their struggle against the dominant liberal state, conservatives gained control of the Republican party by defeating its liberal eastern wing." Donald T. Critchlow, historian, The Conservative Ascendancy: How the GOP Right Made Political History, 2007 The success of the new conservative movement in achieving its goals was most challenged by the a reform of federal tax policies b ongoing popularity of many social welfare programs c Persian Gulf War in the early 1990s d increased role of Christian evangelicals in the Republican Party

b ongoing popularity of many social welfare programs

This political cartoon most directly reflects a government support of big business b poltiical corruption and graft in city politics c how new immigrants changed American culture d opportunities for economic advancement in a city

b poltiical corruption and graft in city politics

(map showing immigration rates across nation) The Impact of Immigration Nancy A. Hewitt and Steven F. Lawson, Exploring American Histories, Bedford/St. Martin's, p. 564. Reprinted by permission. Between 1880 and 1900, the largest group of immigrants to the United States came from a Latin America. b southern and eastern Europe. c Africa. d northern and western Europe.

b southern and eastern Europe.

"To those of my race who depend on bettering their condition in a foreign land or who underestimated the importance of cultivating friendly relations with the Southern white man, who is their next-door neighbor, I say: 'Cast down your bucket where you are'—cast it down in making friends in every manly way of the people of all races by whom we are surrounded....To those of the white race who look to the incoming of those of foreign birth and strange tongue and habits for the prosperity of the South, were I permitted I would repeat what I say to my own race, 'Cast down your bucket where you are.' Cast it down among the eight million of Negroes whose habits you know." Booker T. Washington, Atlanta Exposition Address, 1885 A decade after the speech above, segregation polices were reinforced by a the president. b the Supreme Court. c monopolies and corporate interests. d Congress.

b the Supreme Court. think plessy v ferguson

*picture of A conductor evicting a black man from a first-class coach* The image most directly reflects which of the following? a the nativist fear of foreigners in nineteenth-century America b the establishment of racial segregation in the post-Reconstruction South c the prosperity of black merchants in the New South d the capture of a fugitive slave in antebellum Massachusetts

b the establishment of racial segregation in the post-Reconstruction South

*map showing the creation of independent states in Eastern Europe after the collapse of the Soviet Union* One result of the changes depicted in the map above was a decreased U.S. dependence on fossil fuels. b the formation of new diplomatic relationships abroad. c growing debate over the benefits of free-trade policies. d increased international concern about climate change.

b the formation of new diplomatic relationships abroad.

The image of "Woman's holy war" most directly reflects which of the following? a the boycotting of imported goods by patriotic American women during the Spanish-American War b the involvement of many nineteenth-century American women in the movement to ban alcohol c the participation of some nineteenth-century American women in military service d the influence of nineteenth-century religious revivals on American women

b the involvement of many nineteenth-century American women in the movement to ban alcohol

"[I am] commanded to explain to the Japanese that...[the United States] population has rapidly spread through the country, until it has reached the shores of the Pacific Ocean; that we have now large cities, from which, with the aid of steam vessels, we can reach Japan in eighteen or twenty days; [and] that. ... the Japan seas will soon be covered with our vessels. "Therefore, as the United States and Japan are becoming every day nearer and nearer to each other, the President desires to live in peace and friendship with your imperial majesty, but no friendship can long exist, unless Japan ceases to act toward Americans as if they were her enemies.... "Many of the large ships-of-war destined to visit Japan have not yet arrived in these seas, though they are hourly expected; and [the United States has], as an evidence of [its] friendly intentions.... brought but four of the smaller ones, designing, should it become necessary, to return to Edo [Tokyo] in the ensuing spring with a much larger force." Commodore Matthew C. Perry to the emperor of Japan, letter, 1853 The excerpt best supports the conclusion that in the 1850s, the United States government a attempted to monopolize Japanese commerce and to exclude the participation of Europeans in trade b was willing to intimidate Asian countries like Japan to secure economic opportunities c was interested in making the government of Japan more democratic d sought to prevent Japan from forming a naval alliance with the British empire

b was willing to intimidate Asian countries like Japan to secure economic opportunities

Between 1860 and 1900, railroads in the United States were a subject to laissez-faire treatment by government. b given government subsidies to open new markets. c nearly shut down by labor unrest and strikes. d resistant to the consolidation of corporate power.

b. given government subsidies to open new markets

*levittown advertisement* During the 1950s, which group most directly challenged the portrayal of American life depicted in the illustration above? a. "Sun Belt" migrants b. Artists and intellectuals c. Political conservatives d. Private-sector businesses

b. Artists and intellectuals

"New France enter[ed] its golden age in the first decades of the eighteenth century. . . . In Louisiana, theIllinois country, and the Great Lakes basin, French cities and villages developed alongside Indian villages. . . . Here, natives and Europeans found that their different goals were complementary. The French posed no demographic threat. . . . The landscape of Indian life had not been seriously altered. The fur trade depended on the integrity of that landscape." Jay Gitlin, historian, "Empires of Trade, Hinterlands of Settlement," 1994 The relationship between American Indians and the French described in the excerpt was most similar to the relationship between American Indians and the a. British b. Dutch c. Portuguese d. Spanish

b. Dutch

*graph showing increase in American Birthrate from 1940-1960* The increase in births best reflects which of the following trends from 1945 to 1960? a. White resistance toward desegregation b. Economic growth and social mobility c. Increased immigration from southern and eastern Europe d. The Great Migration of black Americans out of the South

b. Economic growth and social mobility

"For in your time we have the opportunity to move...upward to the Great Society. The Great Society rests on abundance and liberty for all. It demands an end to poverty and racial injustice, to which we are totally committed in our time....So I want to talk to you today about three places where we begin to build the Great Society—in our cities, in our countryside, and in our classrooms....There are those timid souls who say this battle cannot be won, that we are condemned to a soulless wealth. I do not agree. We have the power to shape the civilization that we want. But we need your will, your labor, your hearts, if we are to build that kind of society." President Lyndon Johnson, Commencement Address at the University of Michigan, 1964 The initiatives outlined above are most similar to the legislative goals of which president? a. Abraham Lincoln b. Franklin Roosevelt c. Ronald Reagan d. Woodrow Wilson

b. Franklin Roosevelt

*Map displaying Columbian Exchange & products traded between Europe, Africa, and the Americas* What was the major effect of the introduction of sugar to the New World? a. It turned North America into a global leader in sugar production. b. It led to the development of race-based labor systems. c. It made Europeans turn aways from trade with Africa. d. It led to the integration of Native American economies into a global system.

b. It led to the development of race-based labor systems.

"The truth of the matter is that Europe's requirements for the next 3 or 4 years of foreign food and other essential products -- principally from America -- are so much greater than her present ability to pay that she must have substantial additional help, or face economic, social, and political deterioration of a very grave character. The remedy lies in breaking the vicious circle and restoring the confidence of the European people in the economic future of their own countries and of Europe as a whole. ... It is logical that the United States should do whatever it is able to do to assist in the return of normal economic health in the world, without which there can be no political stability and no assured peace. Our policy is directed not against any country or doctrine but against hunger, poverty, desperation, and chaos. Its purpose should be the revival of working economy in the world so as to permit the emergence of political and social conditions in which free institutions can exist." -- George C. Marshall, remarks made at Harvard University on June 5, 1947 What argument best articulates the idea that the adoption of the policy suggested in this speech should be seen as a turning point in U.S. foreign policy? a. Providing military and economic aid placed the U.S. in the role of global partner, rather than as a superpower. b. It signaled a major U.S. commitment to a foreign policy of collective security, in which economic security was recognized as a major component. c. The U.S. military was increasingly perceived as weak and ineffectual, because of its focus on economic aid rather than military defense. d. Domestic opposition to the costs of the plan led to an extended period of practical isolation from foreign affairs.

b. It signaled a major U.S. commitment to a foreign policy of collective security, in which economic security was recognized as a major

"We conclude that in the field of public education the doctrine of 'separate but equal' has no place. Separate educational facilities are inherently unequal." - Chief Justice Earl Warren, writing the unanimous opinion of the United States Supreme Court in Brown v. Board of Education of Topeka, 1954 The Brown decision reversed which of the following earlier decisions? a. Dred Scott v. Sandford, which proclaimed that slaves could not be citizens b. Plessy v. Ferguson, which endorsed racial segregation laws c. Worcester v. Georgia, which established that the federal government rather than individual states had authority in American Indian affairs d. Marbury v. Madison, which established the principle of judicial review

b. Plessy v. Ferguson, which endorsed racial segregation laws

"The truth of the matter is that Europe's requirements for the next 3 or 4 years of foreign food and other essential products -- principally from America -- are so much greater than her present ability to pay that she must have substantial additional help, or face economic, social, and political deterioration of a very grave character. The remedy lies in breaking the vicious circle and restoring the confidence of the European people in the economic future of their own countries and of Europe as a whole. ... It is logical that the United States should do whatever it is able to do to assist in the return of normal economic health in the world, without which there can be no political stability and no assured peace. Our policy is directed not against any country or doctrine but against hunger, poverty, desperation, and chaos. Its purpose should be the revival of working economy in the world so as to permit the emergence of political and social conditions in which free institutions can exist." -- George C. Marshall, remarks made at Harvard University on June 5, 1947 Which broad U.S. policy goal of the mid-twentieth century was supported by the implementation of the policy proposed in this speech? a. Reduction in the role of the federal government in relation to state authority b. Political and economic resistance to the spread of Soviet communist influence abroad c. Widespread reduction of poverty through active government intervention abroad d. Increased equality of opportunity through expansion of civil and human rights abroad

b. Political and economic resistance to the spread of Soviet communist influence abroad

"We are the people of this generation, bred in at least modest comfort, housed now in universities, looking uncomfortably to the world we inherit. "When we were kids, the United States was the wealthiest and strongest country in the world; the only one with the atom bomb, the least scarred by modern war, an initiator of the United Nations. . . . "As we grew, however, our comfort was penetrated by events too troubling to dismiss. . . . "The conventional moral terms of the age, the politician moralities—'free world,' 'people's democracies' reflect realities poorly, if at all, and seem to function more as ruling myths than as descriptive principles. . . ."The bridge to political power, though, will be built through genuine cooperation, locally, nationally, and internationally, between a new left of young people, and an awakening community of allies." Students for a Democratic Society (SDS), Port Huron Statement, 1962 What earlier movement is most consistent with the sentiments expressed in the quote above? a. Manifest Destiny b. Populism c. The second party system d. Social Darwinism

b. Populism

"An act for the better regulating the government of the province of the Massachusetts's Bay, in New England. I. ...Be it therefore enacted by the King's most excellent Majesty ... That the charter, granted ... to the inhabitants of the said province of the Massachusetts's Bay... be revoked, and is hereby revoked and made void and of none effect; ...And that ... the council, or court of assistants of the said province ... shall be thereunto nominated and appointed by his Majesty, III. And be it further enacted by the authority aforesaid, ... it shall and may be lawful for his Majesty's governor ... to nominate and appoint, ... and also to remove, without the consent of the council, all judges of the inferior courts of common pleas, commissioners of Oyer and Terminer, the attorney general, provosts, marshals, justices of the peace, and other officers to the council or courts of justice belonging ..." -- Massachusetts Government Act (one of the so-called Intolerable or Coercive Acts) Which of the following ideals most justified in many American colonists' view their general political response to laws such as those above? a. Separation of religious affairs from political management b. Rule by the people, under a republican government c. Establishment of a social contract to justify rule by a single authority d. Belief in the value of human reason over superstition

b. Rule by the people, under a republican government

"A philosopher once said, 'Let me make all the ballads of a country and I care not who makes its laws.' He might with more propriety have said, let the ladies of a country be educated properly, and they will not only make and administer its laws, but form its manners and character. ...The influence of female education would be still more extensive and useful in domestic life. ...Children would discover the marks of maternal prudence and wisdom in every station of life; for it has been remarked that there have been few great or good men who have not been blessed with wise and prudent mothers." --Benjamin Rush, physician, educator, and writer, Essays: Literary, Moral, and Philosophical, published in 1798 Which of the following most contributed to the rise of the ideals as expressed in the above excerpt? a. Sentimentality toward and mimicking of traditional English aristocratic views of women b. The Enlightenment ideal of natural rights c. Entrance of women into reform movements such as abolitionism d. Cultural diffusion of social ideas arising from increased trade around the world

b. The Enlightenment ideal of natural rights

"Our people are losing faith, not only in government itself but in their ability as citizens to serve as the ultimate rulers and shapers of our democracy. "We were sure that ours was a nation of the ballot, not the bullet, until the murders of John Kennedy and Robert Kennedy and Martin Luther King Jr. We were taught that our armies were always invincible and our causes were always just, only to suffer the agony of Vietnam. We respected the Presidency as a place of honor until the shock of Watergate. "We remember when the phrase 'sound as a dollar' was an expression of absolute dependability, until ten years of inflation began to shrink our dollar and our savings. We believed that our Nation's resources were limitless until 1973, when we had to face a growing dependence on foreign oil. —Jimmy Carter, Public Papers of the Presidents of the United States, 1979 Which of the following would most strongly support Carter's contention in his phrase "we were taught that our armies were always invincible"? a. The secret expansion of the Vietnam War into Cambodia by Nixon b. The defeat of South Vietnam by North Vietnam in 1975 c. The decision by newspapers to publish the Pentagon Papers d. The killing of protestors at Kent State and Jackson State Universities

b. The defeat of South Vietnam by North Vietnam in 1975

"What began as a protest movement is being challenged to translate itself into a political movement. It is now concerned not merely with removing the barriers to full opportunity but with achieving the fact of equality. From sit-ins and freedom rides we have gone into rent strikes, boycotts, community organization, and political action. As a consequence of this natural evolution, the Negro today finds himself stymied by obstacles of far greater magnitude than the legal barriers he was attacking before: automation, urban decay, de facto school segregation." — Bayard Rustin, "From Protest to Politics," 1965 Statistics on which of the following could best be used to support the argument made in the excerpt? a. The incidences of lynching by year b. The incomes of African Americans as compared to those of White people by year c. The number of African Americans holding political office by year d. The percentage of African Americans registered to vote by year

b. The incomes of African Americans as compared to those of White people by year

"The colonizers brought along plants and animals new to the Americas, some by design and others by accident. Determined to farm in a European manner, the colonists introduced their domesticated livestock—honeybees, pigs, horses, mules, sheep, and cattle—and their domesticated plants, including wheat, barley, rye, oats, grasses, and grapevines. But the colonists also inadvertently carried pathogens, weeds, and rats. . . . In sum, the remaking of the Americas was a team effort by a set of interdependent species led and partially managed (but never fully controlled) by European people." - Alan Taylor, historian, American Colonies, 2001 The patterns described in the excerpt most directly foreshadowed which of the following developments? a. The emergence of racially mixed populations in the Americas b. The population decline in Native American societies c. The gradual shift of European economies from feudalism to capitalism d. The spread of maize cultivation northward from present-day Mexico into the American Southwest

b. The population decline in Native American societies

"Wherefore, security being the trued design and end of government, it unanswerably follows that wheatever form thereof appears most likely to ensure it to us, with the least expense, and greatest benefit, is preferable to all others. ... Here too is the design and end of governmnet, Freedom and Security." Thomas Paine, Common Sense, 1776 "[G]overnments are instituted among Men, deriving their just powers from the consent of the governed. That whenever any Form of Government becomes destructive of these ends, it is the Right of the People to alter or to abolish it, and institute new Government, laying its foundations on such principles and organizing its powers in such form, as to them shall seem mostlikely to effect their Safety and Happiness." Thomas Jefferson, Declaration of Independence, 1776 The principles expressed by Paine and Jefferson best account for which of the following features of the United States during and immediately after the American Revolution? a. The growth of conflict between wealthy elites and poor farmers and laborers b. The relatively limited powers of the Articles of Confederation c. The development of factions and nascent political parties d. The rapid expansion of frontier settlements

b. The relatively limited powers of the Articles of Confederation

"My purpose is not to persuade children from their parents; men from their wives; nor servants from their masters: only, such as with free consent may be spared: But that each [English] parish, or village, in city or country, that will but apparel their fatherless children, of thirteen or fourteen years of age, or young married people, that have small wealth to live on; here by their labor may live exceeding well: provided always that first there be sufficient power to command them, . . . and sufficient masters (as carpenters, masons, fishers, fowlers, gardeners, husbandmen, sawyers, smiths, spinsters, tailors, weavers, and such like) to take ten, twelve, or twenty, or as is their occasion, for apprentices. The masters by this may quickly grow rich; these [apprentices] may learn their trades themselves, to do the like; to a general and an incredible benefit for king, and country, master, and servant." - John Smith, English adventurer, A Description of New England, 1616 The excerpt would be most useful to historians as a source of information about which of the following? a. The interaction of English colonial settlers with native populations in the early seventeenth century b. The role that appeals and advertising played in encouraging men and women to participate in colonization efforts c. The harsh realities of life in the early seventeenth-century American colonies, including illness, high mortality rates, and starvation d. The nature of master and apprentice relationships in England in the early seventeenth century

b. The role that appeals and advertising played in encouraging men and women to participate in colonization efforts

Where the money is to come from which will defray this enormous annual expense of three million sterling [for the American Revolution], and all those other debts, I know not...Certain I am that our commerce and agriculture, the two principal sources of our wealth, will not support such an expense. The whole of our exports from the Thirteen United Colonies, in the year 1769, amounted only to £2,887,898 sterling; which is not so much, by near half a million, as our annual expense would be were we independent of Great Britain. Those exports, with no inconsiderable part of the profits arising from them, it is well known, centered finally in Britain to pay the merchants and manufacturers there for goods we had imported thence—and yet left us still in debt! What then must our situation be, or what the state of our trade, when oppressed with such a burden of annual expense! When every article of commerce, every necessary of life, together with our lands, much be heavily taxed to defray the expense! - Charles Inglis, Anglican church minister in New York City, The Costs of Revolution, 1776. The excerpt above best serves as evidence for which of the following statements about the American War for Independence? a. Americans were united in their support for independence due to economic reasons. b. There were diverse reasons for and against the revolution. c. Most Americans were fighting for the patriot cause. d. Americans were generally divided over supporting the Native Americans.

b. There were diverse reasons for and against the revolution.

*Picture displaying the expansion of British colonies* The change in settlement patterns from 1700 to 1775 had which of the following effects? a. a decrease in the coastal population b. an increase in conflicts between British settlers and American Indians c. an increase in trade with French Canada d. a decrease in the economic importance of slavery and other forms of coerced labor

b. an increase in conflicts between British settlers and American Indians

"I said everything to them I could to divert them from their idolatries, and draw them to a knowledge of God our Lord. Moctezuma replied, the others assenting to what he said, that they had already informed me they were not the aborigines of the country, but that their ancestors had emigrated to it many years ago; and they fully believed that after so long an absence from their native land, they might have fallen into some errors; that I having more recently arrived must know better than themselves what they ought to believe; and that if I would instruct them in these matters, and make them understand the true faith, they would follow my directions, as being for the best. Afterwards, Moctezuma and many of the principal citizens remained with me until I had removed the idols, purified the chapels, and placed the images in them, manifesting apparent pleasure." - Letter from Hernán Cortés to Charles V, Holy Roman Emperor and King of Spain, on his interaction with the Mexica (Aztecs), 1520 The interaction between Cortés and Moctezuma most strongly demonstrates Cortés' a. promotion of long-distance trade b. desire for increased power and status c. support for the European shift from feudalism to capitalism d. consolidation of plantation-based agriculture the

b. desire for increased power and status

The controversy highlighted in the cartoon above was most directly a result of a Radical Republicans' efforts to establish a base for their party in the South. b determined Southern resistance to Northern efforts to change its culture. c temporary rearrangements in the relationships between black and white people in the South. d unresolved questions about the power of the federal government.

b. determined Southern resistance to Northern efforts to change its culture.

"It is now fourteen weeks since the revolution of government here. Future consequences we are ignorant of, yet we know that, at present we are eased of the great oppressions that we groaned under...making the arbitrary commission of Sir Edmund Andros null and void in the law;...although some could not advise to [recommend] the enterprise, yet [all] are hopeful that we shall not be greatly blamed, but shall have a pardon granted for any error the law will charge us with in this matter. ... Nature has taught us self-preservation....Our great remoteness from England denies us the opportunity of direction for the regulation of ourselves in all emergencies, nor have we the means to know the laws and customs of our nation....We have always endeavored to prove ourselves loyal to the Crown of England...and we are not without hopes but that we shall receive from Their Royal Majesties the confirmation of our charter, with such addition of privileges as may advance the revenue of the Crown, and be an encouragement to Their Majesties' subjects here." Letter from Thomas Danforth of Massachusetts to a colleague in London, 1689 The excerpt above is best understood in the context of the a. emergence of a transatlantic print culture. b. efforts by England to integrate the colonies into an imperial structure. c. spread of European Enlightenment ideas. d. development of a close-knit homogeneous society in the New England colonies.

b. efforts by England to integrate the colonies into an imperial structure.

*Picture displaying the expansion of British colonies* The change in settlement patterns from 1700 to 1775 best explains the a. significant proportion of colonists who remained loyal to Great Britain during the American Revolution b. growth of social tensions between backcountry settlers and coastal elites. c. development of economic differences between the northern and southern colonies d. colonists' difficulties in effectively resisting the British military during the American Revolution

b. growth of social tensions between backcountry settlers and coastal elites.

"What induced [American] Indians to go out of their way to trap beaver and trade the skins for glass beads, mirrors, copper kettles, and other goods? . . . Recent scholarship on [American] Indians? ' motives in this earliest stage of the trade indicates that they regarded such objects as the equivalents of the quartz, mica, shell, and other sacred substances that had formed the heart of long-distance exchange in North America for millennia. . . . While northeastern [American] Indians recognized Europeans as different from themselves, they interacted with them and their materials in ways that were consistent with their own customs and beliefs. " Neal Salisbury, historian, "The Indians' Old World: Native Americans and the Coming of Europeans, " 1996 A direct result of European exploration of North America during the 1500s and early 1600s was the a. smaller role played by Spain in European affairs b. introduction of new animals and crops to North America c. large-scale migration of American Indians to Europe d. decline of African slavery in North America

b. introduction of new animals and crops to North America

"Friday June 15th, 1787 Mr. Patterson laid before the Convention the plan which he said several of the duputations withsed to be submitted in place of that proposed by Mr. Randolph. 1. Resolved that the articles of Conederation ought to be so revised, corrected and enlarged, as to render the federal Constitution adequate to the exigences of Government and the preservation of the Union 2. Resolved that ... Congress...be authorized to pass acts for raising a revenue ... [and] to pass Acts for the regulation of trade and commerce as well with foreign nationa as with each other. 4. Resolved that the United States in Congress be authorized to elect a federal Executive 5. Resolved that a federal Judiciary be established to consist of a supeme [court] of Judges of which to be appointed by the Executive, and to hold their offices during good behavior." - Notes from the Constitutional Convention as recored by James Madison, June 15th, 1787. The except above best demonstrates the concept of which of the following? a. limited government b. separation of power c. divine right d. federalism

b. separation of power

Resolutions proposed by Ms. Randolph in Convention May 29, 1787. 2. Resolved that the rights of suffrage in the National Legislature ought to be proportioned ... to the number of free inhabitants. 3. Resolved that the National Legislature ought to consist of two branches. 4. Resolved that the memers of hte first branch of the National Legislature ought to be elected by the people of the several States... 5. Resolved that the members of the second branch of the National Legislature ought to be elected by those of the first ... nominated by the individual Legislatures. 7. Resolved that a National Executive be instituted; ... [and granted] a general authority to execute the National laws. 9. Resolved that a National Judiciary be established to consist of one or more supreme [courts] ...to hold their offices during good behavior. Notes from the Constitutional Convention as recorded by James Madison, May 29, 1787. The excerpt above best demonstrates the concept of which of the following? a. limited government b. separation of powers c. divine right d. federalism

b. separation of powers

"Young ladies in town, and those that live round, Let a friend at this season advise you: Since money's so scarce, and times growing worse, Strange things may soon hap and surprise you; First then, throw aside your high top knots of pride, Wear none but your own country linen, Of Economy boast, let your pride be the most To show clothes of your own make and spinning." The Daughters of Liberty Urge Americans to Boycott British Goods According to the excerpt, groups of patriotic women who called themselves "Daughters of Liberty" met to do which of the following? a. bake goods to sell for their household needs b. spin their own yarn and make their own cloth c. shop for new linen and get their hair curled d. sip tea and gossip about their neighbors

b. spin their own yarn and make their own cloth

*nine blind men and the tiger cartoon* The image most directly illustrates a United States foreign policy that emphasized a. containing communism in Eastern Europe b. stopping the spread of communism in Africa, Asia, and the Middle East c. placing greater importance on human rights d. supporting the Non-Aligned Movement among Middle Eastern and Asian countries

b. stopping the spread of communism in Africa, Asia, and the Middle East

"ARTICLE IV. It is likewise agreed that the Western boundary of the United States which separates them from the Spanish Colony of Louisiana, is in the middle of the channel or bed of the River Mississippi from the Northern boundary of the said States to the completion of the thirty first degree of latitude North of the Equator; and his Catholic Majesty has likewise agreed that the navigation of the said River in its whole breadth from its source to the Ocean shall be free only to his Subjects, and the Citizens of the United States, unless he should extend this privilege to the Subjects of other Powers by special convention...in consequence of the stipulations contained in the IV. article his Catholic Majesty will permit the Citizens of the United States for the space of three years from this time to deposit their merchandise and effects in the Port of New Orleans, and to export them from thence without paying any other duty than a fair price for the hire of the stores, and his Majesty promises either to continue this permission if he finds during that time that it is not prejudicial to the interests of Spain, or if he should not agree to continue it there, he will assign to them on another part of the banks of the Mississippi an equivalent establishment." Pinckney's Treaty, 1796 The excerpt from Pinckney's Treaty most directly reflects which of the following? a. the Spanish refusal to sell the Louisiana territory to the Americans b. the Spanish agreement to open up the Mississippi River and the port of New Orleans to U.S. commerce c. the conflict between Catholic Spanish and Protestant U.S. cultures d. the Spanish negotiations for the annexation of Florida

b. the Spanish agreement to open up the Mississippi River and the port of New Orleans to U.S. commerce

"Had these principles been adhered to, we should not have been brought to this alarming transition, from a confederacy to a consolidated government...Here is a resolution as radical as that which separated us from Great Britain. It is radical in this transition; our rights and privileges are endangered, and the sovereignty of the states will be relinquished: and cannot we plainly see that this is actually the case? The rights of conscience, trial by jury, liberty of the press, all your immunities and franchises, all pretensions to human rights and privileges are rendered insecure, if not lost, by this change, so loudly talked of by some, and inconsiderately by others. Is this tame relinquishment of rights worthy of freemen?...Guard with jealous attention the public liberty" Patrick Henry, "Speech against the Federal Constitution," June 5, 1788 Which of the following was the most direct effect of the sentiments expressed in the excerpt? a. the eruption of the Nullification Crisis b. the addition of constitutional amendments known as the Bill of Rights c. the development of the doctrine of judicial review d. the invention of the Electoral College

b. the addition of constitutional amendments known as the Bill of Rights

"I. Resolved, that the several States composing the United States of America... delegated to [the national] government certain definite powers, reserving each State to itself, the residuary mass of right to their own self government.... II....therefore the act of Congress, passed on the 14th day of July, 1798 ... [is] altogether void... and that the power to create, define, and punish such other crimes is reserved solely and exclusively to the respective States.... VIII....that every State has a natural right in cases not within the compact, to nullify of their own authority all assumptions of power by others within their limits." - Thomas Jefferson, Kentucky Resolutions, 1798 The ideas expressed in the passage are best understood as part of the debate in American politics over a. racial justice b. the legitimacy of the U.S. Constitution c. the relative power of the national government d. economic equality and the class system

b. the legitimacy of the U.S. Constitution

*Picture of slave ship layout* Which of the following was the most direct effect of the harsh conditions of the Middle Passage? a. advances in medicine evoked by the exposure to African illnesses b. the slaves' high death rate due to the spread of disease. c. a decrease in piracy raids on the Atlantic shipping d. a rise in the number of slaves imported from the Middle East

b. the slaves' high death rate due to the spread of disease.

"The town is now quite filled with inhabitants, which in regard to their country, religion, and trade, are very different from each other...Every one who acknowledges God to be the Creator...is at liberty to settle, stay, and carry on his trade here, be his religious principles ever so strange. No one is here molested on account of the erroneous principles of the doctrine which he follows...On a careful consideration of what I have already said, it will be easy to conceive how this city should rise so suddenly from nothing into such grandeur and perfection...It has not been necessary to force people to come and settle here; on the contrary, foreigners of different languages have left their country, houses, property, and relations, and ventured over wide and stormy seas, in order to come hither." Peter Kalm, A Swedish Visitor Tells About Philadelphia, 1748 Pennsylvania's establishment as a proprietary colony and a tolerant refuge for a persecuted religious group had the most in common with which of the following? a. the establishment of the colony of Georgia b. the establishment of the colony of Maryland c. the establishment of the colony of Virginia d. the establishment of the Massachusetts Bay Colony

b: the establishment of the colony Maryland

"As [political leader Henry] Clay envisioned it [in the 1820s], the American System constituted the... basis for social improvement. ... Through sale of its enormous land holdings, the federal government could well afford to subsidize internal improvements. By levying protective tariffs, the government should foster the development of American manufacturing and agricultural enterprises that, in their infancy, might not be able to withstand foreign competition. The promotion of industry would create a home market for agricultural commodities, just as farms provided a market for manufactured products." Daniel Walker Howe, historian, What Hath God Wrought: The Transformation of America, 1815-1848, published in 2007 Based on the excerpt, which of the following groups would have been most likely to oppose Henry Clay's ideas? a. Antebellum reformers b. Abolitionists c. Members of the Democratic Party d. Members of the Whig Party

c

"As [political leader Henry] Clay envisioned it [in the 1820s], the American System constituted the... basis for social improvement. ... Through sale of its enormous land holdings, the federal government could well afford to subsidize internal improvements. By levying protective tariffs, the government should foster the development of American manufacturing and agricultural enterprises that, in their infancy, might not be able to withstand foreign competition. The promotion of industry would create a home market for agricultural commodities, just as farms provided a market for manufactured products." Daniel Walker Howe, historian, What Hath God Wrought: The Transformation of America, 1815-1848, published in 2007 The ideas described in the excerpt contributed most directly to which of the following? a. Large-scale European immigration to the South b. Business leaders consolidating corporations into trusts and holding companies c. More Americans producing goods for national markets. d. A decline in the internal slave trade

c

"British cruisers have been in the continued practice of violating the American flag on the great highway of nations, and of seizing and carrying off persons sailing under it, not in the exercise of a belligerent right founded on the law of nations against an enemy, but of a municipal [internal] prerogative over British subjects. British jurisdiction is thus extended to neutral vessels... " James Madison, 1812 Which of the following is James Madison speaking of? a. The British still believing the citizens of the U.S. are British citizens b. The taking of the American flag by the British c. Impressment of American sailors d. The British not trading fairly

c

"I must confine myself to few examples, but am ready to furnish other and more complete details, if required. If my pictures are displeasing, coarse, and severe, my subjects must be recollected, offer no tranquil, refined, or composing features. The condition of human beings, reduced to the extremest states of degradation and misery, cannot be exhibited in softened language, or adorn a polished page. I proceed, gentlemen, briefly to call your attention to the present state of insane persons, confined within this Commonwealth, in cages, closets, cellars, stalls, pens! Chained, naked, beaten with rods, and lashed into obedience." Dorothea Dix, 1843 Many reforms of the antebellum period were caused by a. a demand for federal government reform programs. b. a fear of growing democracy. c. both the industrial and democratic growth of the country. d. a fear of too much power of the national government.

c

"Joseph Smith... came from nowhere. Reared in a poor Yankee farm family, he had less than two years of formal schooling and began life without social standing or institutional backing. His family rarely attended church. Yet in the fourteen years he headed the Church of Jesus Christ of Latter-day Saints, Smith created a religious culture that survived his death, flourished in the most desolate regions of the United States, and continues to grow worldwide. . . . In 1830 at the age of twenty-four, he published the Book of Mormon. . . .He built cities and temples and gathered thousands of followers before he was killed at age thirty-eight." Richard Lyman Bushman, historian, Joseph Smith Rough Stone Rolling: A Cultural Biography of Mormonism's Founder, 2005 The developments described in the excerpt best illustrate which of the following? a. The ideal of republican motherhood b. The abolitionist movement c. The Second Great Awakening d. The increased acceptance of Catholics

c

"No Negro shall leave the place at any time without my permission...No Negro shall be allowed to marry out of the plantation...No rule I have stated is of more importance than that relating to negroes marrying out of the plantation...It creates a feeling of independence, from being, of right, out of the control of the masters for a time. Never allow any man to talk to your Negroes, nothing more injurious." Rules of Highland Plantation, 1838 In the decades before the Civil War, slave owners a. empowered by the slave revolts became less harsh toward their slaves. b. formed a union to prevent the abolitionists from gaining power. c. created a set of proslavery arguments justifying slavery as a positive good. d. provided separate villages for the slaves to come together for worship and marriage.

c

"On the subject of slavery . . . I will be as harsh as truth, and as uncompromising as justice. . . . On this subject, I do not wish to think, or speak, or write, with moderation. . . . I am in earnest—I will not equivocate—I will not excuse—I will not retreat a single inch—AND I WILL BE HEARD.' — William Lloyd Garrison, first issue of abolitionist newspaper The Liberator, January 1831 A person who agreed with Garrison's views expressed in the excerpt would most likely have advocated a. encouraging free Black people to migrate to Africa b. the preservation of the Missouri Compromise c. immediate emancipation of enslaved people d. paying slaveholders to gradually free enslaved people

c

"[I promise]...to demonstrate in the course of...my Appeal...that we Coloured People of these United States, are, the most wretched, degraded and abject set of beings that ever lived since the world began, down to the present day, and that the white Christians of America, who hold us in slavery, (or, more properly speaking, pretenders to Christianity,) treat us more cruel and barbarous than any Heathen nation did any people whom it had subjected, or reduced to the same condition....I advance it therefore to you...as an unshaken and forever immoveable fact, that your full glory and happiness, as well as all other coloured people under Heaven, shall never be fully consummated, but with the entire emancipation of your enslaved brethren all over the world." David Walker, Appeal to the Coloured Citizens of the World, 1829David Walker, Walker's Appeal in Four Articles; Together with a Preamble, To the Coloured Citizens of the World, but in Particular, and Very Expressly, to Those of the United States of America Written in Boston, State of Massachusetts, September 28, 1829 (Boston: Revised and Published by David Walker, 1830). The arguments in the excerpt above are best understood in the context of a. continued restrictions on African American citizenship in Northern states. b. sectional tensions over the institution of slavery. c. the emergence of African American abolitionist movements. d. the growth of the internal slave trade in the United States.

c

"[T]he condition of the African race throughout all the States where the ancient relation between the two [races] has been retained enjoys a degree of health and comfort which may well compare with that of the laboring population of any country in Christendom; and, it may be added that in no other condition, or in any other age or country, has the Negro race ever attained so high an elevation in morals, intelligence, or civilization." - John C. Calhoun, political leader, 1844 Which of the following groups would have been most likely to support Calhoun's views expressed in the excerpt? a. Members of nativist political parties b. Members of the Whig Party c. Southern landowners d. Northern industrialists

c

The sentiments such as those depicted in the cartoon above most directly contributed to which of the following? a The willingness of abolitionists to use violence to achieve their goals b Repeated attempts at political compromise c Breakdown in trust between sectional leaders d The secession of Southern states

c Breakdown in trust between sectional leaders

"[G]ranting all their mistakes, the radical governments were by far the most democratic the South had ever known. They were the only governments in southern history to extend to Negroes complete civil and political equality, and to try to protect them in the enjoyment of the rights they were granted." — Kenneth M. Stampp, historian, The Era of Reconstruction, 1865-1877 Which of the following provides the best evidence in support of the argument in the excerpt? a Divisions within the women's movement over the Fourteenth and Fifteenth Amendments b Political corruption in Southern state governments c Changes in voting patterns and office holding that occurred during Reconstruction d The emergence of the sharecropping system during Reconstruction

c Changes in voting patterns and office holding that occurred during Reconstruction

"Excepting only Yosemite, Hetch Hetchy is the most attractive and wonderful valley within the bounds of the great Yosemite National Park and the best of all the camp grounds. People are now flocking to it in ever-increasing numbers for health and recreation of body and mind. Though the walls are less sublime in height than those of Yosemite, its groves, gardens, and broad, spacious meadows are more beautiful and picturesque. . . . Last year in October I visited the valley with Mr. William Keith, the artist. He wandered about from view to view, enchanted, made thirty-eight sketches, and enthusiastically declared that in varied picturesque beauty Hetch Hetchy greatly surpassed Yosemite. It is one of God's best gifts, and ought to be faithfully guarded." John Muir, Century Magazine, 1909 Muir's position regarding wilderness was most strongly opposed by which of the following? a Members of the Progressive movement b Urban political bosses c Companies involved in natural resource extraction d American Indians living on reservations

c Companies involved in natural resource extraction

Which of the following was most likely a significant cause of the sentiments depicted in the cartoon above? a Environmentalist lobbying for greater conservation of natural resources. b Progressive action to investigate political corruption in the early 20th century. c Corporate consolidation and abuse of power during the Gilded Age. d Demand by Progressive reformers to create a stronger financial regulatory system.

c Corporate consolidation and abuse of power during the Gilded Age.

Whoever, when the United States is at war, shall willfully make or convey false reports or false statements with intent to interfere with the operation or success of the military or naval forces of the United States, or to promote the success of its enemies, or shall willfully make or convey false reports, or false statements, ...or incite insubordination, disloyalty, mutiny, or refusal of duty, in the military or naval forces of the United States...hall be punished by a fine of not more than $10,000 or imprisonment for not more than 20 years, or both. The Sedition Act of 1918 What prompted the passage of this law? a A rise in labor disputes due to the growth of the IWW b Large numbers of citizens whose loyalty the government questioned c Draft riots in prominent cities such as New York that were led by Irish immigrants d Unrestricted submarine warfare

c Draft riots in prominent cities such as New York that were led by Irish immigrants (i think)

Commodore Perry and his Entourage Arrive at the "Treaty House" in Yokohama, March 8, 1854 The image illustrated above best reflects which continuities in U.S. History? a The effectiveness of cotton diplomacy in opening Asian markets b An isolationist foreign policy c Expansion of international trade and diplomacy d Expansion of white supremacy and superior American values

c Expansion of international trade and diplomacy

Which statement below accurately compares German and Irish immigrants in the United States? a Both Irish and German immigrants were loyal supporters of the Whig party. b Both Irish and German immigrants were mainly temporary visitors to the United States and returned to their homelands after a few years. c German immigrants typically settled in the Midwest while Irish immigrants settled in northeastern cities. d German immigrants routinely worked in factories while Irish immigrants were primarily farmers.

c German immigrants typically settled in the Midwest while Irish immigrants settled in northeastern cities.

Let us know the modern city in its weakness and wickedness, and then seek to rectify and purify it until it shall be free at least from the grosser temptations which now beset the young people who are living in its tenement houses and working in its factories." - Excerpt from Jane Addams, The Spirit of Youth and the City Streets, 1909. Which of the following would have been most likely to support Jane Addam's view expressed in the excerpt? a John Muir b Know-Nothing Party c Jacob Riis d Andrew Carnegie

c Jacob Riis

The painting by George Bellows above best demonstrates which of the following developments in American society during the Gilded Age? a Large urban cities were hubs for entertainment venues due to the increased immigrant populations. b Middle class settlement house workers and temperance movement workers sharply criticized the violence of boxing. c New sports such as football, boxing and basketball developed as Americans had more leisure time. d Entertainments like boxng became the sport of the wealthy philanthropists.

c New sports such as football, boxing and basketball developed as Americans had more leisure time.

"I believe that progressivism was a radical movement thought not by the common measures of economic and political radicalism...Progressives were radical in their conviction that other social classes must be transformed and in their boldness in going about the business of that transformation...The sweep of progressivism was remarkable but because the progressive agenda was so often carried out in settlement houses, churches, and school rooms, in rather unassuming day-to-day activities the essential audacity of the enterprise can be missed. Progressivism demanded a social transformation that remains at once profoundly impressive and profoundly disturbing a century later." - Michael McGerr, A Fierce Discontent: The Rise and Fall of the Progressive Movement in American, 1870 - 1920. 2003. Which of the following activities from the middle of the 19th century most closely resembles the Progressive Era reform that McGerr describes? a Calls for the annexation of Texas b Efforts by natives to restrict immigration c Participation by women in moral reform efforts d Removal of American Indians from the Southeast to the West

c Participation by women in moral reform efforts

"I believe that progressivism was a radical movement thought not by the common measures of economic and political radicalism...Progressives were radical in their conviction that other social classes must be transformed and in their boldness in going about the business of that transformation...The sweep of progressivism was remarkable but because the progressive agenda was so often carried out in settlement houses, churches, and school rooms, in rather unassuming day-to-day activities the essential audacity of the enterprise can be missed. Progressivism demanded a social transformation that remains at once profoundly impressive and profoundly disturbing a century later." - Michael McGerr, A Fierce Discontent: The Rise and Fall of the Progressive Movement in American, 1870 - 1920. 2003. Which of the following activities from the middle of the 19th century most closely resembles the Progressive Era reform that McGerr describes? a Removal of American Indians from the Southeast to the West b Calls for the annexation of Texas c Participation by women in moral reform efforts d Efforts by natives to restrict immigration

c Participation by women in moral reform efforts

"We demand a graduated income tax.... Transportation being a means of exchange and a public necessity, the government should own and operate the railroads.... The land, including all the natural sources of wealth, is the heritage of the people, and should not be monopolized for speculative purposes, and alien ownership of land should be prohibited.... [W]e demand a free ballot and a fair count...to every legal voter.... [W]e favor a constitutional provision limiting the office of President and Vice-President to one term, and providing for the election of Senators of the United States by a direct vote of the people." People's (Populist) Party platform, 1892 The ideas of the Populist Party, as expressed in the excerpt, had the most in common with the ideas of the a Civil Rights movement b Whigs in the 1830s c Progressive movement d Federalists in the 1790s

c Progressive movement

"We have conquered many of the neighboring tribes of Indians, but we have never thought of holding them in subjection—never of incorporating them into our Union To incorporate Mexico, would be the very first instance of the kind of incorporating an Indian race; for more than half of the Mexicans are Indians, and the other is composed chiefly of mixed tribes.... Ours, sir, is the Government of a white race. ... [I]t is professed and talked about to erect these Mexicans into a Territorial Government, and place them on an equality with the people of the United States. I protest utterly against such a project." Senator John C. Calhoun, "Conquest of Mexico" speech, 1848 Based on the excerpt, Calhoun would also be most likely to support which of the following? a United States sale of disputed territory b Expanded United States federal authority c Proslavery arguments d Policies favoring immigration

c Proslavery arguments

Section 1. Be it ordained by the police jury of the parish of St. Landry, that no negro shall be allowed to pass within the limits of said parish without special permit in writing from his employer... Section 3... no negro shall be permitted to rent or keep a house within said parish... Section 4...Every negro is required to be in the regular service of some white person or former owner, who shall be held responsible for the conduct of said negro... Section 7...No negro who is not in the military service shall be allowed to carry fire-arms, or any kind of weapons, within the parish... Section 11...It shall be the duty of every citizen to act as a police officer for the detection of offences and the apprehension of offenders, who shall immediately be handed over to the proper captain or chief of patrol." The Louisiana Black Code, 1865 Which 19th-century group would most likely oppose the regulations in the legislation above? a The Supreme Court b Supporters of Social Darwinism c Republican Party members d New international migrants

c Republican Party members

Which of the following provided the strongest justification for the practices shown in the image? a Manifest Destiny b Romantic belief in human perfectibility c Social Darwinism d Socialism and utopianism

c Social Darwinism

"Be it enacted by the Senate and House of Representatives of the United States of America in Congress assembled, That any person who is the head of a family, or who has arrived at the age of twenty-one years, and is a citizen of the United States, or who shall have filed his declaration of intention to become such, as required by the naturalization laws of the United States, and who has never borne arms against the United States Government or given aid and comfort to its enemies, shall, from and after the first January, eighteen hundred and. sixty-three, be entitled to enter one quarter section or a less quantity of unappropriated public lands, upon which said person may have filed a preemption claim, ... at one dollar and twenty-five cents, or less, per acre; ... to be located in a body, in conformity to the legal subdivisions of the public lands." -- The Homestead Act, 1862 Which of these mid-nineteenth century goals did the government policy given here mostly support? a Reconstruction of the Southern economy and society b Creation of a distinctive native-born American identity c Support for rapid economic development of the West d Establishment of a large and cheap immigrant workforce

c Support for rapid economic development of the West

"The question is simply this: can a negro whose ancestors were imported into this country and sold as slaves become a member of the political community formed and brought into existence by the Constitution of the United States, and as such become entitled to all the rights, and privileges, and immunities, guaranteed by that instrument to the citizen, one of which rights is the privilege of suing in a court of the United States in the cases specified in the Constitution?... It is the judgment of this court that it appears... that the plaintiff in error is not a citizen... in the sense in which that word is used in the Constitution." United States Supreme Court Chief Justice Roger B. Taney, Dred Scott v. Sandford, 1857 Which of the following was the most immediate result of the decision in the excerpt? a The United States fought a war with Mexico. b Support grew for the Republican Party. c Tensions over slavery diminished. d Most slave states voted to secede from the Union.

c Tensions over slavery diminished.

One Way Ticket (Langston Hughes) I am fed up With Jim Crow laws, People who are cruel And afraid, Who lynch and run, Who are scared of me And me of them. I pick up my life And take it away On a one-way ticket Gone Up North Gone Out West Gone! The author, Langston Hughes, is most closely identified with which literary movement in American History? a Realism b Modernism c The Harlem Renaissance d Transcendentalism

c The Harlem Renaissance

"Our nation will continue to be steadfast, and patient and persistent in the pursuit of two great objectives. First, we will shut down terrorist camps, disrupt terrorist plans and bring terrorists to justice. And second, we must prevent the terrorists and regimes who seek chemical, biological or nuclear weapons from threatening the United States and the world....Our second goal is to prevent regimes that sponsor terror from threatening America or our friends and allies with weapons of mass destruction....States like these [North Korea, Iran, and Iraq], and their terrorist allies, constitute an axis of evil, arming to threaten the peace of the world. By seeking weapons of mass destruction, these regimes pose a grave and growing danger....And all nations should know: America will do what is necessary to ensure our nation's security." President George W. Bush, State of the Union Address, January 2002 The sentiments expressed in the excerpt above most directly contributed to which of the following? a U.S. peacekeeping missions in Latin America, Asia and Africa b A national sense that the Cold War was finally over c The U.S.-led invasion of Iraq d Debates about the merits of a "military-industrial complex"

c The U.S.-led invasion of Iraq

"[I am] commanded to explain to the Japanese that...[the United States] population has rapidly spread through the country, until it has reached the shores of the Pacific Ocean; that we have now large cities, from which, with the aid of steam vessels, we can reach Japan in eighteen or twenty days; [and] that. ... the Japan seas will soon be covered with our vessels. "Therefore, as the United States and Japan are becoming every day nearer and nearer to each other, the President desires to live in peace and friendship with your imperial majesty, but no friendship can long exist, unless Japan ceases to act toward Americans as if they were her enemies.... "Many of the large ships-of-war destined to visit Japan have not yet arrived in these seas, though they are hourly expected; and [the United States has], as an evidence of [its] friendly intentions.... brought but four of the smaller ones, designing, should it become necessary, to return to Edo [Tokyo] in the ensuing spring with a much larger force." Commodore Matthew C. Perry to the emperor of Japan, letter, 1853 Which of the following was a major United States foreign policy change in the Pacific region at the end of the nineteenth century? a The United States supported the formation of the Kingdom of Hawaii. b The United States acquired new colonial possessions such as the Philippines. c The United States began to accept unrestricted immigration from East Asia. d The United States reduced the size of its navy through international agreements.

c The United States began to accept unrestricted immigration from East Asia. tbh idk if this is right

"To those of my race I would say: 'Cast down your bucket where you are' -- cast it down in making friends with the Southern white man, who is your next-door neighbor. Cast it down in agriculture, mechanics, in commerce, in domestic service. ... No race can prosper till it learns that there is as much dignity in tilling a field as in writing a poem. It is at the bottom of life we must begin, and not at the top." -- Booker T. Washington, Speech in Atlanta, Georgia on September 18, 1895 "They do not expect that the free right to vote, to enjoy civic rights, and to be educated, will come in a moment; they do not expect to see the bias and prejudices of years disappear...but they are absolutely certain that the way for a people to gain their reasonable rights...[and]...respect is not by continually belittling and ridiculing themselves; that, on the contrary, Negroes must insist continually, in season and out of season, that voting is necessary to modern manhood, that color discrimination is barbarism, and that black boys need education as well as white boys...." -- W. E. B. Du Bois, The Souls of Black Folk, 1903 Which of the following view(s) most closely parallels the controversy addressed in the the excerpts above? a The surfacing of the fear of Communism after the Bolshevik Revolution b The desire of Union members to gain greater rights in the workplace c The clashing ideals of Black separatism and non-violent protests d Anti-Immigration sentiments in the early twentieth century

c The clashing ideals of Black separatism and non-violent protests (i think)

"It was in suburbs such as Garden Grove, Orange County [California] . . . that small groups of middle-class men and women met in their new tract homes, seeking to turn the tide of liberal dominance. Recruiting the like-minded, they organized study groups, opened 'Freedom Forum' bookstores, filled the rolls of the John Birch Society, entered school board races, and worked within the Republican Party, all in an urgent struggle to safeguard their particular vision of freedom and the American heritage. In doing so, they became the ground forces of a conservative revival—one that transformed conservatism from a marginal force preoccupied with communism in the early 1960s into a viable electoral contender by the decade's end." — Lisa McGirr, historian, Suburban Warriors: The Origins of the New American Right, 2001 Which of the following historical developments most directly resulted from the trend described in the excerpt? a The emergence of environmental activism in the 1970s b The passage of new immigration laws in 1965 c The election of Ronald Reagan as president in 1980 d The Supreme Court decisions expanding individual freedoms in the 1960s and 1970s

c The election of Ronald Reagan as president in 1980

"Since the days when the fleet of Columbus sailed into the waters of the New World, America has been another name for opportunity, and the people of the United States have taken their tone from the incessant expansion which has not only been open but has even been forced upon them. ...Movement has been its dominant fact, and, unless this training has no effect upon a people, the American energy will continually demand a wider field for its exercise. But never again will such gifts of free land offer themselves. ...each frontier did indeed furnish a new field of opportunity, a gate of escape from the bondage of the past; and freshness, and confidence, and scorn of older society, impatience of its restraints and its ideas, and indifference to its lessons, have accompanied the frontier. What the Mediterranean Sea was to the Greeks, breaking the bond of custom, offering new experiences, calling out new institutions and activities, that, and more, the ever retreating frontier has been to the United States directly, and to the nations of Europe more remotely. And now, four centuries from the discovery of America, at the end of a hundred years of life under the Constitution, the frontier has gone, and with its going has closed the first period of American history." --Frederick Jackson Turner, historian, essay titled "The Significance of the Frontier in American History," first presented in 1893 and later collected in a book of essays by Turner titled The Frontier in American History, ©1920 Which American democratic ideal is Turner arguing was facilitated by the American frontier? a The ideal of religious freedom b The ideal of the establishment of utopian communities c The ideal of social mobility d The ideal of orderly and planned settlement of communities

c The ideal of social mobility

"I feel very little like writing, but you will be wanting to know the whys of course. One of those dreadful prairie fires, accompanied by a hurricane of wind swept through here Tuesday night, the 14th, and took everything but our house and stock. All our hay and oats that Rob and I had worked so hard to get up and stack, harness, saddle, 26 hens and chickens...all burned etc. At least a hundred dollars worth swept away in a few moments....Imagine yourselves for instance with nothing but land, house, and stock—for that's where we are. Not a tree, particle of water, grass, stable, fence or anything else....Eastern people may think us homesteaders are doing a fine thing to get 160 acres of land for nothing—all but the nothing. Oh, the suffering that the poor endure here, and privations you have not the remotest idea of, and poor means nearly all homesteaders." Letter from Mary Abell to her family, 1873 The ideas expressed in the excerpt above most directly reflect which of the following continuities in United States history? a Struggles between business interests and conservationists b The orderly incorporation of new territories into the nation c The migration of peoples seeking new economic opportunities d Threats to American Indian culture and identity

c The migration of peoples seeking new economic opportunities

Alfred R. Waud, " "I stand before you under indictment for the alleged crime of having voted in the last presidential election, without having a lawful right to vote. It shall be my work this evening to prove to you that in thus doing, I not only committed no crime, but instead simply exercised my citizen's right, guaranteed to me and all United States citizens by the National Constitution beyond the power of any State to deny....If once we establish the false principle that United States citizenship does not carry with it the right to vote in every state in this Union, there is no end to the petty tricks and cunning devices which will be attempted to exclude one and another class of citizens from the right of suffrage....Establish this precedent, admit the State's right to deny suffrage, and there is no limit to the confusion, discord, and disruption that may await us. There is and can be but one safe principle of government—equal rights to all." Susan B. Anthony, Speech before the Court, 1873 Which of the following was a major consequence of the ideas expressed in the passage above? a Supreme Court decisions began stripping away the civil rights of women. b Short-term successes opened up some political opportunities to women that previously had not existed. c The movement for women's suffrage was temporarily halted in order to concentrate on granting African Americans and immigrants equal protection under the laws. d Some women's equality activists abandoned the movement to grant and protect African American rights.

c The movement for women's suffrage was temporarily halted in order to concentrate on granting African Americans and immigrants equal protection under the laws.

"I feel very little like writing, but you will be wanting to know the whys of course. One of those dreadful prairie fires, accompanied by a hurricane of wind swept through here Tuesday night, the 14th, and took everything but our house and stock. All our hay and oats that Rob and I had worked so hard to get up and stack, harness, saddle, 26 hens and chickens...all burned etc. At least a hundred dollars worth swept away in a few moments....Imagine yourselves for instance with nothing but land, house, and stock—for that's where we are. Not a tree, particle of water, grass, stable, fence or anything else....Eastern people may think us homesteaders are doing a fine thing to get 160 acres of land for nothing—all but the nothing. Oh, the suffering that the poor endure here, and privations you have not the remotest idea of, and poor means nearly all homesteaders." Letter from Mary Abell to her family, 1873 The activities described in the excerpt above most clearly show the influence of which of the following? a The substantial numbers of new immigrants that entered the country b The idea of Manifest Destiny c The passage of legislation promoting national economic development d The challenges faced by the United States as it transitioned from a rural, agricultural society to an urban, industrial one in the late 19th century

c The passage of legislation promoting national economic development

"The remedy for . . . inefficiency lies in systematic management. . . . The fundamental principles of scientific management are applicable to all kinds of human activities, from our simplest individual acts to the work of our great corporations. . . . At the works of Bethlehem Steel, for example, . . . thousands of stop-watch observations were made to study just how quickly a laborer . . . can push his shovel into the pile of materials and then draw it out properly loaded. . . .With data of this sort before him, . . . the man who is directing shovelers can first teach them the exact methods which should be employed to use their strength to the very best advantage." Frederick Winslow Taylor, The Principles of Scientific Management, 1911 Taylor's ideas expressed in the excerpt emerged most directly in response to which of the following developments in the United States? a Excessive government regulation of business b The need for rebuilding infrastructure after the Civil War c The rise of industrial capitalism d An increase in the standard of living

c The rise of industrial capitalism

*graph showing immigration rates going up each decade* The trend depicted in the graph most directly contributed to which of the following developments after 1920 ? a A decline in internal migration b Restrictions on immigration from eastern and southern Europe c Total exclusion of immigration from China d Federal efforts to return Mexican immigrants to their homeland

c Total exclusion of immigration from China

"If we do not follow the most scientific approved methods, the most modern discoveries of how to conserve and propagate and renew wherever possible those resources which Nature in her providence has given to man for his use but not abuse, the time will come when the world will not be able to support life, and then we shall have no need of conservation of health, strength, or vital force because we must have the things to support life or everything else is useless. . . . [D]o not forget that the conservation of life itself must be built on the solid foundation of conservation of natural resources, or it will be a house built upon the sands that will be washed away." Marion Crocker, General Federation of Women's Clubs, 1912 Based on the excerpt, Marion Crocker was most likely a an advocate for African American civil rights b an advocate for an expansion of the New Deal c a Progressive Era reformer d a member of the Populist Party

c a Progressive Era reformer

The poster was intended to a promote the ideals of republican motherhood b advocate for the elimination of sex discrimination in employment c convince women that they had an essential role in the war effort d persuade women to enlist in the military

c convince women that they had an essential role in the war effor

The controversy highlighted in the cartoon above most directly led to the a development of African American efforts to support vocational education. b issuance of court rulings such as Plessy v. Ferguson sanctioning racial segregation c emergence of more vigorous Southern resistance to African American rights d industrialization of some segments of the Southern economy

c emergence of more vigorous Southern resistance to African American rights

When the Sun comes back And the first quail calls Follow the Drinking Gourd, For the old man is a-waiting for to carry you to freedom If you follow the Drinking Gourd The riverbank makes a very good road. The dead trees will show you the way. Left foot, peg foot, travelling on, Follow the Drinking Gourd. The river ends between two hills Follow the Drinking Gourd. There's another river on the other side Follow the Drinking Gourd. When the great big river meets the little river Follow the Drinking Gourd. For the old man is a-waiting for to carry to freedom If you follow the Drinking Gourd. --Follow the Drinking Gourd, American folksong Which of the following most likely explains the creation and use of this song? a Abolitionists secretly printed and distributed the song lyrics to enslaved people so that they could find their way to freedom. b White abolitionists wrote and used the song to pass on secret information during church services to enslaved people c enslaved people and recently escaped slaves changed the lyrics to an existing song to secretly pass along escape routes to the North. d Enslaved people created the song as a way to mentally escape their drudgery and dream of freedom.

c enslaved people and recently escaped slaves changed the lyrics to an existing song to secretly pass along escape routes to the North.

While industries such as the one in the photograph above led to increasing conflicts between management and labor from 1890 to 1930, they also contributed to a an economy much less prone to economic distress. b a decrease in tensions between native-born and new immigrants. c improved standards of living. d better relations between the races.

c improved standards of living.

"Excepting only Yosemite, Hetch Hetchy is the most attractive and wonderful valley within the bounds of the great Yosemite National Park and the best of all the camp grounds. People are now flocking to it in ever-increasing numbers for health and recreation of body and mind. Though the walls are less sublime in height than those of Yosemite, its groves, gardens, and broad, spacious meadows are more beautiful and picturesque. . . . Last year in October I visited the valley with Mr. William Keith, the artist. He wandered about from view to view, enchanted, made thirty-eight sketches, and enthusiastically declared that in varied picturesque beauty Hetch Hetchy greatly surpassed Yosemite. It is one of God's best gifts, and ought to be faithfully guarded." - John Muir, Century Magazine, 1909 Muir's ideas are most directly a reaction to the a increase in urban populations, including immigrant workers attracted by a growing industrial economy b westward migration of groups seeking religious refuge c increasing usage and exploitation of western landscapes d opening of a new frontier in recently annexed territory

c increasing usage and exploitation of western landscapes

One Way Ticket (Langston Hughes) I am fed up With Jim Crow laws, People who are cruel And afraid, Who lynch and run, Who are scared of me And me of them. I pick up my life And take it away On a one-way ticket Gone Up North Gone Out West Gone! In the poem, the term "Jim Crow" laws refer to Southern a laws that placed poll taxes on voting for African Americans. b laws passed during Reconstruction to protect African Americans. c laws that mandated segregation of the races in public places. d laws that prevented African American males from holding public office.

c laws that mandated segregation of the races in public places.

"The Cold War is now behind us. Let us not wrangle over who won it. It is in the common interest of our two countries and nations not to fight this trend toward cooperation but rather to promote it." Soviet Premier Mikhail Gorbachev, 1990 Mikhail S. Gorbechev, "Address at Stanford University," New York Times, 1990. Early in his presidency, Ronald Reagan's relationship with the Soviets was a based on isolationism. b generally friendly. c marked by bellicose rhetoric. d fueled by a desire for territorial expansion.

c marked by bellicose rhetoric

In response to the conditions depicted in the photograph above, many American families a advocated for overseas expansion. b disrupted society with racial strife. c migrated within the United States. d resented President Roosevelt's unwillingness to use government power to provide them with relief.

c migrated within the United States.

The scene depicted in the photograph above was made possible by a greater market and credit stability. b a decline in the domination of the United States economy by large corporations. c new technologies and manufacturing techniques. d new economic opportunities for women.

c new technologies and manufacturing techniques.

"The system of quotas . . . was the first major pillar of the Immigration Act of 1924. The second provided for the exclusion of persons ineligible to citizenship. . . . Ineligibility to citizenship and exclusion applied to the peoples of all the nations of East and South Asia. Nearly all Asians had already been excluded from immigration. . . . The exclusion of persons ineligible to citizenship in 1924 . . . completed Asiatic exclusion. . . . Moreover, it codified the principle of racial exclusion into the main body of American immigration and naturalization law." - Mae M. Ngai, historian, Impossible Subjects: Illegal Aliens and the Making of Modern America, 2004 The Immigration Act of 1924 produced highly discriminatory results because it a relied on a series of literacy tests and physical examinations to manage immigration b created a guest worker program that encouraged temporary immigration but denied citizenship c placed restrictions on immigration by national origin, ethnicity, and race d encouraged immigration of people with highly sought after skills or family in the United States

c placed restrictions on immigration by national origin, ethnicity, and race

(map showing immigration rates across nation) Late 19th-century urban immigrants identified in the map above most commonly relied on support from a the federal government. b nativists. c settlement houses. d the philanthropy of business leaders.

c settlement houses.

"Now, we can see a new world coming into view. . . . In the words of Winston Churchill, a world order in which 'the principles of justice and fair play protect the weak against the strong. . . .' Aworld where the United Nations . . . is poised to fulfill the historic vision of its founders. A world in which freedom and respect for human rights find a home among all nations. The Gulf War put this new world to its first test. And my fellow Americans, we passed that test." - President George H. W. Bush, address to Congress, March 6, 1991 The principles on human rights articulated by President Bush are most similar to a the ideas expressed by President George Washington in his 1796 Farewell Address b President James K. Polk's support for Manifest Destiny in the 1840s c the ideas expressed by President Woodrow Wilson during and after the First World War d President Ronald Reagan's support for reinvigoration of anticommunism in the 1980s

c the ideas expressed by President Woodrow Wilson during and after the First World War

*Can't find the picture, it's a graph displaying the South Carolinian representation in House of Representatives. Number of republicans rose around the 1850s-70s while the number of democrats declined, but in 1876, the amount of democrats rose and republicans declined* The pattern depicted on the chart best serves as evidence of which of the following? a the end of independent political parties in American politics b the beginning of segregation in the South c the re-admittance of South Carolina to the Union d the growth of African American voting rights in the South

c the re-admittance of South Carolina to the Union

Acontinuation of the policies alluded to in the cartoon would best be exemplified by a Supreme Court decisions that applied the Sherman Anti-trust Act to unions when the efforts of organized labor resulted in "restraint of trade" b Deregulation of the telecommunications industry in the 1980s to increase competition and lower prices for all consumers. c Governmental efforts to break up monopolistic giants such as the American Telephone and Telegraph (AT&T) company or Standard Oil d Rulings of the Supreme Court that applied to protections of the 14th amendment to corporations.

c. Governmental efforts to break up monopolistic giants such as the American Telephone and Telegraph (AT&T) company or Standard Oil

The right of citizens of the United States to vote in any primary or other election for President or Vice President, for electors for President or Vice President, or for Senator or Representative in Congress, shall not be denied or abridged by the United States or any state by reason of failure to pay any poll tax or other tax. The Congress shall have power to enforce this article by appropriate legislation." Amendment XXIV, ratified 1964 By passing the 24th Amendment, Congress was fulfilling a promise that had begun with the passage of this amendment a. 19th b. 17th c. 15th d. 23rd

c. 15th

"I congratulate you on the successes of our two allies. Those of the Hollanders are new, and therefore pleasing. It proves there is a god in heaven, and the he will not slumber without end on the iniquities of tyrants, or would-be tyrants, as their Stadtholders [political leaders]. This ball of liberty, I believe most piously, is now so well in motion that it will roll round the globe, at least the enlightened part of it, for light & liberty go together. It is our glory that we first put it into motion, & our happiness that being foremost we had not bad examples to follow. What a tremendous obstacle to the future attempts at liberty will be the atrocities of Robespierre!" Letter from Thomas Jefferson to Tench Coxe, June 1, 1795 Based on the above excerpt, with which of the following statements would Thomas Jefferson most likely agree? a. Americans should actively make efforts to replace tyrannical governments around the world. b. The U.S. government should form a military alliance with the Netherlands and France. c. American democratic ideals should be spread as an inspiration for people around the world. d. U.S. soldiers should try to overthrow the new French government.

c. American democratic ideals should be spread as an inspiration for people around the world.

"Our people are losing faith, not only in government itself but in their ability as citizens to serve as the ultimate rulers and shapers of our democracy. "We were sure that ours was a nation of the ballot, not the bullet, until the murders of John Kennedy and Robert Kennedy and Martin Luther King Jr. We were taught that our armies were always invincible and our causes were always just, only to suffer the agony of Vietnam. We respected the Presidency as a place of honor until the shock of Watergate. "We remember when the phrase 'sound as a dollar' was an expression of absolute dependability, until ten years of inflation began to shrink our dollar and our savings. We believed that our Nation's resources were limitless until 1973, when we had to face a growing dependence on foreign oil. —Jimmy Carter, Public Papers of the Presidents of the United States, 1979 Which of the following best identifies the effect of the speech from which the above excerpt is taken? a. The Federal Reserve raised interest rates to support the President b. Consumers increased their efforts to conserve energy c. Americans blamed Carter for weak and ineffective leadership d. Carter increased his approval ratings with his honesty

c. Americans blamed Carter for weak and ineffective leadership

*Map displaying amount of slaves sent to each part of Americas* The major pattern on the map best supports which of the following statements? a. Fewer people lived in the British colonies than in the Spanish colonies. b. Enslaved Africans were more likely to live on large plantations in North America than in the Caribbean. c. British colonies in North America typically had a lower demand for slave labor than did the colonies of other European countries. d. Africans in North America were more likely to resist slavery than were Africans in South America.

c. British colonies in North America typically had a lower demand for slave labor than did the colonies of other European countries.

"Wherefore, security being the trued design and end of government, it unanswerably follows that wheatever form thereof appears most likely to ensure it to us, with the least expense, and greatest benefit, is preferable to all others. ... Here too is the design and end of governmnet, Freedom and Security." Thomas Paine, Common Sense, 1776 "[G]overnments are instituted among Men, deriving their just powers from the consent of the governed. That whenever any Form of Government becomes destructive of these ends, it is the Right of the People to alter or to abolish it, and institute new Government, laying its foundations on such principles and organizing its powers in such form, as to them shall seem mostlikely to effect their Safety and Happiness." Thomas Jefferson, Declaration of Independence, 1776 The excerpts were written in response to the a. British government's failure to protect colonists from attacks by American Indians b. colonial governments' attempts to extend political rights to new groups c. British government's attempt to assert greater control over the North American colonies d. colonial governments' failure to implement mercantilist policies

c. British government's attempt to assert greater control over the North American colonies

"The first we heard [while Smith was exploring the James River in May] was that 400 Indians the day before had assaulted the fort and surprised it. . . .With all speed we palisadoed [built barricades around] our fort; . . .The day before the ship 's departure the king of [the] Pamunkey sent [an] Indian . . . to assure us peace, our fort being then palisadoed round, and all our men in good health and comfort, albeit . . . it did not so long continue. "[By September] most of our chiefest men [were] either sick or discontented, the rest being in such despair as they would rather starve and rot with idleness than be persuaded to do anything for their own relief without constraint. Our victuals being now within eighteen days spent, and the Indian trade decreasing, I was sent to the mouth of the river to Kegquouhtan, an Indian town, to trade for corn, and try the river for fish, but our fishing we could not effect by reason of the stormy weather. The Indians, thinking us near famished, with careless kindness offered us little pieces of bread and small handfuls of beans or wheat for a hatchet or a piece of copper. In like manner I entertained their kindness and in like . . . offered them like commodities, but the children, or any that showed extraordinary kindness, I liberally contented with free gift of such trifles as well contented them." John Smith, English explorer relating events in the Virginia colony, 1608 Smith's description of the Pamunkey people's interactions with theVirginia colonists best serves as evidence of which of the following characteristics of American Indians along the Eastern Seaboard in the 1600s? a. Decreased populations due to epidemic disease b. A dependence on English trade goods c. Complex societies with permanentsettlements d. Elaborate caste systems

c. Complex societies with permanentsettlements

The building of Levittown and similar communities reflected all of the following post-war developments EXCEPT a. Release of enormous pent-up demand due to 15 years of hardship and sacrifice b. A baby boom spurred on by the return of young war veterans c. Consumers' ability to buy their homes outright without loans from the government d. Young families' desire to escape crowded cities

c. Consumers' ability to buy their homes outright without loans from the government

"We conclude that in the field of public education the doctrine of 'separate but equal' has no place. Separate educational facilities are inherently unequal." - Chief Justice Earl Warren, writing the unanimous opinion of the United States Supreme Court in Brown v. Board of Education of Topeka, 1954 Which of the following sets of Supreme Court decisions demonstrated the strongest continuity with the idea expressed in the excerpt? a. Decisions endorsing the constitutionality of the death penalty, such as Gregg v. Georgia b. Decisions defining election laws, such as Citizens United v. Federal Election Commission c. Decisions defining individual rights and protections, such as Miranda v. Arizona d. Decisions limiting affirmative action programs, such as Regents of the University of California v. Bakke

c. Decisions defining individual rights and protections, such as Miranda v. Arizona

"Slavery, though imposed and maintained by violence, was a negotiated relationship.... First, even as they confronted one another, master and slave had to concede, however grudgingly, a degree of legitimacy to the other.... [T]he web of interconnections between master and slave necessitated a coexistence that fostered cooperation as well as contestation. Second, because the circumstances of such contestation and cooperation continually changed, slavery itself continually changed....Slavery was never made, but instead was continually remade, for power—no matter how great—was never absolute, but always contingent." Ira Berlin, historian, Many Thousands Gone: The First Two Centuries of Slavery in North America, 1998 Which of the following contributed most to the increasing use of African slave labor in North America during the 1600s and 1700s? a. A decline in regional distinctiveness among the colonies b. The spread of European Enlightenment ideas in the colonies c. European demand for agricultural products grown in the colonies d. Successful colonial attempts to convert enslaved Africans to Christianity

c. European demand for agricultural products grown in the colonies

"In 1739 arrived among us from Ireland the Reverend Mr. [George] Whitefield, who had made himself remarkable there as an itinerant preacher. He was at first permitted to preach in some of our churches; but the clergy, taking a dislike to him, soon refused him their pulpits, and he was obliged to preach in the fields. The multitudes of all sects and denominations that attended his sermons were enormous. . . . It was wonderful to see the change soon made in the manners of our inhabitants. From being thoughtless or indifferent about religion, it seemed as if all the world were growing religious, so that one could not walk thro' the town in an evening without hearing psalms sung in different families of every street." Benjamin Franklin, The Autobiography of Benjamin Franklin Whitefield's open-air preaching contributed most directly to which of the following trends? a. The pursuit of social reform b. Movement of settlers to the backcountry c. Greater independence and diversity of thought d. The growth of the ideology of republican motherhood

c. Greater independence and diversity of thought

*Map displaying Columbian Exchange & products traded between Europe, Africa, and the Americas* As a result of the process illustrated above, what major shift occurred in the Spanish colonies' encomienda system? a. There were many new converts to Christianity. b. Social, religious, political, and economic competition with other European nations developed. c. Indian labor was gradually replaced with African slavery. d. The Spanish began debating the proper treatment of American Indians.

c. Indian labor was gradually replaced with African slavery.

*picture at little rock rally, protesting integration of Little Rock Central High School* Which federal measure most directly led to the protest depicted in the above photo? a. Executive branch desegregation of the armed services b. Judicial decision upholding the idea of "separate, but equal" c. Judicial decision overturning the precedent set by Plessy v. Ferguson d. Congressional passage of the Civil Rights Act

c. Judicial decision overturning the precedent set by Plessy v. Ferguson

"The Declaration of the People. 1. For having upon specious pretences of publiqe works raised greate unjust taxes upon the Comonality for the advancement of private favorites and other sinister ends, but noe visible effects in any measure adequate, For not haveing dureing this long time of his Gouvernement in any measure advanced this hopefull Colony either by fortificacons Townes or Trade...4. For having protected, favoured, and Imboldned the Indians against his Majesties loyall subjects, never contriveing, requireing, or appointing any due or proper meanes of satisfaction for theire many Invasions, robberies, and murthers comitted upon us..." Nathaniel Bacon's Challenge to William Berkeley, 1676. Which of the following groups would be most likely to support the perspective of Nathaniel Bacon in the excerpt? a. Coastal Virginia tobacco merchants b. Susquehanaug Indians c. Landless freeman d. British royal officials

c. Landless freeman

Where the money is to come from which will defray this enormous annual expense of three million sterling [for the American Revolution], and all those other debts, I know not...Certain I am that our commerce and agriculture, the two principal sources of our wealth, will not support such an expense. The whole of our exports from the Thirteen United Colonies, in the year 1769, amounted only to £2,887,898 sterling; which is not so much, by near half a million, as our annual expense would be were we independent of Great Britain. Those exports, with no inconsiderable part of the profits arising from them, it is well known, centered finally in Britain to pay the merchants and manufacturers there for goods we had imported thence—and yet left us still in debt! What then must our situation be, or what the state of our trade, when oppressed with such a burden of annual expense! When every article of commerce, every necessary of life, together with our lands, much be heavily taxed to defray the expense! - Charles Inglis, Anglican church minister in New York City, The Costs of Revolution, 1776. The point of view of the author is that of a a. Patriot b. Son of Liberty c. Loyalist d. Whig

c. Loyalist

"The national dignity and justice require that the arms of the Union should be called forth in order to chastise the Creek nation of Indians, for refusing to treat with the United States on reasonable terms, and for their hostile invasion of the State of Georgia....The most effectual mode of reducing the Creeks to submit to the will of the United States...would be by an adequate army, to be raised and continued until the objects of the war should be accomplished....But, in future, the obligations of policy, humanity, and justice, together with that respect which every nation sacredly owes to its own reputation, unite in requiring a noble, liberal, and disinterested administration of Indian affairs....In the administration of the Indians, every proper expedient that can be devised to gain their affections, and attach them to the interest of the Union, should be adopted....Missionaries, of excellent moral character, should be appointed to reside in their nation....They should be their friends and fathers." - From Henry Knox to George Washington, July 7, 1789, in American State Papers, Class II: Indian Affairs The proposals outlined in the excerpt above are most similar to the prior approach of which European country toward American Indians in the 1600s and 1700s? a. France b. Portugal c. Spain d. England

c. Spain

"We regard the decision of the Supreme Court in the school cases as clear abuse of judicial power....This unwarranted exercise of power by the court, contrary to the Constitution is creating chaos and confusion in the states principally affected. It is destroying the amicable relations between the white and Negro races that have been created through ninety years of patient effort by the good people of both races. It has planted hatred and suspicion where there has been heretofore friendship and understanding. Without regard to the consent of the governed, outside agitators are threatening immediate and revolutionary changes in our public school systems. If done, this is certain to destroy the system of public education in some of the states." The Southern Declaration on Integration, March 11, 1956 Which landmark development largely ended the possibility of support for the author's goals outlined in the quote above? a. The New Deal b. The 15th Amendment c. The Civil Rights Act of 1964 d. Post-September 11, 2001, civil rights debates

c. The Civil Rights Act of 1964

*map of Western hemisphere, showing how US intervened in many Latin American nations* American foreign policy as represented in the above map echoes which of the following stated U.S. policies from the early 1800s? a. America would remain neutral in Latin America's struggle against Spanish imperialism. b. The U.S. would not interfere in the internal affairs of Europe. c. The U.S. would consider any imperialistic movements by the European powers in the American continents as a hostile act. d. The U.S. would diplomatically recognize all new countries in Latin America arising from revolts against the Spanish empire.

c. The U.S. would consider any imperialistic movements by the European powers in the American continents as a hostile act.

"For the increase of shipping . . . from thenceforward, no goods or commodities whatsoever shall be imported into or exported out of any lands, islands, plantations, or territories to his Majesty belonging . . . but in ships or vessels as do . . . belong only to the people of England . . . and whereof the master and three-fourths of the mariners at least are English. . . . "And it is further enacted . . . that . . . no sugars, tobacco, cottonwool, indigos, ginger, fustic, or other dyeing wood, of the growth, production, or manufacture of any English plantations in America, Asia, or Africa, shall be . . . transported from any of the said English plantations [colonies] to any land . . . other than to such other English plantations as do belong to his Majesty." English Parliament, Navigation Act of 1660 Which of the following most directly led to the passage of the Navigation Act of 1660 ? a. The dominance of market capitalism b. The spread of Enlightenment ideas c. The emergence of an Atlantic economy d. The development of the English system of slavery

c. The emergence of an Atlantic economy

*Picture displaying the expansion of British colonies* a. The large size of Britich colonial populations relative to American Indian populations b. British recognition of Native American sovereignty c. The orientation of the British colonies toward producing commodities for export to Europe d. British government attempts to impose greater control over the colonies in the late 1600s

c. The orientation of the British colonies toward producing commodities for export to Europe

"English expectations of the New World and its inhabitants died hard. America was supposed to be a land of abundance, peopled by natives who would not only share that abundance with the English but increase it under English direction. Englishmen simply did not envisage a need to work for the mere purpose of staying alive. The problem of survival as they saw it was at best political and at worst military. "Although Englishmen long remained under the illusion that the Indians would eventually become useful English subjects, it became apparent fairly early that Indian labor was not going to sustain the founders of Jamestown [Virginia]." — Edmund S. Morgan, historian, "The Labor Problem at Jamestown, 1607-18," published in 1971 Which of the following was a long-term result of the situation in Jamestown described in the excerpt? a. The removal of Spanish and French military threats to regional control b. The prioritizing of trade and shipbuilding over agricultural production c. The rise of the plantation system and the use of African slaves d. The rapid growth of family-centered towns and villages

c. The rise of the plantation system and the use of African slaves

*Picture of Hamilton's Financial Plan* Enactment of the financial construct represented in the above graphic reflects which viewpoint in the debates within the newly formed United States over interpretation of the Constitution? a. Those who argued that the Constitution was vague in outlining the powers of Congress. b. Those who argued that Congress could not exercise powers which the Constitution did not explicitly enumerate. c. Those who argued that the Constitution allowed for broader powers of Congress by implication. d. Those who argued for strict interpretation of the Constitution.

c. Those who argued that the Constitution allowed for broader powers of Congress by implication.

"The first we heard [while Smith was exploring the James River in May] was that 400 Indians the day before had assaulted the fort and surprised it. . . .With all speed we palisadoed [built barricades around] our fort; . . .The day before the ship 's departure the king of [the] Pamunkey sent [an] Indian . . . to assure us peace, our fort being then palisadoed round, and all our men in good health and comfort, albeit . . . it did not so long continue. "[By September] most of our chiefest men [were] either sick or discontented, the rest being in such despair as they would rather starve and rot with idleness than be persuaded to do anything for their own relief without constraint. Our victuals being now within eighteen days spent, and the Indian trade decreasing, I was sent to the mouth of the river to Kegquouhtan, an Indian town, to trade for corn, and try the river for fish, but our fishing we could not effect by reason of the stormy weather. The Indians, thinking us near famished, with careless kindness offered us little pieces of bread and small handfuls of beans or wheat for a hatchet or a piece of copper. In like manner I entertained their kindness and in like . . . offered them like commodities, but the children, or any that showed extraordinary kindness, I liberally contented with free gift of such trifles as well contented them." John Smith, English explorer relating events in the Virginia colony, 1608 Smith most likely wrote his account for which of the following reasons? a. To encourage the Virginia colonists to abandon the colony b. To promote an alliance between colonists in Virginia and colonists elsewhere in the Americas c. To increase support for the colony from the monarchy and investors d. To recruit missionaries to come to the Virginia colony

c. To increase support for the colony from the monarchy and investors

"By the end of 1949, only one out of three heroines in the women's magazines was a career woman—and she was shown in the act of renouncing her career and discovering that what she really wanted to be was a housewife. In 1958, and again in 1959, I went through issue after issue of the three major women's magazines (the fourth, Woman's Home Companion, had died) without finding a single heroine who had a career, a commitment to any work, art, profession, or mission in the world, other than "Occupation: housewife." Only one in a hundred heroines had a job; even the young unmarried heroines no longer worked except at snaring a husband." --Betty Friedan, journalist, The Feminine Mystique, published in 1963 For which labor issue did the women's movement make some legal progress by the mid-20th century? a. Society's view of women's employment outside of the home as immoral b. Unsafe physical working conditions c. Unequal pay d. Long working hours

c. Unequal pay

SEC.5(b) Within sixty calendar days after a report is submitted or required to be submitted persuant to section 4(a)(1), whichever is earlier, the President shall terminate any use of United States Armed Forces with respect to which such report was submitted (or required to be submitted), unless the Congress (1) has declared war or has enacted a specific authorization for such use of United States Armed Forces, (2) has extended by law such sixty-day period, or (3) is physically unable to meet as a result of an armed attack upon the United States. -War Powers Act, 1973 The Congress passed the War Powers Act of 1973 in order to a. affirm United States support for the United Nations b. increase United States participation in international peacekeeping operations c. assert the role of Congress in the commitment of troops overseas d. strengthen the policy of détente

c. assert the role of Congress in the commitment of troops overseas

*Map displaying fights of Pontiac's War* Pontiac's War, as illustrated by the map above, was provoked primarily by the a. increasing number of clashes between French and British soldiers in Indian lands b. British initiatives to capture Indian lands west of the Appalachians. c. continue expansion of British colonists into Indian lands. d. broken French promise to allow Indian control of all lands west of the Appalachians.

c. continue expansion of British colonists into Indian lands.

*Map displaying fights of Pontiac's War* Pontiac's main purpose in waging the war was to a. help the French to win in their war against the British. b. win back lands lost tot he British east of the Appalachians c. drive the British out of the area of land around the Ohio River. d. drive the British out of Canada

c. drive the British out of the area of land around the Ohio River.

"That your petitioners are greatly aggrieved by the present operation of an Excise Law...by which we are made subject to a duty of four pence per gallon on all spirituous liquors distilled and consumed amongst us from the productions of our farms, even for private and domestic uses...excise laws...have...created...tumults amongst the people...In this new country, labourers are exceedingly scarce, and their hire excessively high, and we find that liquor proves a necessary means of engaging their service and securing their continuance through the several important seasons of the year, when the pressing calls of labour must be attended to, let the conditions be what they may. Our remote situation from the channels of commerce, has long ago prohibited the use of all imported liquors amongst us, and as we are aiming at independence in our manner of living, we have neither the abilities or inclination to aspire to their use. We freely resign them to our eastern neighbours,... we are perfectly content with the humble produce of our own farms, and it is our only wish to be permitted to enjoy them in freedom." Farmers Protest the New Whiskey Tax, 1790 One direct effect of President Washington's use of force to end the Whiskey Rebellion was that it a. weakened the authority of the federal government. b. led western farmers to vote for John Adams in the next election. c. ensured that the authority of the federal government was supreme. d. prompted many western farmers to immigrate to Canada.

c. ensured that the authority of the federal government was supreme.

"Wee must be knitt together in this worke as one man, wee must entertaine each other in brotherly Affeccion... wee must delight in eache other, make others Condicions our owne rejoyce together, mourne together, labour, and suffer together... soe that wee shall see much more of his wisdome power goodnes and truthe then formerly wee have beene acquainted with, wee shall finde that the God of Israell is among us, when tenn of us shall be able to resist a thousand of our enemies, when hee shall make us a prayse and glory, that men shall say of succeeding plantacions: the lord make it like that of New England: for wee must Consider that wee shall be as a Citty upon a Hill, the eies of all people are uppon us; soe that if wee shall deale falsely with our god in this worke wee have undertaken and soe cause him to withdrawe his present help from us, wee shall be made a story and a byword through the world, wee shall open the mouthes of enemies to speake evill of the wayes of god and all professours for Gods sake." John Winthrop, A Model of Christian Charity, 1630 The passage reinforces the idea that the New England colonies were largely a. based on a single, labor-intensive crop. b. aided by favorable environmental conditions. c. homogenous, tightly-knit colonies. d. heterogeneous, loosely connected colonies

c. homogenous, tightly-knit colonies.

"The Secretary of the Treasury having perused with attention the papers containing the opinions of the Secretary of State and Attorney General concerning the constitutionality of the bill for establishing a National Bank proceeds according to the order of the President [George Washington] to submit the reasons which have induced him to entertain a different opinion...the objections of the Secretary of State and Attorney General are founded on a general denial of the authority of the United States to erect corporations...It is not denied, that there are implied, as well as express powers...They are "to make all laws, necessary & proper for carrying into execution the foregoing powers & all other powers vested by the constitution in the government of the United States, or in any department or officer thereof." ...A bank has a natural relation to the power of collecting taxes; to that of borrowing money; to that of regulating trade; to that of providing for the common defence...it brings the case within the provision of the clause of the constitution which immediately respects the property of the United States." Alexander Hamilton, "An Opinion on the Constitutionality of an Act to Establish a Bank," 1791 In the decade following the ratification of the Constitution, divisions emerged between Hamilton's political party, the Federalists, and their rivals, the Democratic-Republicans, over all of the following issues EXCEPT the a. federal government's degree of support for manufacturing and finance b. United States policy toward the French Revolution c. relationship between religion and the federal government d. relationship between national and state governments

c. relationship between religion and the federal government

"Friday June 15th, 1787 Mr. Patterson laid before the Convention the plan which he said several of the duputations withsed to be submitted in place of that proposed by Mr. Randolph. 1. Resolved that the articles of Conederation ought to be so revised, corrected and enlarged, as to render the federal Constitution adequate to the exigences of Government and the preservation of the Union 2. Resolved that ... Congress...be authorized to pass acts for raising a revenue ... [and] to pass Acts for the regulation of trade and commerce as well with foreign nationa as with each other. 4. Resolved that the United States in Congress be authorized to elect a federal Executive 5. Resolved that a federal Judiciary be established to consist of a supeme [court] of Judges of which to be appointed by the Executive, and to hold their offices during good behavior." - Notes from the Constitutional Convention as recored by James Madison, June 15th, 1787. The debates at the Constitutional Convention deadlocked over the issue of a. issue of slavery b. separation of powers c. representation in the National Legislature d. powers of the Executive branch

c. representation in the National Legislature

The demographic changes that resulted in the development of housing projects like the one in the image could be described as: a. the Great Migration. b. westward migration. c. suburbanization. d. immigration.

c. suburbanization.

"Friday June 15th, 1787 Mr. Patterson laid before the Convention the plan which he said several of the duputations withsed to be submitted in place of that proposed by Mr. Randolph. 1. Resolved that the articles of Conederation ought to be so revised, corrected and enlarged, as to render the federal Constitution adequate to the exigences of Government and the preservation of the Union 2. Resolved that ... Congress...be authorized to pass acts for raising a revenue ... [and] to pass Acts for the regulation of trade and commerce as well with foreign nationa as with each other. 4. Resolved that the United States in Congress be authorized to elect a federal Executive 5. Resolved that a federal Judiciary be established to consist of a supeme [court] of Judges of which to be appointed by the Executive, and to hold their offices during good behavior." - Notes from the Constitutional Convention as recored by James Madison, June 15th, 1787. Which of the following was NOT a compromise or negotation at the Constitution Convention excerpted above? a. the 3/5th Compromise b. the Great Compromise c. the Bill of Rights Compromise d. the Connecticut Compromise

c. the Bill of Rights Compromise

"We are all in it together. This is a war. We take a few shots and it will be over. We will give them a few shots and it will be over. Don't worry. I wouldn't want to be on the other side right now....I want the most comprehensive notes on all those who tried to do us in. They didn't have to do it. If we had had a very close election and they were playing the other side I would understand this. No—they were doing this quite deliberately and they are asking for it and they are going to get it....We have not used the Bureau, and we have not used the Justice Department, but things are going to change now....And who the hell are they after? They are after us. It is absolutely ridiculous. It is not going to be that way anymore." Transcript of President Nixon speaking to John Dean in the Oval Office, September 5,1972 U.S. Congress, House. National Archives. The excerpt above most directly contributed to renewed debates about a. the proper balance between liberty and order. b. the proper degree of government activism. c. the power of the presidency and the federal government. d. official restrictions on freedom of speech.

c. the power of the presidency and the federal government.

"...Whereas the enforcing of the conscience in matters of religion has frequently fallen out to be of dangerous consequence in those commonwealths where it has been practiced, and for the more quiet and peaceable government of this Province, and the better to preserve mutual love and amity among the inhabitants thereof. Be it therefore enacted that no person or persons whatever in the Province...professing to believe in Jesus Christ, shall from henceforth be any ways troubled, molested, or discountenanced for or in respect of his or her religion nor in the free exercise thereof within the Province...nor in any way compelled to the belief or exercise of any other religion against his or her consent, so [long] as they not be unfaithful to the Lord Proprietary, or molest or conspire against the civil government established in this Province under him..." Maryland Act Concerning Religion, 1644 The ideas expressed in the passage above most directly led to political controversies over a. the presence of slavery in the colonies. b. Britain's efforts to pursue mercantilist economic aims. c. the spread of European Enlightenment ideas. d. Britain's indifference to colonial governance.

c. the spread of European Enlightenment ideas.

" Let us, then, fellow-citizens, unite with one heart and one mind. Let us restore to social intercourse that harmony and affection without which liberty and even life itself are but dreary things. And let us reflect that, having banished from our land that religious intolerance under which mankind so long bled and suffered, we have yet gained little if we countenance a political intolerance as despotic, as wicked, and capable of as bitter and bloody persecutions...We have called by different names brethren of the same principle. We are all Republicans, we are all Federalists. If there be any among us who would wish to dissolve this Union or to change its republican form, let them stand undisturbed as monuments of the safety with which error of opinion may be tolerated where reason is left free to combat it." Thomas Jefferson, first inaugural address, 1801 Which of the following issues of the period was Jefferson most likely concerned with in the excerpt? a. The growth of slave labor b. The expansion of the right to vote c. The growth of various Protestant religious groups d. The creation of political parties

d

"As [political leader Henry] Clay envisioned it [in the 1820s], the American System constituted the... basis for social improvement. ... Through sale of its enormous land holdings, the federal government could well afford to subsidize internal improvements. By levying protective tariffs, the government should foster the development of American manufacturing and agricultural enterprises that, in their infancy, might not be able to withstand foreign competition. The promotion of industry would create a home market for agricultural commodities, just as farms provided a market for manufactured products." Daniel Walker Howe, historian, What Hath God Wrought: The Transformation of America, 1815-1848, published in 2007 One major change in United States politics from the 1820s to the mid-1850s was the a. collapse of the Democratic Party in the South b. adoption of an antislavery platform by the Whig Party c. decline of competition between parties d. rise of political parties defined largely by regional interests

d

"British cruisers have been in the continued practice of violating the American flag on the great highway of nations, and of seizing and carrying off persons sailing under it, not in the exercise of a belligerent right founded on the law of nations against an enemy, but of a municipal [internal] prerogative over British subjects. British jurisdiction is thus extended to neutral vessels... " James Madison, 1812 The War of 1812 ended a. with the Great Lakes under the control of Canada. b. with the United States losing Canada. c. with the boundary of the United States extended to the Pacific. d. as antebellum status quo.

d

"British cruisers have been in the continued practice of violating the American flag on the great highway of nations, and of seizing and carrying off persons sailing under it, not in the exercise of a belligerent right founded on the law of nations against an enemy, but of a municipal [internal] prerogative over British subjects. British jurisdiction is thus extended to neutral vessels... " James Madison, 1812As a result of the War of 1812, a. the United States lost a devastating war, which took years to recover from. b. British sympathizers were forced to leave the United States. c. British soldiers left the forts in the Northwest Territory that they had occupied since colonial times. d. there was a great surge of nationalism in the United States.

d

"Few historians would dispute that the market revolution brought substantial material benefits to most northeasterners, urban and rural. . . . Those who benefited most from the market revolution—merchants and manufacturers, lawyers and other professionals, and successful commercial farmers, along with their families—faced life situations very different from those known to earlier generations. The decline of the household as the locus of production led directly to a growing impersonality in the economic realm;household heads, instead of directing family enterprises or small shops, often had to find ways to recruit and discipline a wage-labor force; in all cases, they had to stay abreast of or even surpass their competitors." Sean Wilentz, historian, "Society, Politics, and the Market Revolution,1815-1848," published in 1997 Which of the following historical developments contributed most directly to the market revolution? a. The emergence of southern opposition to tariffs b. The decline of slavery in the Northeast c. The increased number of women in the paid workforce d. The emergence of new forms of transportation

d

"Few historians would dispute that the market revolution brought substantial material benefits to most northeasterners, urban and rural. . . . Those who benefited most from the market revolution—merchants and manufacturers, lawyers and other professionals, and successful commercial farmers, along with their families—faced life situations very different from those known to earlier generations. The decline of the household as the locus of production led directly to a growing impersonality in the economic realm;household heads, instead of directing family enterprises or small shops, often had to find ways to recruit and discipline a wage-labor force; in all cases, they had to stay abreast of or even surpass their competitors." Sean Wilentz, historian, "Society, Politics, and the Market Revolution,1815-1848," published in 1997 Which of the following pieces of historical evidence from the United States census could best be used to support the argument in the excerpt? a. Data showing changes in cotton production and price b. Data showing the growth of the slave population c. Data showing population growth in the West d. Data showing changes in the number of textile mills

d

"I come to present the strong claims of suffering humanity. I come as the advocate of helpless, forgotten, insane and idiotic men and women; of beings sunk to a condition from which the most unconcerned would start with real horror; of beings wretched in our Prisons, and more wretched in our Alms-Houses....I proceed, Gentlemen, briefly to call your attention to the present state of Insane Persons confined within this Commonwealth, in cages, closets, cellars, stalls, pens! Chained, naked, beaten with rods, and lashed into obedience!...The crying evil and abuse of institutions, is not confined to our almshouses. The warden of a populous prison near this metropolis, populous, not with criminals only, but with the insane in almost every stage of insanity...has declared that: "the prison has often more resembled the infernal regions than any place on earth!"...Gentlemen, I commit to you this sacred cause. Your action upon this subject will affect the present and future condition of hundreds and of thousands. In this legislation, as in all things, may you exercise that "wisdom which is the breath of the power of God." Dorothea Dix, Memorial to the Legislature of Massachusetts, 1843Dorothea L. Dix, Memorial to the Legislature of Massachusetts (Boston: Munroe & Francis, 1843). The concerns articulated by Dorothea Dix in the excerpt above were most similar to those of a. Social Darwinists in the late 1800s. b. nativists in the 1840s and 1850s. c. conservatives in the 1970s and 1980s. d. Progressives in the early 1900s.

d

"There are a few focal points upon which Jackson's modern reputation has turned for better or for worse. One is his attack on corporate privilege and on the concentrated political influence of wealth. In his famous Bank Veto of 1832, Jackson juxtaposed "the rich and powerful" against "the humble members of society—the farmers, mechanics, and laborers," and lamented that the former "too often bend the acts of government to their selfish purposes." No president before and few since have spoken so bluntly of economic antagonisms between Americans. Jackson went on, in his Farewell Address in 1837, to warn of an insidious "money power," made up of banks and corporations, that would steal ordinary citizens' liberties away from them. (It said something of Jackson's sense of his own importance that he presumed to deliver a Farewell Address, an example set by Washington that no previous successor had dared to follow.)"Daniel Feller, Professor of History at the University of Tennessee, Knoxville, 2008"Jackson's ideological conviction about the flexible nature of the law and Constitution in the face of dangers confronting the still-fledgling nation can be seen in many subsequent Jacksonian battles. When President Jackson confronted the Bank of the United States in 1832, he did so with the belief that it was a corrupt fiscal monster threatening the nation's economic security. He not only vetoed the Bank's re charter, which was within his right as chief executive, but went a step further by removing federal deposits even after Congress had deemed them safe. Jackson transferred one secretary of the treasury and fired another in order to secure the deposit removals. His actions were questionable, if not completely illegal, and the Senate censured him by making a notation in their journal. They didn't attempt impeachment for lack of support." Matthew Warshauer, Professor of History at Central Connecticut State University, 2008 According to Daniel Feller, Andrew Jackson's main criticism of the Bank of the United States was that a. it did not have enough money to pay off the nation's debts. b. the Bank was making the United States vulnerable to an economic take over by foreign powers. c. he wanted to prove that he was a stronger president than Washington had been, so he used the veto power more. d. the Bank did not operate in the interest of the "common man".

d

"We, therefore, the people of the State of South Carolina in Convention assembled, do declare and ordain...That the several acts and parts of acts of the Congress of the United States, purporting to be laws for the imposing of duties and imposts on the importation of foreign commodities...and, more especially...[the tariff acts of 1828 and 1832]...are unauthorized by the Constitution of the United States, and violate the true meaning and intent thereof, and are null, void, and no law, nor binding upon this State, its officers or citizens....And we, the People of South Carolina...Do further Declare that we will not submit to the application of force, on the part of the Federal Government, to reduce this State to obedience; but that we will consider the passage, by Congress, of any act...to coerce the State...to be null and void, inconsistent with the longer continuance of South Carolina in the Union..." South Carolina Ordinance of Nullification, 1832Paul Leicester Ford, The Federalist: A Commentary on the Constitution of the United States (New York: Henry Holt, 1898). In which of the following areas did regional interests and perspectives have the LEAST impact on national policy? a. Internal improvements b. National bank c. Tariffs d. American Indian policy

d

"We, therefore, the people of the State of South Carolina in Convention assembled, do declare and ordain...That the several acts and parts of acts of the Congress of the United States, purporting to be laws for the imposing of duties and imposts on the importation of foreign commodities...and, more especially...[the tariff acts of 1828 and 1832]...are unauthorized by the Constitution of the United States, and violate the true meaning and intent thereof, and are null, void, and no law, nor binding upon this State, its officers or citizens....And we, the People of South Carolina...Do further Declare that we will not submit to the application of force, on the part of the Federal Government, to reduce this State to obedience; but that we will consider the passage, by Congress, of any act...to coerce the State...to be null and void, inconsistent with the longer continuance of South Carolina in the Union..." South Carolina Ordinance of Nullification, 1832Paul Leicester Ford, The Federalist: A Commentary on the Constitution of the United States (New York: Henry Holt, 1898). The sentiments expressed in the excerpt above most closely parallel those expressed in the political debates a. between imperialists and anti-imperialists in the early 1900s. b. surrounding World War I and World War II. c. over the cultural conflicts of the 1920s. d. during the first national administrations in the 1790s.

d

"[I promise]...to demonstrate in the course of...my Appeal...that we Coloured People of these United States, are, the most wretched, degraded and abject set of beings that ever lived since the world began, down to the present day, and that the white Christians of America, who hold us in slavery, (or, more properly speaking, pretenders to Christianity,) treat us more cruel and barbarous than any Heathen nation did any people whom it had subjected, or reduced to the same condition....I advance it therefore to you...as an unshaken and forever immoveable fact, that your full glory and happiness, as well as all other coloured people under Heaven, shall never be fully consummated, but with the entire emancipation of your enslaved brethren all over the world." David Walker, Appeal to the Coloured Citizens of the World, 1829David Walker, Walker's Appeal in Four Articles; Together with a Preamble, To the Coloured Citizens of the World, but in Particular, and Very Expressly, to Those of the United States of America Written in Boston, State of Massachusetts, September 28, 1829 (Boston: Revised and Published by David Walker, 1830). Which of the following resulted from the sentiments expressed in the excerpt above? a. Growing national support among Whigs and Democrats for abolition b. Increasing numbers of Northerners insisting that the federal government should defend slavery c. Decreased demand from Northern factories for Southern cotton cultivated by slaves d. Increasingly bitter national debates over the institution of slavery

d

The above painting illustrates a. dangers of living in the American West. b. land won by the United States in the War of 1812. c. the need for a canal system to encourage settlement. d. a sense of unity between the American man and nature.

d

The image of President Thomas Jefferson addressing a group of disgruntled men most directly reflects which of the following? a. Jefferson defending his policy of the Louisiana Purchase of 1803 b. Jefferson defending his policy of republicanism c. Jefferson defending his policy of the separation of church and state d. Jefferson defending his policy of the Embargo Act of 1807

d

The painting represents an idea similar to that of a. opponents of the temperance movement. b. American realist authors. c. critics of growing democracy. d. American romantic authors.

d

Which of the following groups would have been least likely to support the authors views expressed in the cartoon? a. Democratic-Republicans b. Federalists c. Merchant Marines d. New England ship builders

d

During Reconstruction, which of following was a change that took place in the South? a Many African Americans found manufacturing employment. b African Americans favored the Democratic Party. c Many White Southerners supported African Americans? ' rights. d African Americans were able to exercise political rights

d African Americans were able to exercise political rights

The cartoon suggests that the disparate groups that favored the People's (Populist) Party typically shared which of the following a The idea that wealthy people had some obligation to help people living in poverty b Advocacy of individual rights c Support for United States expansionism d Belief in a stronger federal government role in the United States economic system

d Belief in a stronger federal government role in the United States economic system

SEC. 2. And be it further enacted, That the right of way through the public lands be, and the same is hereby, granted to said company for the construction of said railroad and telegraph line; and the right, power, and authority is hereby given to said company to take from the public lands adjacent to the line of said road earth, stone, timber, and other materials for the construction thereof; said right of way is granted to said railroad to the extent of two hundred feet in width on each side of said railroad where it may pass over the public lands, including all necessary grounds for stations, buildings, workshops, and depots, machine shops, switches, side tracks, turntables, and water stations. The United States shall extinguish as rapidly as may be the Indian titles to all lands falling under the operation of this act and required for the said right of way and grants hereafter made. Pacific Railway Act Besides the building of this railroad, the federal government took what other steps to attract settlers to the west? a Cleared out Native Americans from the prime territory in the west. b Initiated an assimilation policy to control the Native Americans. c Created a policy so that citizens purchased federal land only with gold or silver. d Created programs to give away land to settlers, if they agreed

d Created programs to give away land to settlers, if they agreed

"DEPARTMENT OF THE INTERIOR, Washington, December 2, 1882 SIR: I desire to call your attention to what I regard as a great hindrance to the civilization of the Indians, viz, the continuance of the old heathenish dances, such as the sun-dance, scalp-dance, & c. These dances, or feasts, as they are sometimes called, ought, in my judgment, to be discontinued, and if the Indians now supported by the Government are not willing to discontinue them, the agents should be instructed to compel such discontinuance. These feasts or dances are not social gatherings for the amusement of these people, but, on the contrary, are intended and calculated to stimulate the warlike passions of the young warriors of the tribe." -- Office of Indian Affairs, U.S. Department of the Interior, Code of Indian Offenses, 1883 What was the intended larger effect of the implementation of policies such as that being suggested in the above excerpt? a Reinstatement of American Indian sovereignty rights b Discontinuance of American Indian spiritual revivalism c Formal American Indian recognition of white cultural superiority d Destruction of American Indian tribal culture

d Destruction of American Indian tribal culture

The pattern of political party support shown on this map mostly reflects ongoing sectional divisions of the early- and mid-1800s over which issue? a The complete abolition of slavery in the United States b The passage and application of fugitive slave laws c The constitutionality of the doctrine of nullification d Expansion of slavery into the new western territories

d Expansion of slavery into the new western territories

It was in suburbs such as Garden Grove, Orange County [California] . . . that small groups of middle-class men and women met in their new tract homes, seeking to turn the tide of liberal dominance. Recruiting the like-minded, they organized study groups, opened 'Freedom Forum' bookstores, filled the rolls of the John Birch Society, entered school board races, and worked within the Republican Party, all in an urgent struggle to safeguard their particular vision of freedom and the American heritage. In doing so, they became the ground forces of a conservative revival—one that transformed conservatism from a marginal force preoccupied with communism in the early 1960s into a viable electoral contender by the decade's end." — Lisa McGirr, historian, Suburban Warriors: The Origins of the New American Right, 2001 The groups described in the excerpt most likely opposed a the Vietnam War b organized public prayer c racial segregation d Great Society programs

d Great Society programs

*picture of "the sacrilegious candidate" political cartoon* Which of the following policies most closely resembles the Democratic party's platform for the election of 1896 as a result of the idea depicted in the cartoon? a The continuance of Reconstruction b The implementation of greenbacks c Maintaining the gold standard d Inflation through the unlimited coinage of silver

d Inflation through the unlimited coinage of silver

Which of the following groups was most likely the intended audience of the cartoon above? a Former slaves b The Supreme Court c The Confederate leadership d Moderate Republicans

d Moderate Republicans

The sentiments of the image above are most clearly a continuation of what event or processes below? a The civil rights movement to protect the civil liberties of former slaves after the Civil War. b Technological change and modernization of the early 20th century. c Women's involvement in the reforms movements inspired by the Second Great Awakening. d Progressive movements to address social problems associated with an industrial society.

d Progressive movements to address social problems associated with an industrial society.

How did Progressive reformers attempt to better the lives of workers such as those in the photograph above? a Progressives focused their reform efforts exclusively at the local level in order to assist workers more directly. b Progressives pushed for a transition from a rural, agricultural society to an urban, industrial one. c Progressives called for less government intervention in the economy. d Progressives urged the creation of new organizations aimed at addressing social problems associated with an industrial society.

d Progressives urged the creation of new organizations aimed at addressing social problems associated with an industrial society.

"To begin with, then, I should say that more than two-thirds of the negroes of the town where I live are menial servants of one kind or another, and besides that more than two-thirds of the negro women here, whether married or single, are compelled to work for a living,--as nurses, cooks, washerwomen, chambermaids, seamstresses, hucksters, janitresses, and the like. I will say, also, that the condition of this vast host of poor colored people is just as bad as, if not worse than, it was during the days of slavery. Tho today we are enjoying a nominal freedom, we are literally slaves... Of course, nothing is being done to increase our wages, and the way things are going at the present it would seem that nothing could be done to cause an increase of wages. .... So that, the truth is, we have to work for little or nothing, or become vagrants! And that, of course, in this State would mean that we would be arrested, tried, and dispatched to the "State Farm," where we would surely have to work for nothing or be beaten with many stripes!" "More Slavery at the South," by an African American Nurse, independent A large number of African Americans in the South worked in what occupation after the Civil War? a Independent farmers b Strikebreakers c Factory workers d Sharecroppers

d Sharecroppers

One Way Ticket (Langston Hughes) I am fed up With Jim Crow laws, People who are cruel And afraid, Who lynch and run, Who are scared of me And me of them. I pick up my life And take it away On a one-way ticket Gone Up North Gone Out West Gone! The poem above refers to which event in U.S. history? a Reconstruction b World War I c The Civil War d The Great Migration

d The Great Migration

"New York is, I firmly believe, the most charitable city in the world. Nowhere is there so eager a readiness to help. When it is known that the help is worthily wanted; nowhere are such armies of devoted workers, nowhere such abundance of means ready to the hand of those who know the need and how rightly to supply it. Its poverty, its slums, and its suffering are the result of unprecedented growth with the consequent disorder and crowding, and the common penalty of metropolitan greatness....The Day Nurseries, the numberless Kindergartens and charitable schools in the poor quarters, the Fresh Air Funds, the thousands and one charities that in one way or another reach the homes and the lives of the poor with sweetening touch..." Jacob Riis, How the Other Half Lives, 1890 Which 19th-century reform movement was most closely associated with the activities described above? a Conservation b Populism c Social Darwinism d The Social Gospel

d The Social Gospel

"[I am] commanded to explain to the Japanese that...[the United States] population has rapidly spread through the country, until it has reached the shores of the Pacific Ocean; that we have now large cities, from which, with the aid of steam vessels, we can reach Japan in eighteen or twenty days; [and] that. ... the Japan seas will soon be covered with our vessels. "Therefore, as the United States and Japan are becoming every day nearer and nearer to each other, the President desires to live in peace and friendship with your imperial majesty, but no friendship can long exist, unless Japan ceases to act toward Americans as if they were her enemies.... "Many of the large ships-of-war destined to visit Japan have not yet arrived in these seas, though they are hourly expected; and [the United States has], as an evidence of [its] friendly intentions.... brought but four of the smaller ones, designing, should it become necessary, to return to Edo [Tokyo] in the ensuing spring with a much larger force." Commodore Matthew C. Perry to the emperor of Japan, letter, 1853 The population trend described in the excerpt most directly reflected which of the following domestic developments in the nineteenth century? a The dispute over whether Congress should reestablish a national bank b The question of the role of government in funding internal improvements c The claim that the United States should limit European colonialism in the Western Hemisphere d The belief that it was the Manifest Destiny of the United States to control territory across the continent

d The belief that it was the Manifest Destiny of the United States to control territory across the continent

"The era of big government is over but we can't go back to a time when our citizens were just left to fend for themselves. We will meet them by going forward as one America, by working together in our communities, our schools, our churches and synagogues, our workplaces across the entire spectrum of our civic life." - President Bill Clinton, radio address to the nation, 1996 Which of the following actions by the Clinton administration best reflects the ideas about the scope of government expressed in the excerpt? a The negotiation of new free trade agreements among North American countries b The effort to enact universal health care legislation c The decision to pursue military peacekeeping interventions in the Balkans and Somalia d The enactment of welfare reform to restrict benefits and encourage self-reliance

d The enactment of welfare reform to restrict benefits and encourage self-reliance

"Much of the national harmony had rested upon the existence of a kind of balance between the northern and southern parts of the United States. The decision to fight the [Mexican-American War] had disturbed this balance, and the acquisition of a new empire which each section desired to dominate endangered the balance further. Thus, the events which marked the culmination of six decades of exhilarating national growth at the same time marked the beginning of sectional strife which for a quarter century would subject American nationalism to its severest testing." — David M. Potter, historian, The Impending Crisis: America Before the Civil War, 1848-1861, published in 1976 Which of the following most directly contributed to the decision by the United States to fight the Mexican-American War? a The overcultivation of land in the Southeast b The economic competition that emerged during the market revolution c The efforts of the federal government to assert authority over American Indians d The idea of Manifest Destiny

d The idea of Manifest Destiny

"Excepting only Yosemite, Hetch Hetchy is the most attractive and wonderful valley within the bounds of the great Yosemite National Park and the best of all the camp grounds. People are now flocking to it in ever-increasing numbers for health and recreation of body and mind. Though the walls are less sublime in height than those of Yosemite, its groves, gardens, and broad, spacious meadows are more beautiful and picturesque. . . . Last year in October I visited the valley with Mr. William Keith, the artist. He wandered about from view to view, enchanted, made thirty-eight sketches, and enthusiastically declared that in varied picturesque beauty Hetch Hetchy greatly surpassed Yosemite. It is one of God's best gifts, and ought to be faithfully guarded." John Muir, Century Magazine, 1909 Which of the following aspects of Muir's description expresses a major change in Americans' views of the natural environment? a The idea that wilderness areas are worthy subjects for artistic works b The idea that wilderness areas serve as evidence of divine creation c The idea that mountainous scenery is more picturesque and beautiful than flat terrain d The idea that government should preserve wilderness areas in a natural state

d The idea that government should preserve wilderness areas in a natural state

"Much of the national harmony had rested upon the existence of a kind of balance between the northern and southern parts of the United States. The decision to fight the [Mexican-American War] had disturbed this balance, and the acquisition of a new empire which each section desired to dominate endangered the balance further. Thus, the events which marked the culmination of six decades of exhilarating national growth at the same time marked the beginning of sectional strife which for a quarter century would subject American nationalism to its severest testing." — David M. Potter, historian, The Impending Crisis: America Before the Civil War, 1848-1861, published in 1976 Which of the following historical developments during the nineteenth century best supports Potter's argument about the underlying cause of sectional conflict? a The influx of Irish and German immigrants to Northern cities b The increasing prominence of the Whig Party in national politics c The expansion of voting rights for White males d The increase in economic opportunities in the West

d The increase in economic opportunities in the West

*cartoon of old japanese man saying "patience young man... they get over it eventually", to which a US Muslim says "who are you", to which the man says "wwII interment camp survivor* The cartoon above is a response to which underlying issue? a Debates regarding how to preserve the racial identity of the United States b U.S. dependence on oil from the Middle East c Controversy over the U.S. attacks on Afghanistan and Iraq d The protection of civil liberties and human rights

d The protection of civil liberties and human rights

Which of the following trends of the 1920s is most clearly portrayed in this advertisement? a The emergence of General Motors as the largest company b The growth of middle-class incomes c The expansion of auto dealers throughout the country d The use of extended payment plans to purchase consumer goods

d The use of extended payment plans to purchase consumer goods

"We assert that fourteen of the states have deliberately refused for years past to fulfill their constitutional obligations, and we refer to their own statutes for proof....Those states have assumed the right of deciding upon the propriety of our domestic institutions; and have denied the rights of property established...and recognized by the Constitution...they have permitted the open establishment among them of societies, whose avowed object is to disturb the peace of and eloign [take away] the property of citizens of other States....A sectional party has found within...the Executive Department, the means of subverting the Constitution itself....On the 4th of March next this party will take possession of the Government....The guarantees of the Constitution will then no longer exist; the equal rights of the States will be lost. The Slaveholding States will no longer have the power of self-government, or self-protection, and the Federal Government will have become their enemy." South Carolina Declaration of the Causes of Secession, December 24, 1860 The ideas expressed in the passage above most clearly show the influence of which of the following? a Public debates about how to set national goals and priorities b The principles of federalism and separation of powers c Resistance to initiatives for democracy and inclusion d The use of states' rights for the defense of slavery

d The use of states' rights for the defense of slavery

*cartoon of old japanese man saying "patience young man... they get over it eventually", to which a US Muslim says "who are you", to which the man says "wwII interment camp survivor* Which post-World War II conflict is the proper context for the editorial cartoon above? a The Vietnam War b The Cold War c The Korean War d The war on terror

d The war on terror

"Article 2: [T]he United States now solemnly agrees that no persons . . . shall ever be permitted to pass over, settle upon, or reside in . . . this reservation for the use of said Indians. "Article 6: If any individual belonging to said tribes of Indians, or legally incorporated with them, being the head of a family, shall desire to commence farming, he shall have the privilege to select . . . a tract of land within said reservation, not exceeding three hundred and twenty acres in extent. "Article 11: [T]he tribes who are parties to this agreement hereby stipulate that they will relinquish all right to occupy permanently the territory outside their reservations . . . but yet reserve the right to hunt on any lands north of North Platte, and on the Republican Fork of the Smoky Hill river, so long as the buffalo may range thereon in such numbers as to justify the chase. . . . They will withdraw all opposition to the construction of the railroads now being built on the plains. . . . They will not attack any persons at home, or traveling, nor molest or disturb any wagon trains, coaches, mules, or cattle belonging to the people of the United States." Second Treaty of Fort Laramie, agreed between the United States government and various bands of the Sioux nation, 1868 Which of the following was typical of agreements such as the Fort Laramie Treaty between the United States government and American Indians in the post-Civil War West? a They generally led to the formation of strong, independent American Indian nations. b They frequently led to the formation of a common interest between the United States government and American Indians in controlling the activities of White settlers. c They led to the abandonment of most reservations as American Indian families sought economic opportunities in urban areas. d They usually lasted a short time before being broken by settlers' incursions onto American Indian reservations.

d They usually lasted a short time before being broken by settlers' incursions onto American Indian reservations.

We have been asked to believe that the inhabitants of Texas have been fighting to maintain the sacred principles of Liberty, and the natural, inalienable Rights of Man:---whereas, their motives have been exactly the opposite. The immediate cause and main goal of this war—led by the slaveholders of this country, (with land speculators and slave traders)---has been to grab the large and valuable territory of Texas from the Mexican Republic, in order to re- establish the SYSTEM OF SLAVERY; to open a vast and profitable SLAVEMARKET; and, ultimately, to annex it to the United States. Benjamin Lundy, The War in Texas, 1836 Which of the following groups would most likely support the views expressed in the excerpt a American settlers in Texas b Supporters of President Polk c Cotton Whigs d abolitionists

d abolitionists

Those people most opposed to President James K. Polk's expansionist program were the a proslavery Whigs b western Democrats c southerners d antislavery forces

d antislavery forces

"I object in the strongest possible way to having the United States agree, directly or indirectly, to be controlled by a [organization] which may at any time, and perfectly lawfully and in accordance with the terms of the covenant, be drawn in to deal with internal conflicts in other countries, no matter what those conflicts may be... There can be no genuine dispute whatever about the meaning of the first clause of article 10...In article 10 the United States is bound on the appeal of any member of the [organization] not only to respect but to preserve its independence and its boundaries, and that pledge if we give it, must be fulfilled." Henry Cabot Lodge Excerpt from speech delivered to Senate, August 12, 1919 The excerpt best symbolizes Lodge's efforts to a derail the Republican Party's foreign policy b object to support for the United Nations c rally against the New Deal programs d bolster opposition to the League of Nation

d bolster opposition to the League of Nation

"The Negro race, like all races, is going to be saved by its exceptional men. The problem of education, then, among Negroes must first of all deal with the Talented Tenth; it is the problem of developing the Best of this race that they may guide the Mass away from the contamination and death of theWorst, in their own and other races. Now the training of men is a difficult and intricate task. Its technique is a matter for educational experts, but its object is for the vision of seers. If we make money the object of man-training, we shall develop money-makers but not necessarily men; if we make technical skill the object of education, we may possess artisans but not, in nature, men. Men we shall have only as we make manhood the object of the work of the schools —intelligence, broad sympathy, knowledge of the world that was and is, and of the relation of men to it." W. E. B. Du Bois, "The Talented Tenth," 1903 The ideas expressed in the excerpt most directly contributed to the a persistence of economic discrimination based on race b expansion of legal segregation by the Supreme Court c large-scale African American migration to northern cities d emergence of organizations pursuing equality for African Americans

d emergence of organizations pursuing equality for African Americans

"Our...destiny [is] to overspread the continent allotted by Providence for the free development of our yearly multiplying millions...The Anglo-Saxon foot is already on [California's] borders. Already the advance guard of the irresistible army of the Anglo-Saxon emigration has begun to pour down upon it, armed with the [plow] and the rifle, and marking its trail with schools and colleges, courts and representative halls, mills, and meeting houses. A population will soon be in actual occupation of California...Their right to independence will be the natural right of self-government belonging to any community strong enough to maintain it." - John L. O'Sullivan, 1845 The process described in the passage above most directly led to political controversies in the 1840s and 1850s over the a authority of the Supreme Court to overturn federal laws b role of the federal government in economic development c use of natural resources in newly acquired territories d expansion of slavery into newly acquired territories

d expansion of slavery into newly acquired territories

"It was in suburbs such as Garden Grove, Orange County [California] . . . that small groups of middle-class men and women met in their new tract homes, seeking to turn the tide of liberal dominance. Recruiting the like-minded, they organized study groups, opened 'Freedom Forum' bookstores, filled the rolls of the John Birch Society, entered school board races, and worked within the Republican Party, all in an urgent struggle to safeguard their particular vision of freedom and the American heritage. In doing so, they became the ground forces of a conservative revival—one that transformed conservatism from a marginal force preoccupied with communism in the early 1960s into a viable electoral contender by the decade's end." — Lisa McGirr, historian, Suburban Warriors: The Origins of the New American Right, 2001 By the 1980s and 1990s, the strength of the movement described in the excerpt was best illustrated by the a increase in union membership among public employees b growth in the size and scope of the federal government c expanded role of the United States in international peacekeeping operations d prominence of evangelical Christian organizations in politics

d prominence of evangelical Christian organizations in politics ( i think)

The territorial changes shown in the southwestern region of the map most directly resulted from a treaties made with American Indian nations b the purchase of land from France and Spain c the Spanish-American War d the Mexican-American War

d the Mexican-American War

The Social Security Act was one of the most significant accomplishments of which of the following? a the First New Deal b the Socialist Party c the First Hundred Days d the Second New Deal

d the Second New Deal

*graph showing immigration rates going up each decade* The pattern depicted in the graph in the first half of the nineteenth century most directly resulted in a the formation of a political party that promoted nativism b a more unified national culture that embraced immigrants c federal provision of financial assistance to immigrants d the establishment of settlement houses

d the establishment of settlement houses (i think)

"I believe that progressivism was a radical movement thought not by the common measures of economic and political radicalism...Progressives were radical in their conviction that other social classes must be transformed and in their boldness in going about the business of that transformation...The sweep of progressivism was remarkable but because the progressive agenda was so often carried out in settlement houses, churches, and school rooms, in rather unassuming day-to-day activities the essential audacity of the enterprise can be missed. Progressivism demanded a social transformation that remains at once profoundly impressive and profoundly disturbing a century later." - Michael McGerr, A Fierce Discontent: The Rise and Fall of the Progressive Movement in American, 1870 - 1920. 2003. Which of the following movements from the period of 1870 to 1920 would most directly support McGerr's argument in the excerpt above? a The movement by the federal government to conserve and protect environmental resources b The movement to protect consumers from unfair practices by businesses c the movement by good-government advocates, who sought to eliminate public corruption d the movement for temperance and Prohibition, which sought to limit consumption of alcohol

d the movement for temperance and Prohibition, which sought to limit consumption of alcohol

"Our nation will continue to be steadfast, and patient and persistent in the pursuit of two great objectives. First, we will shut down terrorist camps, disrupt terrorist plans and bring terrorists to justice. And second, we must prevent the terrorists and regimes who seek chemical, biological or nuclear weapons from threatening the United States and the world....Our second goal is to prevent regimes that sponsor terror from threatening America or our friends and allies with weapons of mass destruction....States like these [North Korea, Iran, and Iraq], and their terrorist allies, constitute an axis of evil, arming to threaten the peace of the world. By seeking weapons of mass destruction, these regimes pose a grave and growing danger....And all nations should know: America will do what is necessary to ensure our nation's security." President George W. Bush, State of the Union Address, January 2002 The passage above was most likely a response to a a renewal of Cold War tensions with Russia. b fears of a resumption of the Korean War. c the continued U.S. dependence on fossil fuels from the Middle East. d the terrorist attacks that destroyed the World Trade Center.

d the terrorist attacks that destroyed the World Trade Center.

"The Republican electoral triumph in 2004 was the culmination of a half-century of struggle by the Right to achieve political power in the United States. In the immediate aftermath of the Second World War, a small band of intellectuals launched a movement to stop what they saw as the advance of the collectivist state embodied in modern liberalism and the New Deal political order. They were joined by anti-Communist activists across grassroots America....In their struggle against the dominant liberal state, conservatives gained control of the Republican party by defeating its liberal eastern wing." Donald T. Critchlow, historian, The Conservative Ascendancy: How the GOP Right Made Political History, 2007 The new conservative movement most consistently criticized modern liberalism by claiming that it a engaged in imperialist expansionism abroad b devoted too few resources to alleviating poverty c ignored racial discrimination and poverty at home d threatened traditional visions of morality

d threatened traditional visions of morality

The poster most directly reflects the a expanded access to consumer goods during wartime b emergence of the United States as a leading world power c wartime repression of civil liberties d wartime mobilization of United States society

d wartime mobilization of United States society

"What induced [American] Indians to go out of their way to trap beaver and trade the skins for glass beads, mirrors, copper kettles, and other goods? . . . Recent scholarship on [American] Indians? ' motives in this earliest stage of the trade indicates that they regarded such objects as the equivalents of the quartz, mica, shell, and other sacred substances that had formed the heart of long-distance exchange in North America for millennia. . . . While northeastern [American] Indians recognized Europeans as different from themselves, they interacted with them and their materials in ways that were consistent with their own customs and beliefs. " Neal Salisbury, historian, "The Indians' Old World: Native Americans and the Coming of Europeans, " 1996 Which of the following types of evidence would best support the argument in the excerpt? a. Diaries from British settlers in Philadelphia b. Spanish government reports about American Indians c. Shipping inventories from trade between Liverpool and Boston d. Artifacts from American Indian settlements

d. Artifacts from American Indian settlements

"An act for the better regulating the government of the province of the Massachusetts's Bay, in New England. I. ...Be it therefore enacted by the King's most excellent Majesty ... That the charter, granted ... to the inhabitants of the said province of the Massachusetts's Bay... be revoked, and is hereby revoked and made void and of none effect; ...And that ... the council, or court of assistants of the said province ... shall be thereunto nominated and appointed by his Majesty, III. And be it further enacted by the authority aforesaid, ... it shall and may be lawful for his Majesty's governor ... to nominate and appoint, ... and also to remove, without the consent of the council, all judges of the inferior courts of common pleas, commissioners of Oyer and Terminer, the attorney general, provosts, marshals, justices of the peace, and other officers to the council or courts of justice belonging ..." -- Massachusetts Government Act (one of the so-called Intolerable or Coercive Acts) Laws like that above were long-term consequences of events related to which of the following? a. Britain needed to exert political control over the colonists in order to rebuild its imperial army after several imperialistic wars. b. Britain failed to develop as many colonies around the world for commerce and taxation as its imperial rivals had. c. Britain hoped to win the allegiance of the colonies from its European rivals in North America. d. Britain needed Americans to help pay the debts of the Seven Years' War as well as military expenses for defending the colonists against American Indians.

d. Britain needed Americans to help pay the debts of the Seven Years' War as well as military expenses for defending the colonists against American Indians.

The advertisement depicted above provides evidence of which of the following? a. Results of a shifting population migrating from the Northeast to the West and South. b. A call to action to participate in Kennedy's call for volunteerism from his Inaugural Address. c. Effects of the abuses of American Indians' civil rights. d. Consequences of excessive industrialization and the corresponding economic prosperity.

d. Consequences of excessive industrialization and the corresponding economic prosperity.

*picture of gas station with closed pumps* the United States first suffered gasoline shortages such as depicted above during the Nixon Administration primarily because Arab oil exporters (OPEC) placed an oil embargo on a. Communist countries. b. Counties that were not a member of OPEC. c. Countries that did not support the Iranian Revolution in 1979. d. Counties which supported Israel during the Yom Kippur War

d. Counties which supported Israel during the Yom Kippur War

"For the increase of shipping . . . from thenceforward, no goods or commodities whatsoever shall be imported into or exported out of any lands, islands, plantations, or territories to his Majesty belonging . . . but in ships or vessels as do . . . belong only to the people of England . . . and whereof the master and three-fourths of the mariners at least are English. . . . "And it is further enacted . . . that . . . no sugars, tobacco, cottonwool, indigos, ginger, fustic, or other dyeing wood, of the growth, production, or manufacture of any English plantations in America, Asia, or Africa, shall be . . . transported from any of the said English plantations [colonies] to any land . . . other than to such other English plantations as do belong to his Majesty." English Parliament, Navigation Act of 1660 What accounts most for the scant success of the above law in modifying English colonial behavior in North America? a. The rejection of Anglicanism in the colonies b. The presence of slavery and growing colonial wars c. Weak economic growth and the lack of external competition d. Decades of the British government's relative indifference to colonial governance

d. Decades of the British government's relative indifference to colonial governance

"Slavery, though imposed and maintained by violence, was a negotiated relationship.... First, even as they confronted one another, master and slave had to concede, however grudgingly, a degree of legitimacy to the other.... [T]he web of interconnections between master and slave necessitated a coexistence that fostered cooperation as well as contestation. Second, because the circumstances of such contestation and cooperation continually changed, slavery itself continually changed....Slavery was never made, but instead was continually remade, for power—no matter how great—was never absolute, but always contingent." Ira Berlin, historian, Many Thousands Gone: The First Two Centuries of Slavery in North America, 1998 Which of the following primary sources would most likely support Berlin's argument in the excerpt? a. Records of purchases and sales of slaves from a plantation in the South b. Data showing the growth of the enslaved population during the 1700s c. Speeches about slavery given by officials in the British colonial government d. Diary entries from a slaveholder discussing plantation life

d. Diary entries from a slaveholder discussing plantation life

"There was with General Washington, during most of the summer, a Seneca chief, called The Great Tree, who, on leaving the head-quarters of the Commander-in-chief, professed the strongest friendship for the American cause, and his first object, after his return to his own people, was to inspire them with his own friendly sentiments. While passing through the Oneida nation on his way home, he professed the strongest confidence in his ability to keep his own tribe bound in the chain of friendship, and pledged himself, in the event of his failure, to come down with his friends and adherents and join the Oneidas. The Oneida messengers were farther told that all the Indians west of their own tribe, including, of course, the Onondagas, together with the Indian settlements on the Susquehanna and its branches, were to join them. They were to rendezvous somewhere on the Tioga, and make a descent either upon the Pennsylvania or New Jersey frontier." -- William E. Stone, Life of Joseph Brant - Thayendanegea: including the Border Wars of the American Revolution, published in 1838 The above excerpt most directly reflects which dominate goal of Native Americans during the colonial war for independence? a. Avoid alliances with other tribes b. Protect fur trade networks with the French c. To remain neutral in the conflict d. Forge advantageous alliances in hopes of protecting their own interests

d. Forge advantageous alliances in hopes of protecting their own interests

"Unlike some anticommunists. . . I have always believed that we can and must communicate and, when possible, negotiate with Communist nations. . . . "There were, however, a few things in our favor. The most important and interesting was the Soviet split with China. . . . "It was often said that the key to a Vietnam settlement lay in Moscow and Peking rather than in Hanoi. . . . Aside from wanting to keep Hanoi from going over to Peking, Moscow had little stake in the outcome of the North Vietnamese cause, especially as it increasingly worked against Moscow's own major interests vis-à-vis the United States. While I understood that the Soviets were not entirely free agents where their support for North Vietnam was concerned, I nonetheless planned to bring maximum pressure to bear on them in this area." —Richard Nixon, RN: Memoirs of Richard Nixon, 1978 Which of the following was Nixon able to negotiate as a result of his new relationship with China? a. Withdrawal of Chinese troops from Vietnam b. Autonomy for Poland in Eastern Europe c. A peace treaty between South and North Korea d. Freeze on the number of US and USSR ballistic missiles

d. Freeze on the number of US and USSR ballistic missiles

"New France enter[ed] its golden age in the first decades of the eighteenth century. . . . In Louisiana, theIllinois country, and the Great Lakes basin, French cities and villages developed alongside Indian villages. . . . Here, natives and Europeans found that their different goals were complementary. The French posed no demographic threat. . . . The landscape of Indian life had not been seriously altered. The fur trade depended on the integrity of that landscape." Jay Gitlin, historian, "Empires of Trade, Hinterlands of Settlement," 1994 Which of the following evidence could best be used to support Gitlin's argument in the excerpt? a. The use of slave labor to produce sugar in Louisiana b. Military conflicts between the French, Dutch, British, and Spanish c. European efforts to arm American Indians d. Intermarriage between French colonists and American Indians

d. Intermarriage between French colonists and American Indians

"[W]e must, by means of a rapid and sustained build-up of the political, economic, and military strength of the free world, and by means of an affirmative program intended to wrest the initiative from the Soviet Union, confront it with convincing evidence of the determination and ability of the free world to frustrate the Kremlin design of a world dominated by its will. Such evidence is the only means short of war which eventually may force the Kremlin to abandon its present course of action and to negotiate acceptable agreements on issues of major importance. "The whole success of the proposed program hangs ultimately on recognition by this Government, the American people, and all free peoples, that the cold war is in fact a real war in which the survival of the free world is at stake. . . . The prosecution of the program will require of us all the ingenuity, sacrifice, and unity demanded by the vital importance of the issue and the tenacity to persevere until our national objectives have been attained." NSC-68: United States Objectives and Programs for National Security, 1950 Which of the following United States actions most directly resulted from the goals expressed in the excerpt? a. Passage of new immigration laws b. Investigations to find communist subversives in government jobs c. Application of federal power to try to end poverty d. Intervention in the conflict in Korea

d. Intervention in the conflict in Korea

The goals presented in the advertisement have the most in common with which of the following? a. Attempts to civilize the American Indians during the late 19th century with the development of educational institutions where young Indians were assimilated. b. Populist backlash to tariffs and the monetization of silver. c. Deregulation efforts in the 1980s. d. Legislative action initiated by President Theodore Roosevelt in the early 1900s to preserve nationals lands.

d. Legislative action initiated by President Theodore Roosevelt in the early 1900s to preserve nationals lands.

The "women's lib" movement is not an honest effort to secure better jobs for women whowant or need to work outside the home. This is just the superficial sweet-talk to win broad support for a radical "movement." Women's lib is a total assault on the role of the American woman as wife and mother, and on the family as the basic unit of society. Women's libbers are trying to make wives and mothers unhappy with their career, make them feel that they are "second-class citizens" and "abject slaves." Women's libbers are promoting free sex instead of the "slavery" of marriage. They are promoting Federal "day-care centers" for babies instead of homes. They are promoting abortions instead of families. - Phyllis Schlafly, What's Wrong with 'Equal Rights' for Women?, February 1972 In contrast to Phyllis Schlafly, Betty Friedan's book The Feminist Mystique, which launched a new wave of feminism when it was first published in 1963, concluded that a. A majority of women already worked outside the home but were unhappy b. Most women were content to serve their husbands and children in the home c. Most women felt that going to college and having a career was unfeminine d. Many women were dissatisfied with being homemakers and felt unfulfilled

d. Many women were dissatisfied with being homemakers and felt unfulfilled

You must understand—I must make you understand—that our membership and the hopes and aspirations of the hundreds of thousands of the poor and dispossessed that have been raised on our account are, above all, human beings... The colors of our skins, the languages of our cultural and native origins, the lack of formal education, the exclusion from the democratic process, the numbers of our slain in recent wars—all these burdens generation after generation have sought to demoralize us, to break our human spirit. But God knows that we are not beasts of burden, agricultural implements, or rented slaves; we are men. . . . We advocate militant nonviolence as our means for social revolution and to achieve justice for our people, but we are not blind or deaf to the desperate and moody winds of human frustration, impatience and rage that blow among us. . . . We hate the agribusiness system that seeks to keep us enslaved, and we shall overcome and change it not by retaliation or bloodshed but by a determined nonviolent struggle carried on by those masses of farm workers who intended to be free and human. Cesar Chavez, Letter from Delano (1969) Cesar Chavez and other leaders of migrant farm workers were inspired by the tactics of: a. the zoot suit riots. b. the Stonewall Inn riot. c. Franklin Roosevelt. d. Martin Luther King, Jr.

d. Martin Luther King, Jr.

"For the increase of shipping . . . from thenceforward, no goods or commodities whatsoever shall be imported into or exported out of any lands, islands, plantations, or territories to his Majesty belonging . . . but in ships or vessels as do . . . belong only to the people of England . . . and whereof the master and three-fourths of the mariners at least are English. . . . "And it is further enacted . . . that . . . no sugars, tobacco, cottonwool, indigos, ginger, fustic, or other dyeing wood, of the growth, production, or manufacture of any English plantations in America, Asia, or Africa, shall be . . . transported from any of the said English plantations [colonies] to any land . . . other than to such other English plantations as do belong to his Majesty." English Parliament, Navigation Act of 1660 Which of the following had the most impact in shaping colonial resistance to the above law? a. The growth of the African slave trade b. Conflicts with American Indian groups c. Colonial dedication to mercantilist principles d. Political thought shaped by the Enlightenment

d. Political thought shaped by the Enlightenment

*graph showing general increase in government spending in various sectors (national defense, social service)* Which of the following developments after the Second World War most directly enabled the total spending shown in the table? a. The economic impact of declining northern industrial cities b. The conspicuous consumption of the baby-boom generation c. Significant increases in immigration that transformed fiscal policy d. Rising general prosperity and an expanding middle class

d. Rising general prosperity and an expanding middle class

The idea in the political cartoon was most directly associated with which of the following events in United States history? a. The Cold War b. The Marshall Plan c. Containment d. The Cuban Missile Crisis

d. The Cuban Missile Crisis

"For the increase of shipping . . . from thenceforward, no goods or commodities whatsoever shall be imported into or exported out of any lands, islands, plantations, or territories to his Majesty belonging . . . but in ships or vessels as do . . . belong only to the people of England . . . and whereof the master and three-fourths of the mariners at least are English. . . . "And it is further enacted . . . that . . . no sugars, tobacco, cottonwool, indigos, ginger, fustic, or other dyeing wood, of the growth, production, or manufacture of any English plantations in America, Asia, or Africa, shall be . . . transported from any of the said English plantations [colonies] to any land . . . other than to such other English plantations as do belong to his Majesty." English Parliament, Navigation Act of 1660 Which of the following most likely motivated Parliament to pass the law in the excerpt? a. The desire to promote migration to the colonies b. The decline of a strong English identity among colonists c. The formation of colonial governments that differed from English models d. The desire to pursue mercantilist goals

d. The desire to pursue mercantilist goals

*graph showing a sharp increase in households with TV sets from 1940-1960* Which of the following best characterizes the cultural impact of the growth shown in this graph? a. Unique programming available in various markets led to increasing regionalization of American values. b. The material gap between the rich and poor exacerbated the social divisions along class lines. c. Women's role outside the home was increasingly accepted, based in part on progressive depictions of working women in television. d. The dominant portrayals of traditional parents and their children led to the idealization of the nuclear- family life.

d. The dominant portrayals of traditional parents and their children led to the idealization of the nuclear- family life (i think)

"In colonial New England, two sets of human communities which were also two sets of ecological relationships confronted each other, one Indian and one European. They rapidly came to inhabit a single world, but in the process the landscape of New England was so transformed that the Indians' earlier way of interacting with the environment became impossible. The task before us is not only to describe the ecological changes that took place in New England but to determine what it was about Indians and colonists—in their relations both to nature and to each other—that brought those changes about." William Cronon, historian, Changes in the Land: Indians, Colonists, and the Ecology of New England, 1983 Which of the following best supports the general argument in the excerpt about how Europeans changed North America? a. The efforts to restrict settlement beyond the Appalachian Mountains b. The emergence of racially and culturally mixed populations c. The spread of maize agriculture d. The establishment of fenced fields on family farms

d. The establishment of fenced fields on family farms

"For each of us, as for the robin in Michigan or the salmon in the Miramichi, this is a problem of ecology, of interrelationships, of interdependence. We poison the caddis flies in the stream and the salmon runs dwindle and die.... We spray our elms and the following springs are silent of robin song, not because we sprayed the robins directly but because the poison traveled, step by step, through the now familiar elm leaf-earthworm-robin cycle. These are matters of record, observable, part of the visible world around us." Rachel Carson, Silent Spring, 1962 Which of the following developments contributed most directly to the conditions described in the excerpt? a. The erosion of soil on the Great Plains b. The increase in government regulation of natural resources c. The growth of higher education d. The growth of suburban housing on previously undeveloped land

d. The growth of suburban housing on previously undeveloped land

"The colonizers brought along plants and animals new to the Americas, some by design and others by accident. Determined to farm in a European manner, the colonists introduced their domesticated livestock—honeybees, pigs, horses, mules, sheep, and cattle—and their domesticated plants, including wheat, barley, rye, oats, grasses, and grapevines. But the colonists also inadvertently carried pathogens, weeds, and rats. . . . In sum, the remaking of the Americas was a team effort by a set of interdependent species led and partially managed (but never fully controlled) by European people." - Alan Taylor, historian, American Colonies, 2001 The trends described by Taylor most directly illustrate which of the following major historical developments in the Atlantic world? a. The growth of mercantile empires that stretched across the Atlantic b. The rise of the trans-Atlantic slave trade c. The increasing anglicization of the English colonies d. The phenomenon known as the Columbian Exchange

d. The phenomenon known as the Columbian Exchange

"What began as a protest movement is being challenged to translate itself into a political movement. It is now concerned not merely with removing the barriers to full opportunity but with achieving the fact of equality. From sit-ins and freedom rides we have gone into rent strikes, boycotts, community organization, and political action. As a consequence of this natural evolution, the Negro today finds himself stymied by obstacles of far greater magnitude than the legal barriers he was attacking before: automation, urban decay, de facto school segregation." — Bayard Rustin, "From Protest to Politics," 1965 The excerpt was written most directly in response to which of the following? a. The development of a counterculture that rejected many of the values of the previous generation b. The emergence of a distinctive African American arts and literature during the Harlem Renaissance c. The racial desegregation of the United States military d. The success of the Civil Rights movement in achieving legal and legislative victories

d. The success of the Civil Rights movement in achieving legal and legislative victories

"We are a people of cultivators, scattered over an immense territory, communicating with each other by means of good roads and navigable rivers, united by the silken bands of mild government, all respecting the laws, without dreading their power, because they are equitable. We are all animated with the spirit of an industry which is unfettered and unrestrained, because each person works for himself...whence came all these people? They are a mixture of English, Scotch, Irish, French, Dutch, Germans, and Swedes. From this promiscuous breed, that race now called Americans have arisen...What then is the American, this new man? He is either an European, or the descendant of an European, hence that strange mixture of blood, which you will find in no other country. I could point out to you a family whose grandfather was an Englishman, whose wife was Dutch, whose son married a French woman, and whose present four sons have now four wives of different nations. He is an American, who, leaving behind him all his ancient prejudices and manners, receives new ones from the new mode of life he has embraced, the new government he obeys, and the new rank he holds." J. Hector St. John Crèvecoeur, "What Is an American?" 1782 In this excerpt, Crévecoeur's description of America has the most in common with which of the following? a. Charles Cotesworth Pinckney's vision of a nation based on slave labor b. Alexander Hamilton's vision of a great manufacturing nation c. Tecumseh's vision of an independent Indian nation d. Thomas Jefferson's vision of a nation of independent and diverse laborers.

d. Thomas Jefferson's vision of a nation of independent and diverse laborers.

"Current sit-ins and other demonstrations are concerned with something much bigger than a hamburger. . . .Whatever may be the difference in approach to their goal . . . students, North and South, are seeking to rid America of the scourge of . . . discrimination—not only at lunch counters, but in every aspect of life." Ella Baker, "Bigger Than a Hamburger," Raleigh, NC, 1960 The tactics described in the excerpt best represent which of the following? a. Filing legal challenges b. Petitioning government officials c. Learning self-defense d. Using nonviolence

d. Using nonviolence

"We know through painful experience that freedom is never voluntarily given by the oppressor; it must be demanded by the oppressed. Frankly, I have yet to engage in a direct-action campaign that was 'well timed' in the view of those who have not suffered unduly from the disease of segregation. . . . We must come to see, with one of our distinguished jurists, that 'justice too long delayed is justice denied.' We have waited for more than 340 years for our constitutional and God-given rights. The nations of Asia and Africa are moving with jet like speed toward gaining political independence, but we still creep at horse-and-buggy pace toward gaining a cup of coffee at a lunch counter." Martin Luther King, Jr., African American leader, "Letter from Birmingham Jail," 1963 "The White man knows that the Black revolution is worldwide. . . . So I cite these various revolutions, brothers and sisters, to show you that you don't have a peaceful revolution. You don't have a turn-the-other-cheek revolution. There's no such thing as a nonviolent revolution. The only kind of revolution that's nonviolent is the Negro revolution. The only revolution in which the goal is loving your enemy is the Negro revolution. It's the only revolution in which the goal is a desegregated lunch counter, a desegregated theater, a desegregated park, and a desegregated public toilet. . . . That's no revolution. Revolution is based on land. . . . Land is the basis of freedom, justice, and equality. . . . A revolutionary wants land so he can set up his own nation, an independent nation." Malcolm X, African American leader, "Message to the Grass Roots," 1963 In noting that he had "yet to engage in a direct-action campaign that was 'well timed,'' Martin Luther King, Jr., was most likely arguing against a. environmental activists who believed that the struggle for more effective conservation measures should take precedence over the Civil Rights movement b. African American activists who asserted that King's tactics were taking too long to produce results c. Civil Rights activists who believed that more thought should be given to the precise timing of boycotts, demonstrations, and direct actions d. White Americans who argued that African Americans should be more patient and that civil rights should not be brought about by civil disobedience tactics

d. White Americans who argued that African Americans should be more patient and that civil rights should not be brought about by civil disobedience tactics

"By the end of 1949, only one out of three heroines in the women's magazines was a career woman—and she was shown in the act of renouncing her career and discovering that what she really wanted to be was a housewife. In 1958, and again in 1959, I went through issue after issue of the three major women's magazines (the fourth, Woman's Home Companion, had died) without finding a single heroine who had a career, a commitment to any work, art, profession, or mission in the world, other than "Occupation: housewife." Only one in a hundred heroines had a job; even the young unmarried heroines no longer worked except at snaring a husband." --Betty Friedan, journalist, The Feminine Mystique, published in 1963 The mid-20th century idealization of womanhood as described by Friedan is most like which conception of womanhood from the 19th century? a. Women's role earning income outside the home, such as in the factories b. Moral superiority of women c. Limiting of "women's" work to social reform efforts d. Women's separate sphere of domesticity

d. Women's separate sphere of domesticity

"The colonizers brought along plants and animals new to the Americas, some by design and others by accident. Determined to farm in a European manner, the colonists introduced their domesticated livestock—honeybees, pigs, horses, mules, sheep, and cattle—and their domesticated plants, including wheat, barley, rye, oats, grasses, and grapevines. But the colonists also inadvertently carried pathogens, weeds, and rats. . . . In sum, the remaking of the Americas was a team effort by a set of interdependent species led and partially managed (but never fully controlled) by European people." - Alan Taylor, historian, American Colonies, 2001 The export of New World crops to the Old World transformed European society mostly by a. encouraging enclosure of open lands and pushing workers off of farms b. promoting greater exploration of the interior of the American continents c. fostering conflicts among major powers over access to new food supplies d. improving diets and thereby stimulating population growth

d. improving diets and thereby stimulating population growth

"Good evening, my fellow citizens. This Government, as promised, has maintained the closest surveillance of the Soviet military build-up on the island of Cuba. Within the past week unmistakable evidence has established the fact that a series of offensive missile sites is now in preparation on that imprisoned island. The purposes of these bases can be none other than to provide a nuclear strike capability against the Western Hemisphere...Each of these missiles, in short, is capable of striking Washington, D. C., the Panama Canal, Cape Canaveral, Mexico City, or any other city in the southeastern part of the United States, in Central America, or in the Caribbean area...In addition, jet bombers, capable of carrying nuclear weapons, are now being uncrated and assembled in Cuba, while the necessary air bases are being prepared. This urgent transformation of Cuba into an important strategic base—by the presence of these large, long-range, and clearly offensive weapons of sudden mass destruction—constitutes an explicit threat to the peace and security of all the Americas...This action also contradicts the repeated assurances of Soviet spokesmen, both publicly and privately delivered, that the arms build-up in Cuba would retain its original defensive character and that the Soviet Union had no need or desire to station strategic missiles on the territory of any other nation...But this secret, swift, and extraordinary build-up of Communist missiles—in an area well known to have a special and historical relationship to the United States and the nations of the Western Hemisphere, in violation of Soviet assurances, and in defiance of American and hemispheric policy...is a deliberately provocative and unjustifiable change in the status quo which cannot be accepted by this country if our courage and our commitments are ever to be trusted again by either friend or foe." John F. Kennedy, Cuban Missile Crisis Address, 1962 Which of the following most directly contributed to the missile build-up in Cuba? a. the escape of Cubans taking refuge in the U.S. b. the surveillance of Cuba by U-2 spy planes c. the enmity between Kennedy and Khrushchev d. the American invasion of Cuba at the Bay of Pigs

d. the American invasion of Cuba at the Bay of Pigs

*Picture of states acquired by Northwest Ordinance* The Northwest Ordinance of 1787 was one of the most significant accomplishments of which of the following? a. the United States Congress b. the Second Continental Congress c. the First Continental Congress d. the Articles of Confederation Congress

d. the Articles of Confederation Congress

"Whereas it is... essential to our Interest, and the Security of our Colonies, that the several Nations or Tribes of Indians ...who live under our Protection, should not be molested or disturbed ...--We do therefore,...declare it to be our Royal Will and Pleasure... for the use of the said Indians, [to be reserved] all the Lands and Territories ...lying to the Westward of the Sources of the Rivers [along the Appalachian Mountains] which fall into the Sea....And We do hereby strictly forbid... all our loving Subjects from... taking Possession of any of the Lands above reserved..." The Royal Proclamation, King George III, October 1763 The Proclamation above sought to address British-Indian relations following the conclusion of a. the American Revolution. b. the Pueblo Revolt. c. the Atlantic slave trade. d. the French and Indian War.

d. the French and Indian War.

"Col. MASON. Slavery...bring[s] the judgement of heaven on a Country. Mr. ELSWORTH. Let us not intermeddle. As population increases poor laborers will be so plenty as to render slaves useless. Slavery in time will not be a speck in our Country... abolition has already taken place in Massachusetts. General PINKNEY. S. Carolina and Georgia cannot do without slaves. The more slaves, the more produce to emply the carrying trade; the more consumption also, and the more of this, the more of revenue for the common treasury. He...should consider a rejection of the clause as an exclusion of S. Carola. from the Union." James Madison, Notes from the Constitutional Convention, August 22, 1787 Based on the context of his remarks, Col. Mason would most likely receive support for his position on this issue in a. the Northwest Territories b. the Southwest c. the South d. the North

d. the North

"Thus, fellow citizens, have I pointed out what I thought necessary to be amended in our Federal Constitution. I beg you to call to mind our glorious Declaration of Independence, read it, and compare it with the Federal Constitution; what a degree of apostacy will you not then discover. Therefore, guard against all encroachments upon your liberties so dearly purchased with the costly expense of blood and treasure." A Georgian, Gazette of the State of Georgia, November 15, 1787 The views expressed in the excerpt contributed most directly to a. the creation of a strong central government b. a series of rebellions and revolutions in France, Haiti, and Latin America c. the elimination of the international slave trade d. the addition of the Bill of Rights shortly after the Constitution was adopted

d. the addition of the Bill of Rights shortly after the Constitution was adopted

"...Whereas the enforcing of the conscience in matters of religion has frequently fallen out to be of dangerous consequence in those commonwealths where it has been practiced, and for the more quiet and peaceable government of this Province, and the better to preserve mutual love and amity among the inhabitants thereof. Be it therefore enacted that no person or persons whatever in the Province...professing to believe in Jesus Christ, shall from henceforth be any ways troubled, molested, or discountenanced for or in respect of his or her religion nor in the free exercise thereof within the Province...nor in any way compelled to the belief or exercise of any other religion against his or her consent, so [long] as they not be unfaithful to the Lord Proprietary, or molest or conspire against the civil government established in this Province under him..." Maryland Act Concerning Religion, 1644 The passage above was most likely written in response to a. the expanding social networks of the Atlantic World. b. Protestant evangelism in North America. c. the growth of autonomous political communities based on English models. d. the ethnic and religious diversity of the middle colonies

d. the ethnic and religious diversity of the middle colonies

"We know through painful experience that freedom is never voluntarily given by the oppressor; it must be demanded by the oppressed. Frankly, I have yet to engage in a direct-action campaign that was 'well timed' in the view of those who have not suffered unduly from the disease of segregation. . . . We must come to see, with one of our distinguished jurists, that 'justice too long delayed is justice denied.' We have waited for more than 340 years for our constitutional and God-given rights. The nations of Asia and Africa are moving with jet like speed toward gaining political independence, but we still creep at horse-and-buggy pace toward gaining a cup of coffee at a lunch counter." Martin Luther King, Jr., African American leader, "Letter from Birmingham Jail," 1963 "The White man knows that the Black revolution is worldwide. . . . So I cite these various revolutions, brothers and sisters, to show you that you don't have a peaceful revolution. You don't have a turn-the-other-cheek revolution. There's no such thing as a nonviolent revolution. The only kind of revolution that's nonviolent is the Negro revolution. The only revolution in which the goal is loving your enemy is the Negro revolution. It's the only revolution in which the goal is a desegregated lunch counter, a desegregated theater, a desegregated park, and a desegregated public toilet. . . . That's no revolution. Revolution is based on land. . . . Land is the basis of freedom, justice, and equality. . . . A revolutionary wants land so he can set up his own nation, an independent nation." Malcolm X, African American leader, "Message to the Grass Roots," 1963 Martin Luther King, Jr., and Malcolm X shared the view that a. it was desirable to work for the desegregation of public facilities b. using the courts to challenge the constitutionality of barriers to Black advancement was not a useful strategy c. violence was a necessary part of the movement for Black social and political advancement d. the movement for Black advancement in the United States was part of a global process of social change

d. the movement for Black advancement in the United States was part of a global process of social change

"Good evening, my fellow citizens. This Government, as promised, has maintained the closest surveillance of the Soviet military build-up on the island of Cuba. Within the past week unmistakable evidence has established the fact that a series of offensive missile sites is now in preparation on that imprisoned island. The purposes of these bases can be none other than to provide a nuclear strike capability against the Western Hemisphere...Each of these missiles, in short, is capable of striking Washington, D. C., the Panama Canal, Cape Canaveral, Mexico City, or any other city in the southeastern part of the United States, in Central America, or in the Caribbean area...In addition, jet bombers, capable of carrying nuclear weapons, are now being uncrated and assembled in Cuba, while the necessary air bases are being prepared. This urgent transformation of Cuba into an important strategic base—by the presence of these large, long-range, and clearly offensive weapons of sudden mass destruction—constitutes an explicit threat to the peace and security of all the Americas...This action also contradicts the repeated assurances of Soviet spokesmen, both publicly and privately delivered, that the arms build-up in Cuba would retain its original defensive character and that the Soviet Union had no need or desire to station strategic missiles on the territory of any other nation...But this secret, swift, and extraordinary build-up of Communist missiles—in an area well known to have a special and historical relationship to the United States and the nations of the Western Hemisphere, in violation of Soviet assurances, and in defiance of American and hemispheric policy...is a deliberately provocative and unjustifiable change in the status quo which cannot be accepted by this country if our courage and our commitments are ever to be trusted again by either friend or foe." John F. Kennedy, Cuban Missile Crisis Address, 1962 The resolution of the Cuban Missile Crisis involved which of the following? a. the repudiation of the Platt Amendment b. the involvement of American troops in Cuba c. the provision of American non-military aid to Cuba d. the removal of American missiles in Turkey

d. the removal of American missiles in Turkey

"General Pinkney thought himself bound to declared candidly that he did not think South Carolina would stop her importation of slaves in any short time. He moved to committ the clause that slaves might be made [taxable]. Mr. Rutledge: If the Convention thinks that North Carolina, South Carolina and Georgia will ever agree to the plan, unless their right to import slaves be untouched, the expectation is [in] vain. The people of those States will never be such fools as to give up so important an interest. ... He seconded the motion of General Pinkney for a committment." - Notes from the Constitutional Convention, August 22, 1787 Which of the following best summarizes the discussion excerpted above? a. interstate commerce compromise b. the Electoral College compromise c. the 3/5ths compromise d. the slave trade compromise

d. the slave trade compromise

*Picture of Boston Massacre, showing the British shooting colonists dead* This image would have been used for which of the following purposes? a. to frighten African Americans away from escaping their masters b. to arouse support for the olive Branch Petition c. To warn colonists of the repercussions of disloyalty d. to provoke the anger of the citizens after the end of salutary neglect.

d. to provoke the anger of the citizens after the end of salutary neglect.

"With a profound sense of the solemn and even tragical character of the step I am taking and of the grave responsibilities which it involves, but in unhesitating obedience to what I deem my constitutional duty, I advise that the Congress declare the recent course of the Imperial German Government to be in fact nothing less than war against the...United States; that it formally accept the status of belligerent which has thus been thrust upon it....Neutrality is no longer feasible or desirable where the peace of the world is involved and the freedom of its peoples, and the menace to that peace and freedom lies in the existence of autocratic governments...not by the will of their people. We shall, happily, still have an opportunity to prove that friendship...towards the millions of men and women of German birth and native sympathy, who live amongst us...and we shall be proud to prove it towards all who are in fact loyal to their neighbors and to the Government....They are, most of them, as true and loyal Americans as if they had never known any other...allegiance. If there should be disloyalty, it will be dealt with a firm hand of stern repression...." Woodrow Wilson, Address to Congress's War, April 2, 1917 Which foreign policy approach is most consistent with the sentiments expressed by Wilson in the excerpt above? a The United States taking a leading military role in the war b The maintaining of isolationism by the United States c The pursuit of a unilateral foreign policy d The defense of humanitarian and democratic principles

either c or d, idrk. i think d?

The pattern depicted in the graph in the first half of the nineteenth century most directly resulted in a the formation of a political party that promoted nativism b federal provision of financial assistance to immigrants c the establishment of settlement houses d a more unified national culture that embraced immigrants

the formation of a political party that promoted nativism

"This morning the mob again gathered in front of the Central High School of Little Rock, obviously for the purpose of again preventing the carrying out of the Court's order relating to the admission of Negro children to the school. "Whenever normal agencies prove inadequate to the task and it becomes necessary for the Executive Branch of the Federal Government to use its powers and authority to uphold Federal Courts, the President's responsibility is inescapable. "In accordance with that responsibility, I have today issued an Executive Order directing the use of troops under Federal authority to aid in the execution of Federal law at Little Rock, Arkansas." President Dwight Eisenhower, national television and radio address, 1957 Which of the following events represented a continuation of the development discussed in the excerpt? A The advocacy for the passage of the Civil Rights Act of 1964 B The opposition of activists to the Plessy v. Ferguson decision C The increasing debates over racial equality by African American military veterans D The Great Migration of African Americans from the rural South to urban areas in the North and West

The advocacy for the passage of the Civil Rights Act of 1964

"Article 2: [T]he United States now solemnly agrees that no persons . . . shall ever be permitted to pass over, settle upon, or reside in . . . this reservation for the use of said Indians. "Article 6: If any individual belonging to said tribes of Indians, or legally incorporated with them, being the head of a family, shall desire to commence farming, he shall have the privilege to select . . . a tract of land within said reservation, not exceeding three hundred and twenty acres in extent. "Article 11: [T]he tribes who are parties to this agreement hereby stipulate that they will relinquish all right to occupy permanently the territory outside their reservations . . . but yet reserve the right to hunt on any lands north of North Platte, and on the Republican Fork of the Smoky Hill river, so long as the buffalo may range thereon in such numbers as to justify the chase. . . . They will withdraw all opposition to the construction of the railroads now being built on the plains. . . . They will not attack any persons at home, or traveling, nor molest or disturb any wagon trains, coaches, mules, or cattle belonging to the people of the United States." Second Treaty of Fort Laramie, agreed between the United States government and various bands of the Sioux nation, 1868 Which of the following contributed to reducing the conflict that article 11 and similar provisions of other treaties were designed to address? a The rerouting of several major railroads to avoid tribal lands b The implementation of government conservation policies that protected large areas of public land c The destruction of nearly the entire population of buffalo d A decrease in the number of White settlers traveling near reservations

The destruction of nearly the entire population of buffalo

"...the occasion has been judged proper for asserting, as a principle in which the rights and interests of the United States are involved, that the American continents, by the free and independent condition which they have assumed and maintain, are henceforth not to be considered as subjects for future colonization by any European powers...In the wars of the European powers in matters relating to themselves we have never taken any part, nor does it comport with our policy to do so...With the movements in this hemisphere we are of necessity more immediately connected, and by causes which must be obvious to all enlightened and impartial observers...We owe it, therefore, to candor and to the amicable relations existing between the United States and those powers to declare that we should consider any attempt on their part to extend their system to any portion of this hemisphere as dangerous to our peace and safety. With the existing colonies or dependencies of any European power we have not interfered and shall not interfere. But with the Governments who have declared their independence and maintain it, and whose independence we have, on great consideration and on just principles, acknowledged, we could not view any interposition for the purpose of oppressing them, or controlling in any other manner their destiny, by any European power in any other light than as the manifestation of an unfriendly disposition toward the United States." Monroe Doctrine, 1823 The excerpt most directly reflects which of the following goals for the Americas? a. no further European colonization in the Americas and the U.S. would not allow European interference with independent nations in the AmericasThis answer is correct. b. the foundation of a Pan-American organization of collective security c. no further United States interference in South America in exchange for no European intervention in the Caribbean islands d. the U.S. conquest and government of North and South America

a

"I come to present the strong claims of suffering humanity. I come as the advocate of helpless, forgotten, insane and idiotic men and women; of beings sunk to a condition from which the most unconcerned would start with real horror; of beings wretched in our Prisons, and more wretched in our Alms-Houses....I proceed, Gentlemen, briefly to call your attention to the present state of Insane Persons confined within this Commonwealth, in cages, closets, cellars, stalls, pens! Chained, naked, beaten with rods, and lashed into obedience!...The crying evil and abuse of institutions, is not confined to our almshouses. The warden of a populous prison near this metropolis, populous, not with criminals only, but with the insane in almost every stage of insanity...has declared that: "the prison has often more resembled the infernal regions than any place on earth!"...Gentlemen, I commit to you this sacred cause. Your action upon this subject will affect the present and future condition of hundreds and of thousands. In this legislation, as in all things, may you exercise that "wisdom which is the breath of the power of God." Dorothea Dix, Memorial to the Legislature of Massachusetts, 1843Dorothea L. Dix, Memorial to the Legislature of Massachusetts (Boston: Munroe & Francis, 1843). The efforts described in the excerpt above can best be understood in the context of a. attempts to match democratic political ideals with social realities. b. the development of distinctive cultures by various groups of people. c. governmental and private efforts to promote the American System. d. racist and nativist theories used to justify violence and segregation.

a

"Joseph Smith... came from nowhere. Reared in a poor Yankee farm family, he had less than two years of formal schooling and began life without social standing or institutional backing. His family rarely attended church. Yet in the fourteen years he headed the Church of Jesus Christ of Latter-day Saints, Smith created a religious culture that survived his death, flourished in the most desolate regions of the United States, and continues to grow worldwide. . . . In 1830 at the age of twenty-four, he published the Book of Mormon. . . .He built cities and temples and gathered thousands of followers before he was killed at age thirty-eight." Richard Lyman Bushman, historian, Joseph Smith Rough Stone Rolling: A Cultural Biography of Mormonism's Founder, 2005 Based on the excerpt, the westward migration by the Mormons in the 1830s and 1840s was most likely motivated by the a. need to take refuge from persecution b. declining threat of hardship on the overland trails c. desire to religiously convert American Indians d. interest in establishing law and order in the newly acquired territories

a

"No Negro shall leave the place at any time without my permission...No Negro shall be allowed to marry out of the plantation...No rule I have stated is of more importance than that relating to negroes marrying out of the plantation...It creates a feeling of independence, from being, of right, out of the control of the masters for a time. Never allow any man to talk to your Negroes, nothing more injurious." Rules of Highland Plantation, 1838 Which of the following best supports the excerpt? a. Slave owners had a special status of privilege. b. Many slaves were able to buy their freedom. c. Many slave revolts resulted in freedom for thousands of slaves before the Civil War. d. Slavery was the sole cause for a single crop agricultural system.

a

"Resolved, That woman is man's equal...."Resolved, That woman has too long rested satisfied in the circumscribed limits which corrupt customs... have marked out for her, and that it is time she should move in the enlarged sphere. assigned her."Resolved, That it is the duty of the women of this country to secure to themselves their sacred right to the elective franchise."Resolved,. That, being invested by the Creator with the same capabilities, and the same consciousness of responsibility for their exercise, it is demonstrably the right and duty of woman, equally with man, to promote every righteous cause, by every righteous means." Declaration of Sentiments and Resolutions (Seneca Falls Convention), 1848 The language and themes of the excerpt were most directly inspired by the a. Declaration of Independence b.. Articles of Confederation c. Northwest Ordinance d. United States Constitution

a

"The expansion of the South [from 1800 to 1850] across the Appalachians and the Mississippi River to the fringes of the high plains was one of the great American folk wanderings. Motivated by the longing for fresh and cheap land,... Southerners completed their occupation of a region as large as western Europe. Despite the variety of the land,... the settlers of the Southwest had certain broad similarities. They might be farmers large or small, but most farmed or lived by serving the needs of farmers Not all owned or ever would own slaves, but most accepted slavery as a mode of holding and creating wealth." Albert E. Cowdrey, historian, This Land, This South: An EnvironmentalHistory, 1983 Which of the following contributed most directly to the population movement described in the excerpt? a. The overcultivation of the soil b. The discovery of gold c. The growth of industrial manufacturing d. The decline of the trans-Atlantic trade

a

(could not find image) The image depicting a middle-class family most directly reflects which of the following? a. As nineteenth-century economic transformations increasingly separated home and workplace, the home became idealized as a domestic refuge. b. Some nineteenth-century women whose lives revolved around child care were illiterate. c. Many nineteenth-century men relied on their wives' sewing skills for their clothing needs. d. Children in nineteenth-century folk art were often portrayed as miniature adults.

a

(could not find image) Work in the cotton mills of New England a. provided many women with financial independence for the first time. b. gave women a trade they could take with them when they moved. c. paid less than doing piecework at home. d. allowed women to broaden their search for husbands .

a

By 1844, most of the workers in the mills a. were Irish immigrant women. b. were African-American slaves. c. were young teenage males. d. were young children under the age of 10.

a

The Cult of Domesticity refers to which of the following? a. the ideology that women's role was to preserve morality in the domestic sphere, while men belonged in the public sphere of trade and politics b. the ideology that men and women were to establish a domestic sanctuary by sharing child-rearing and home maintenance roles c. the ideology that female slaves were only to be used as domestic servants and banned from field labor d. the ideology that elevated women as leaders of a domestic crusade against slavery and intemperance

a

The cartoon was probably drawn by a. an artist using a log cabin to symbolize the "common man" b. an opponent of the trend toward more democracy in the United States. c. a critic of the United States military. d. a Democratic Party newspaperman.

a

Which of the following events could best be interpreted as reflecting the exercise of power depicted in the image? a. American Indian removal despite the Supreme Court ruling in Worcester v. Georgia b. The completion of the transcontinental railroad despite concerns about costs c. The publication of the South Carolina Exposition and Protest, which explained nullification d. The formation of the Whig Party

a

*picture of a conductor evicting a black man from a first-class coach* The argument against the Plessy v. Ferguson decision by the Supreme Court in 1896 was based on which of the following? a 14TH Amendment b 17th Amendment c 15th Amendment d 16th Amendment

a 14TH Amendment

"It is a fact well known to every intelligent Southerner that we are compelled to go to the North for almost every article of utility and adornment, from shoepegs and paintings to cotton-mills, steamships and statuary...owing to the absence of a proper system of business amongst us, the North becomes, in one way or another, the proprietor and dispenser of all our floating wealth, and that we are dependent on Northern capitalists...and that, instead of building up...our own States, cities, and towns, we have been spending our substance at the North, and are daily augmenting and strengthening the very power which now has us so completely under its thumb....It is not so much in its moral and religious aspects that we propose to discuss the question of slavery, as in its social and political character and influences." Hinton R. Helper, The Impending Crisis of the South, 1857 Which of the following was most likely the intended audience of the excerpt above? a Agriculturalists of the Southeast and Southwest b African Americans seeking economic refuge in the West c Northern leaders who were proposing economic and political compromise d The emerging middle and working classes

a Agriculturalists of the Southeast and Southwest

*graph of food prices, fuel and lighting prices, and cost of living index lowering from 1870-1899* Which of the following was a direct effect of the trend depicted on American society as a whole? a American standard of living generally increased as real wages increased and cost of food declined. b American standard of living declined as the gap between the rich and the poor vastly increased. c The standard of living increased for all Americans due to better diets and living conditions. d The standard of living declined for some immigrant families as they struggled to compete in an increasingly global marketplace.

a American standard of living generally increased as real wages increased and cost of food declined.

Which of the following would the artist of the political cartoon above be most concerned with? a Business interests were conflicting with the public interest which compromised public officeholders and increased corruption in government. b Although the standard of living for many Americans did increase during this time period, the gap between the rich and the poor was too great. c New business structures allowed for the diversity of the U.S. economy that protected American industry from economic panics. d Business leaders and policymakers were too concerned with consolidated corporations and international trade than the will of the average American.

a Business interests were conflicting with the public interest which compromised public officeholders and increased corruption in government.

Which of the following led to the image depicted in the painting above? a Corporations encouraged increased leisure time due to increased production capacity. b Large Chinese populations carried sporting traditions of boxing with them to America. c Political machines urged factory workers to raise campaign money through gambling. d Growth of the middle class due to an expanding clerical and mangerial staff for large corporations.

a Corporations encouraged increased leisure time due to increased production capacity

"It's been quite a journey this decade, and we held together through some stormy seas. And at the end, together, we're reaching our destination. The fact is, from Grenada to the Washington and Moscow summits, from the recession of '81 to '82, to the expansion that began in late '82 and continues to this day, we've made a difference. The way I see it, there were two great triumphs, two things that I'm proudest of. One is the economic recovery, in which the people of America created—and filled—nineteen million new jobs. The other is the recovery of our morale. America is respected again in the world and looked to for leadership." President Ronald Reagan, Farewell Address to the Nation, January 1989 Which of the following best represents an important aspect of Reagan? 's views about economic growth? a Deregulating industry b Addressing economic inequality c Reducing government defense spending d Increasing manufacturing employment

a Deregulating industry

"On the first of February we intend to begin submarine warfare unrestricted. In spite of this, it is our intention to endeavor to keep neutral the United States of America. "If this attempt is not successful, we propose an alliance on the following basis with Mexico: That we shall make war together and together make peace. We shall give general financial support, and it is understood that Mexico is to reconquer the lost territory in New Mexico, Texas, and Arizona. The details are left to you for settlement." Arthur Zimmermann, German Foreign Minister, January 19, 1917 Which of the following was the most significant effect of the American Expeditionary Forces' impact on WWI? a Entry helped tip the balance in favor of the Allies despite the limited role in trench warfare. b Entry into the war only served to increase anti-British sentiment in the war. c Entry helped to ensure a continued stalemate in Europe. d Entry into the war increased governments protections of freedom of speech.

a Entry helped tip the balance in favor of the Allies despite the limited role in trench warfare.

"The remedy for . . . inefficiency lies in systematic management. . . . The fundamental principles of scientific management are applicable to all kinds of human activities, from our simplest individual acts to the work of our great corporations. . . . At the works of Bethlehem Steel, for example, . . . thousands of stop-watch observations were made to study just how quickly a laborer . . . can push his shovel into the pile of materials and then draw it out properly loaded. . . .With data of this sort before him, . . . the man who is directing shovelers can first teach them the exact methods which should be employed to use their strength to the very best advantage." Frederick Winslow Taylor, The Principles of Scientific Management, 1911 Which of the following groups of people would have been most likely to oppose Taylor? 's management ideas? a Factory workers b White-collar professionals c Tenant farmers d Owners of large businesses

a Factory workers

When the Sun comes back And the first quail calls Follow the Drinking Gourd, For the old man is a-waiting for to carry you to freedom If you follow the Drinking Gourd The riverbank makes a very good road. The dead trees will show you the way. Left foot, peg foot, travelling on, Follow the Drinking Gourd. The river ends between two hills Follow the Drinking Gourd. There's another river on the other side Follow the Drinking Gourd. When the great big river meets the little river Follow the Drinking Gourd. For the old man is a-waiting for to carry to freedom If you follow the Drinking Gourd. --Follow the Drinking Gourd, American folksong Abolitionists' actions and the results in relation to the above song most directly led to which of the following points of contention between the free and slave states? a Increased pressure by the South for the return of fugitive slaves b Controversy over the issuing of the Emancipation Proclamation c Southern states decision to secede from the Union d The passage of the 13th Amendment

a Increased pressure by the South for the return of fugitive slaves

The trend shown in the map led most directly to which of the following? a Increasing divisions between North and South because of questions about the status of slavery in new terriroties b Increasing legal immigration for Asians becuase the United States became a Pacific Rim country c Decreasing tensions between White settlers and Native Americans because expanded United States territory undercut competition d A decreasing gap in wealth becuase land ownership increased among White citizens

a Increasing divisions between North and South because of questions about the status of slavery in new terriroties

"Be it enacted by the Senate and House of Representatives of the United States of America in Congress assembled, That the tract of land in the Territories of Montana and Wyoming, ... is hereby reserved and withdrawn from settlement, occupancy, or sale under the laws of the United States, and dedicated and set apart as a public park or pleasuring-ground for the benefit and enjoyment of the people; and all persons who shall locate or settle upon or occupy the same, or any part thereof, except as hereinafter provided, shall be considered trespassers and removed therefrom. SEC 2. That said public park shall be under the exclusive control of the Secretary of the Interior, whose duty it shall be, as soon as practicable, to make and publish such rules and regulations as he may deem necessary or proper for the care and management of the same. Such regulations shall provide for the preservation, from injury or spoliation, of all timber, mineral deposits, natural curiosities, or wonders within said park, and their retention in their natural condition." --Forty-Second U.S. Congress, Act Establishing Yellowstone National Park, 1872 Which was a more common notion of land conservation in the late 19th century, compared to the general idea reflected in the above legislation? a Land should be protected for careful public management of the natural resources. b Land should be protected the sake of preserving the natural beauty and ecology. c Preservation was a private endeavor, not a public one. d Only development on land limited in water resources should be restricted.

a Land should be protected for careful public management of the natural resources.

Which group was most likely the intended audience of the cartoon above? a Members of the Republican Party b Voluntary organizations promoting religious and secular reforms c State governments in the North d White Americans in the South

a Members of the Republican Party

"Is a tractor bad? Is the power that turns the long furrows wrong? If this tractor were ours, it would be good? —not mine, but ours. . . .We could love that tractor then as we have loved this land when it was ours. But this tractor does two things—it turns the land and turns us off the land. There is little difference between this tractor and a tank. The people are driven, intimidated, hurt by both." John Steinbeck, novelist, The Grapes of Wrath, 1939 Which of the following movements expressed ideas most similar to the ideas expressed in the excerpt? a Populism in the 1890s and early 1900s b The counterculture of the 1960s and 1970s c Nativism in the 1840s and 1850s d Abolitionism in the 1830s and 1840s

a Populism in the 1890s and early 1900s

"The most stringent protection of free speech would not protect a man in falsely shouting fire in a theater and causing a panic.... The question in every case is whether the words used are used in such circumstances and are of such a nature as to create a clear and present danger that they will bring about the substantive evils that Congress has a right to prevent. It is a question of proximity and degree. When a nation is at war, many things that might be said in time of peace are such a hindrance to its effort that their utterance will not be endured so long as men fight, and that no Court could regard them as protected by any constitutional right." - Majority opinion of the United States Supreme Court in Schenck v. United States, 1919 The restrictions imposed by the Schenck decision most directly contradicted which of the following earlier developments in the United States? a Protection of liberties through the adoption of the Bill of Rights in 1791 b Arguments for self-government asserted in the Declaration of Independence c Assertion of federal power over states' rights in the 1819 McCulloch v.Maryland decision d Expansion of voting rights during President Andrew Jackson's

a Protection of liberties through the adoption of the Bill of Rights in 1791 (i think)

"Be it enacted by the Senate and House of Representatives of the United States of America in Congress assembled, That the tract of land in the Territories of Montana and Wyoming, ... is hereby reserved and withdrawn from settlement, occupancy, or sale under the laws of the United States, and dedicated and set apart as a public park or pleasuring-ground for the benefit and enjoyment of the people; and all persons who shall locate or settle upon or occupy the same, or any part thereof, except as hereinafter provided, shall be considered trespassers and removed therefrom. SEC 2. That said public park shall be under the exclusive control of the Secretary of the Interior, whose duty it shall be, as soon as practicable, to make and publish such rules and regulations as he may deem necessary or proper for the care and management of the same. Such regulations shall provide for the preservation, from injury or spoliation, of all timber, mineral deposits, natural curiosities, or wonders within said park, and their retention in their natural condition." --Forty-Second U.S. Congress, Act Establishing Yellowstone National Park, 1872 Which group most expressed similar concerns as the preservationists in the 19th century over the impact of capitalist enterprises on the natural environment? a Romantics who believed in human perfectability b Protestant evangelicals c Those who promoted science d Southern romantic aristocrats

a Romantics who believed in human perfectability

"The establishment of schools for the instruction of the young is gradually being extended among the Indian tribes under our control. ... One of the most important points is that they should be taught to speak and read the English language. ... [T]he teaching of the English language must be considered infinitely more useful. If Indian children are to be civilized, they must learn the language of civilization. they will become far more accessible to civilized ideas and ways of thinking when they are enabled to receive those ideas and ways of thinking through the most direct channel of expression. At first, their minds should not be overburdened with too great a multitude of subjects of instruction, but turned to those practical accomplishments, proficiency in which is necessary to render civilized life possible. In addition to the most elementary schooling boys should receive a good training in household duties and habits of cleanliness. In this way a young generation may be raised up far more open to civilizing influences of a high kind and more fit for a peaceable and profitable intercourse with the white people." Excerpt from the Secretary of the Interior to President Hayes, 1877 The ideas expressed in the excerpt above reflect which of the following effects regarding federal policy toward native Americans during the 1840s and 1850s? a Substantive changes to native culture and great dependence on the U.S. government for economic survival b Reform in the Bureau of Indian Affairs to allow for homeschooling of Indian children c Staunch advocacy of an nativist movement to force relocation outside the U.S. sovereign territory d Isolation on large reservations that furthered a native cultural and religious revival

a Substantive changes to native culture and great dependence on the U.S. government for economic survival

"There was never the least attention paid to what was cut up for sausage; there would come all the way back from Europe old sausage that had been rejected, and that was moldy and white--it would be dosed with borax and glycerine, and dumped into the hoppers, and made over again for home consumption. There would be meat that had tumbled out on the floor, in the dirt and sawdust, where the workers had tramped and spit uncounted billions of consumption germs. There would be meat stored in great piles in rooms; and the water from leaky roofs would drip over it, and thousands of rats would race about on it. It was too dark in these storage places to see well, but a man could run his hand over these piles of meat and sweep off handfuls of the dried dung of rats. These rats were nuisances, and the packers would put poisoned bread out for them; they would die, and then rats, bread, and meat would go into the hoppers together." The Jungle, Upton Sinclair 1906 The creation of which federal agency below was most directly inspired by Sinclair's work in The Jungle? a The Food and Drug Administration b The Bureau of Health and Human Services c The Environmental Protection Agency d The Department of Justice

a The Food and Drug Administration

*graph showing the unemployment rates of nonfarmworkers* The goals of the reform measures attempted in the 1930's were most similar to the goals of which the following reform efforts? a The Populists in the 1890s b The Women's Rights Movement in the 1840's c The Temperance Movement in the early 1900's d The abolitionists in the 1830's

a The Populists in the 1890s (i think)

"We have conquered many of the neighboring tribes of Indians, but we have never thought of holding them in subjection—never of incorporating them into our Union To incorporate Mexico, would be the very first instance of the kind of incorporating an Indian race; for more than half of the Mexicans are Indians, and the other is composed chiefly of mixed tribes.... Ours, sir, is the Government of a white race. ... [I]t is professed and talked about to erect these Mexicans into a Territorial Government, and place them on an equality with the people of the United States. I protest utterly against such a project." Senator John C. Calhoun, "Conquest of Mexico" speech, 1848 Which of the following events best represents a continuity of the sentiments expressed by Senator Calhoun in the speech? a The Supreme Court decision in Plessy v. Ferguson b The Great Migration of African Americans out of the South in the 1910s c Support for assimilationist policies in the 1880s and 1890s d The United States rejection of membership in the League of Nations

a The Supreme Court decision in Plessy v. Ferguson

*cartoon of old japanese man saying "patience young man... they get over it eventually", to which a US Muslim says "who are you", to which the man says "wwII interment camp survivor* Which historical episode most closely parallels the situation illustrated in the cartoon above? a The fear resulting in the post-World War I Red Scare b The embrace of the counterculture movement by many young Americans in the 1960s c The often violent nativist reaction to international migrants in the antebellum era d Federal policies toward Mexican migrants in the 1930s and 1940s

a The fear resulting in the post-World War I Red Scare

The People's (Populist) Party emerged most directly in response to which of the following late-nineteenth-century trends? a The growth of corporate power in agriculture and the economy b The efforts of businesses to gain control over markets and resources abroad c The development of political machines d The influx of migrants from Asia and southern and eastern Europe

a The growth of corporate power in agriculture and the economy

"Give us a business man [to] introduce business methods into politics and government"... yet politics remains corrupt... The commercial spirit is the spirit of profit, not patriotism; of credit, not honor; of individual gain, not national prosperity; of trade and dickering, not principle... "Business is business" is not a political sentiment, but our politician has caught it. He takes essentially the same view of the bribe... But there is hope...in the commercialism of our politics. If our political leaders are to be always a lot of political merchants, they will supply any demand we may create. All we have to do is to establish a steady demand for good government." Lincoln Steffens, The Shame of the Cities, 1904 The passage above is most clearly an example of which of the following trends? a The rise of journalists cooperating with reformers to call for change b The expansion of the democratic process c The consolidation of corporate power into trusts d The growth of nativist and discriminatory political practices

a The rise of journalists cooperating with reformers to call for change

"So many people ask me what they shall do; so few tell me what they can do.Yet this is the pivot wherein all must turn. "I believe that each of us who has his place to make should go where men are wanted, and where employment is not bestowed as alms. Of course, I say to all who are in want of work, GoWest! . . . "On the whole I say, stay where you are; do as well as you can; and devote every spare hour to making yourself familiar with the conditions and dexterity required for the efficient conservation of out-door industry in a new country. Having mastered these, gather up your family and GoWest!" Horace Greeley, editor of the New York Tribune, letter to R. L. Sanderson, 1871 Which of the following late-nineteenth-century federal actions most directly supported the ideas expressed in the excerpt? a The sale of land to settlers at low cost b The purchase of silver by the United States Treasury c The passage of antitrust legislation d The exclusion of immigrants from Asia

a The sale of land to settlers at low cost

"Soldiers, sailors, and airmen of the Allied Expeditionary Force! You are about to embark upon the Great Crusade, toward which we have striven these many months. The eyes of the world are upon you. The hopes and prayers of liberty-loving people everywhere march with you. In company with our brave Allies and brothers-in-arms on other Fronts, you will bring about the destruction of the German war machine, the elimination of Nazi tyranny over the oppressed peoples of Europe, and security for ourselves in a free world. Your task will not be an easy one. Your enemy is well trained, well equipped and battle hardened. He will fight savagely. But this is the year 1944! Much has happened since the Nazi triumphs of 1940-41. The United Nations have inflicted upon the Germans great defeats, in open battle, man-to-man. Our air offensive has seriously reduced their strength in the air and their capacity to wage war on the ground. Our Home Fronts have given us an overwhelming superiority in weapons and munitions of war, and placed at our disposal great reserves of trained fighting men. The tide has turned! The free men of the world are marching together to Victory! I have full confidence in your courage and devotion to duty and skill in battle. We will accept nothing less than full Victory! Good luck! And let us beseech the blessing of Almighty God upon this great and noble undertaking. SIGNED: Dwight D. Eisenhower General Eisenhower stated that "Our Home Fronts have given us an overwhelming superiority in weapons and munitions of war, and placed at our disposal great reserves of trained fighting men." Which of the following events did not contribute to the war effort on the homefront? a Zoot Suit Riots b Gasoline rationing c Purchase of War Bonds d Selective Training and Service Act

a Zoot Suit Riots

"Americans faced an overwhelming task after the Civil War and emancipation: how to understand the tangled relationship between two profound ideas—healing and justice. . . . [T]hese two aims never developed in historical balance. One might conclude that this imbalance between outcomes of sectional healing and racial justice was simply America's inevitable historical condition. . . . But theories of inevitability . . . are rarely satisfying. . . . The sectional reunion after so horrible a civil war was a political triumph by the late nineteenth century, but it could not have been achieved without the resubjugation of many of those people whom the war had freed from centuries of bondage. This is the tragedy lingering on the margins and infesting the heart of American history from Appomattox to World War I." David W. Blight, historian, Race and Reunion: The Civil War in American Memory, 2001 One key change immediately following the Civil War aimed at achieving the "racial justice" that Blight describes was the a establishment of a constitutional basis for citizenship and voting rights b creation of new agencies to ensure racial integration in employment c campaign by the federal government to eliminate poverty d desegregation of the United States armed forces

a establishment of a constitutional basis for citizenship and voting rights

"Excepting only Yosemite, Hetch Hetchy is the most attractive and wonderful valley within the bounds of the great Yosemite National Park and the best of all the camp grounds. People are now flocking to it in ever-increasing numbers for health and recreation of body and mind. Though the walls are less sublime in height than those of Yosemite, its groves, gardens, and broad, spacious meadows are more beautiful and picturesque. . . . Last year in October I visited the valley with Mr. William Keith, the artist. He wandered about from view to view, enchanted, made thirty-eight sketches, and enthusiastically declared that in varied picturesque beauty Hetch Hetchy greatly surpassed Yosemite. It is one of God's best gifts, and ought to be faithfully guarded." John Muir, Century Magazine, 1909 Muir's ideas are most directly a reaction to the a increasing usage and exploitation of western landscapes b increase in urban populations, including immigrant workers attracted by a growing industrial economy c westward migration of groups seeking religious refuge d opening of a new frontier in recently annexed territory

a increasing usage and exploitation of western landscapes

"All through the night I heard people getting up, dragging cots around. I stared at our little window, unable to sleep. I was glad Mother had put up a makeshift curtain on the window for I noticed a powerful beam of light sweeping across it every few seconds. The lights came from high towers placed around the camp....I remembered the wire fence encircling us, and a knot of anger tightened in my breast. What was I doing behind a fence like a criminal? Of one thing I was sure. The wire fence was real. I no longer had the right to walk out of it. It was because I had Japanese ancestors. It was also because some people had little faith in the ideas and ideals of democracy...." Monica Itoi Stone, Nisei Daughter, 1953 Monica Itoi Sone, Nisei Daughter (Boston: Little, Brown & Co., 1953), 176-178. The experience described in the excerpt above was an example of a internment. b opportunities for women. c isolationism. d segregation.

a internment.

The greatest priority of western railroad development as illustrated above was to a open new markets. b consolidate into larger trusts. c promote government power in the region. d remove American Indians from western lands.

a open new markets.

"The President of the United States...hereby is authorized, whenever in his opinion any reservation or any part thereof...is advantageous for agricultural and grazing purposes...to allot the lands in said reservation in severalty to any Indian located thereon in quantities as follows: To each head of family, one-quarter of a section; To each single person over eighteen years of age, one eighth of a section; To each single orphan child under eighteen years of age, one eighth of a section... Every Indian born within the territorial limits of the United States to whom allotments shall have been made...who has voluntarily taken up, within said limits his residence separate and apart from any tribe of Indians therein, and has adopted that habits of civilized life, is hereby declared a citizen of the United States." The Dawes Severalty Act, 1887 During the late 19th century, western Native American life was most affected by a post-Civil War migrations of whites. b evangelical missions. c alliances among Indian nations. d generous treaties.

a post-Civil War migrations of whites.

Which of the following contextual factors most directly contributed to the decline in confidence in the federal government in the 1970s? A A combination of economic challenges, political scandals, and foreign policy crises B A belief that the government no longer served the interests of the people C A growing clash between conservatives and liberals over policy D A sharp increase in immigration to the United States

A A combination of economic challenges, political scandals, and foreign policy crises

"What is important is the claim of the Communists themselves that for every party member there are 10 others ready, willing, and able to do the party's work. Herein lies the greatest menace of communism. . . . ". . . It might be of interest to observe that in 1917 when the Communists overthrew the Russian Government there was one Communist for every 2,277 persons in Russia. In the United States today there is one Communist for every 1,814 persons in the country. "One who accepts the aims, principles, and program of the party, who attends meetings, who reads the party press and literature, who pays dues, and who is active on behalf of the party 'shall be considered a member.' The open, avowed Communist who carries a card and pays dues is no different from a security standpoint than the person who does the party's work but pays no dues, carries no card, and is not on the party rolls. In fact, the latter is a greater menace because of his opportunity to work in stealth." J. Edgar Hoover, director of the Federal Bureau of Investigation (FBI), testimony before the House Un-American Activities Committee (HUAC), 1947 Hoover's testimony in the excerpt could best be used as evidence by historians studying which of the following? A Debates among Americans about the policies and methods used to expose domestic communists B The methods used by the federal government to infiltrate communist organizations C The efforts by Congress to protect the United States against communist influence D Debates among Americans about how to combat Soviet influence in the developing world

A Debates among Americans about the policies and methods used to expose domestic communists

Which of the following best represents the views held by opponents to the women's rights movement in the second half of the twentieth century? A The original intent of the framers of the Constitution did not include citizenship for women. B Feminism threatens the traditional family unit, which is at the core of the social order. C The violent tactics used by members of the feminist movement illustrates the danger they pose to society. D Feminism makes the United States more vulnerable to communist attack.

B Feminism threatens the traditional family unit, which is at the core of the social order.

Which of the following best explains the change in the overall United States military presence in Vietnam between 1964 and 1968 as depicted in the graph? A The belief that democratic governments needed to be protected from the influence of the Soviet Union B The fear that the North Vietnamese forces under Ho Chi Minh would spread communism in Asia C The concern that France was attempting to colonize Vietnam in opposition to international agreements D The reaction of the Lyndon Johnson administration to scandals over presidential authority

B The fear that the North Vietnamese forces under Ho Chi Minh would spread communism in Asia

Which of the following best describes the federal government's response to the overall patterns depicted in the graph? A The construction of highways meant to facilitate the transportation of fuel B The passage of laws meant to combat environmental problems caused by power generation C The enactment of policies meant to eliminate certain sources of power D The commitment of resources to the expansion of nuclear energy production

B The passage of laws meant to combat environmental problems caused by power generation

The American Indian civil rights movement differed from other civil rights movements at the time because it A debated whether to use violent or nonviolent protest tactics B sought to gain compensation for past government land policies C focused on resisting the military draft during the Vietnam War D supported the effort to reduce pollution produced by power plants

B sought to gain compensation for past government land policies

"[The United States federal government in] Washington had a mixed response to Asian decolonization. On the one hand, it was not unhappy to see the European empires dissolved. Washington regarded these empires, which functioned as restricted trading blocs, as obstacles to economic integration and as incubators of communism and anti-Western revolution. On the other hand, Washington recognized that Europe's economic and political stability often depended upon income generated in the colonies. Whether the United States supported or opposed a particular nationalist movement often depended on its relationship to communism. . . . Washington only endorsed nationalist movements, such as those in Indonesia and the Philippines, that promised to preserve Western access after independence. It was willing to abolish formal empire, as long as the relations of informal empire continued uninterrupted." Christina Klein, Cold War Orientalism: Asia in the Middlebrow Imagination, 1945-1961, published in 2003 "Shortly after the outbreak of war between the Vietnamese and the French, Ho Chi Minh's Democratic Republic of [North] Vietnam (DRV) launched a four-month diplomatic initiative in the spring and summer of 1947 designed to secure the support of the [President Harry] Truman administration. . . . [The DRV's] agenda included calls for recognition of the DRV and mediation of the war with the French, requests for rehabilitation loans and promises of economic concessions to U.S. businesses, and appeals for technical assistance and cultural exchange. . . . ". . . With Soviet diplomacy focused on Europe and the Chinese communists preoccupied by civil war, the DRV also faced almost complete isolation from the communist world. . . . [But United States] fears of Vietnamese subservience to Moscow that first had emerged in 1946 intensified with the escalation of Soviet-American tensions in Europe. . . . The commitment of the United States to maintain French political and economic stability in Western Europe complicated its abilities to challenge French policies in Vietnam directly." Mark Philip Bradley, Imagining Vietnam and America: The Making of Post-Colonial Vietnam, 1919-1950, published in 2000 The excerpts could best be used to support which of the following criticisms about United States foreign policy during the Cold War? A The United States took advantage of decolonization to gain more influence in a weakened Europe. B The United States preferred to focus on domestic issues rather than on international concerns. C The United States sometimes supported nondemocratic countries so long as they opposed communism. D The United States devoted too much money and effort to fighting communism in newly independent countries.

C The United States sometimes supported nondemocratic countries so long as they opposed communism.

The overall trend depicted in the graph best reflects which of the following? A The expansion of renewable energy sources to address environmental concerns in the 1970s B The decline of coal as a major source of power beginning in the 1950s C The rejection of nuclear power following the end of the Vietnam War D The increased demand for all sources of power following the Second World War

D The increased demand for all sources of power following the Second World War

*i can't find picture, picture of map with US territory after Mexican War Which of the following ideas contributed most directly to the territorial changes shown in the map? a Containment b Abolitionism c Manifest Destiny d Popular sovereignty

Manifest Destiny

What issue became prominent following the war with Mexico? a Whether or not "blood had been spilt on American soil." b The financial cost of the war. c The human cost of the war. d Whether or not California should be admitted to the union as a slave or free state.

Whether or not California should be admitted to the union as a slave or free state.

"Resolved, That woman is man's equal...."Resolved, That woman has too long rested satisfied in the circumscribed limits which corrupt customs... have marked out for her, and that it is time she should move in the enlarged sphere. assigned her."Resolved, That it is the duty of the women of this country to secure to themselves their sacred right to the elective franchise."Resolved,. That, being invested by the Creator with the same capabilities, and the same consciousness of responsibility for their exercise, it is demonstrably the right and duty of woman, equally with man, to promote every righteous cause, by every righteous means." Declaration of Sentiments and Resolutions (Seneca Falls Convention), 1848Which other "righteous cause" would participants in the Seneca Falls Convention have been most likely to support? a. Abolitionism b. Expansionism c. Conservationism d. Nativism

a

"The system of quotas . . . was the first major pillar of the Immigration Act of 1924. The second provided for the exclusion of persons ineligible to citizenship. . . . Ineligibility to citizenship and exclusion applied to the peoples of all the nations of East and South Asia. Nearly all Asians had already been excluded from immigration. . . . The exclusion of persons ineligible to citizenship in 1924 . . . completed Asiatic exclusion. . . . Moreover, it codified the principle of racial exclusion into the main body of American immigration and naturalization law." - Mae M. Ngai, historian, Impossible Subjects: Illegal Aliens and the Making of Modern America, 2004 Which of the following evidence would best support Ngai's argument in the excerpt? a Census data showing the changing percentages of the foreign-born population from 1920 to 1930 b Narratives describing the challenges of immigrant family life in the 1920s c Diplomatic correspondence reflecting the increasing isolationism of United States foreign policy in the 1920s and 1930s d Census data revealing the Great Migration of African Americans to cities in the North and West in the 1920s

a Census data showing the changing percentages of the foreign-born population from 1920 to 1930

Which of the following groups faced the most difficult economic conditions during the 1920s? a Farmers and many rural areas b Businesses that did not accept credit cards c Non-unionized workers in older factories d Assembly line workers in factories

a Farmers and many rural areas

*picture of "Mexicans entering the United States, United States immigration station, El Paso, Texas, 1938"* What might have been a typical experience of the individuals in the above photo in the years soon after their immigration? a Increased employment opportunities due to wartime labor shortages b Opportunity only for agricultural work c High levels of unemployment for their social group, despite job opportunities arising from WWII d Availability of work for them being restricted solely to menial jobs

a Increased employment opportunities due to wartime labor shortages

Which of the following groups would be most likely to support the perspective of the cartoon? a Radical Republicans b Veterans of the Confederate Army c Southern politicians d Northern opponents of the war

a Radical Republicans

*picture of poster telling public that "[their] duty- buy US government bonds. 2nd liberty loan of 1917"* In 1917, President Wilson brought the United States into World War I based on his stated intention to a defend humanitarian and democratic principles. b expand America's military and economic presence in Europe. c spread American culture and norms to others. d pursue a unilateral foreign policy.

a defend humanitarian and democratic principles.

"We have as yet no certaine proofe or experience concerning the vertues of... Corne, although the... Indians... are constrained to make a virtue of necessitie, and think it a good food: whereas we mayeasily judge that it nourisheth but little, and is of a hard... digestion, a more convenient food for swine than for men." John Gerard, English botanist, The Herball or Generall Historie of Plantes, 1597 Gerard's description of "corne" in the excerpt best reflects which of the following? a. Assumptions about the superiority of European culture b. Inability to find uses for North American crops c. Concerns about economic threats posed by North American imports d. Curiosity about American Indian ways of life

a. Assumptions about the superiority of European culture

*Picture of Boston Massacre, showing the British shooting colonists dead* Which of the following groups would most likely have supported the point of view expressed in the image above? a. The Sons and Daughters of Liberty b. British Soldiers c. Tories d. John Adams

a. The Sons and Daughters of Liberty

*pie chart showing a large portion of US's spending in 1954 was going to Defense, and then remainder, totaling 111 billion* This chart would be used to best support what idea from the time period? a. The existence of a military-industrial complex b. The containment of communism was not a US priority c. The efforts to limit communist espionage was necessary and just d. The de-emphasis of the importance of American schooling in context of the Cold War

a. The existence of a military-industrial complex ( i think)

*pie chart showing a large portion of US's spending in 1954 was going to Defense, and then remainder, totaling 111 billion* The data in the chart serves as evidence of which of the following occurring in the United States at the time? a. US action to contain communism in all parts of the world b. US actions to reduce its budget deficits from World War II c. US action to expand overseas colonies, particularly in Latin America and the Pacific d. US action to limit federal spending in education and domestic programs

a. US action to contain communism in all parts of the world

"The members of this congress, sincerely devoted, with the warmest semtiments of affection and duty to His Majesty's person and government. 2nd. That Hist Majesty's liege subjects in these colonies are entitled to all the inherent rights and privileges of hist natural born subjects within the kindom of Great Britain. 3rd. That it is inseparable essential to the freedom of a people, and the undoubted rights of Englishmen, that no taxes should be imposed on them, but with their own consent, given personally, or by their representatives. 5th. That the only representatives of the people of these colonies are persons chosen therein, by themselves; and that no taxes ever have been or can be constitutionally imposed on them but by their respective legislatures." - Stamp Act Congress, 1765 The passage above most directly reveals the impact on colonial thinking of ideas based on a. a republican form of government as celebrated by Enlightenment philosophers. b. religious tolerance as emphasized by the earliest Founders of the American colonies. c. individual liberty as expressed by the colonies' minority inhabitants. d. economic equality as propsed by freedmen and laborers.

a. a republican form of government as celebrated by Enlightenment philosophers.

"What began as a protest movement is being challenged to translate itself into a political movement. It is now concerned not merely with removing the barriers to full opportunity but with achieving the fact of equality. From sit-ins and freedom rides we have gone into rent strikes, boycotts, community organization, and political action. As a consequence of this natural evolution, the Negro today finds himself stymied by obstacles of far greater magnitude than the legal barriers he was attacking before: automation, urban decay, de facto school segregation." — Bayard Rustin, "From Protest to Politics," 1965 The activism described in the excerpt most directly helped inspire renewed social and political activism by a. environmentalists b. American Indians c. labor unions d. military veterans

a. environmentalists b. American Indians

"The town is now quite filled with inhabitants, which in regard to their country, religion, and trade, are very different from each other...Every one who acknowledges God to be the Creator...is at liberty to settle, stay, and carry on his trade here, be his religious principles ever so strange. No one is here molested on account of the erroneous principles of the doctrine which he follows...On a careful consideration of what I have already said, it will be easy to conceive how this city should rise so suddenly from nothing into such grandeur and perfection...It has not been necessary to force people to come and settle here; on the contrary, foreigners of different languages have left their country, houses, property, and relations, and ventured over wide and stormy seas, in order to come hither." Peter Kalm, A Swedish Visitor Tells About Philadelphia, 1748 The prosperity and populousness of Philadelphia described by Peter Kalm best reflect the influence of which of the following? a. freedom of religion b. racial hierarchy c. royal authority d. religious orthodoxy

a. freedom of religion

*Keep within compass picture* Women's status during the late 1700s changed as they a. gained new standing in American political culture. b. developed cultures reflecting their interests and experiences. c. became involved in various reform efforts outside the home. d. began to earn the right to vote in some places.

a. gained new standing in American political culture.

The "women's lib" movement is not an honest effort to secure better jobs for women whowant or need to work outside the home. This is just the superficial sweet-talk to win broad support for a radical "movement." Women's lib is a total assault on the role of the American woman as wife and mother, and on the family as the basic unit of society. Women's libbers are trying to make wives and mothers unhappy with their career, make them feel that they are "second-class citizens" and "abject slaves." Women's libbers are promoting free sex instead of the "slavery" of marriage. They are promoting Federal "day-care centers" for babies instead of homes. They are promoting abortions instead of families. - Phyllis Schlafly, What's Wrong with 'Equal Rights' for Women?, February 1972 While Schlafly battled ratification of the Equal Rights Amendment during the 1970s, which organization actively promoted its ratification? a. Organization of Women's Freedom b. National Organization for Women c. Homemakers' Equal Rights Association d. Concerned Women for America

b. National Organization for Women

*Picture of graph displaying native population declining* Which of the following contributed most to the changes shown in the graph? a. Loss of territory and access to natural resources b. The introduction of new diseases c. Warfare with Europeans d. The introduction of new plants and animals

b. The introduction of new diseases

*Picture of Hamilton's Financial Plan* The above graphic best reflects which of the following groups' economic goals for the newly formed United States? a. Those who argued against a national bank. b. Those who argued for a centralized national economic policy. c. Those who argued that the new Constitution should require a balanced federal budget. d. Those who wanted to abolish all internal taxation.

b. Those who argued for a centralized national economic policy.

"Joseph Smith... came from nowhere. Reared in a poor Yankee farm family, he had less than two years of formal schooling and began life without social standing or institutional backing. His family rarely attended church. Yet in the fourteen years he headed the Church of Jesus Christ of Latter-day Saints, Smith created a religious culture that survived his death, flourished in the most desolate regions of the United States, and continues to grow worldwide. . . . In 1830 at the age of twenty-four, he published the Book of Mormon. . . .He built cities and temples and gathered thousands of followers before he was killed at age thirty-eight." Richard Lyman Bushman, historian, Joseph Smith Rough Stone Rolling: A Cultural Biography of Mormonism's Founder, 2005 The goals of the Mormons, as described in the excerpt, were most like the goals of which of the following colonial groups? a. Planters in the Chesapeake region b. French missionaries in the Great Lakes region c. Puritans in New England d. Spanish settlers in California

c

(could not find image) Map of Slave Populations in 1820 and 1860 What contributed most to the process illustrated in the maps above? a. The Missouri Compromise of 1820 b. The rise in the number of free African Americans in the South c. The overcultivation of arable land in the Southeast d. The outlawing of the international slave trade

c

Which of the following was the main reason for the Louisiana Purchase? a. The U.S. wanted to dominate the North American continent b. To build canals and roads c. The acquisition of new lands for farming and markets d. The extension of slavery

c

*picture of woman protesting for "self government, sign that says Kaiser Wilson" The call for self-government called for on the banner pictured above was accomplished in the United States as a the Supreme Court ruled that denying women the right to vote was unconstitutional in 1920 b One state at a time slowly gave women the right to vote in the early 20th century. c A constitutional amendment prohibited any state to deny the right to vote on account of sex. d Congress passed legislation granting women the right to vote in federal elections in 1935.

c A constitutional amendment prohibited any state to deny the right to vote on account of sex.

*lucky star cigarette ad* In response to the increased consumption due to advertising and mass production such as those illustrated above, episodes of credit and market instability such as the Great Depression increased. Which of the following was the response by the federal government? a Decreased oversight of the stock market by the Federal Reserve Board (the Fed). b Continued policy of laissez-faire. c Creation of Glass-Steagall Act and the Federal Deposit Insurance Corporation. d Increased tariffs such as the Ford-McCumber Tariff.

c Creation of Glass-Steagall Act and the Federal Deposit Insurance Corporation.

*picture similar to this* Which area of the United States offered the most amount of new cultural opportunities such as that depicted above? a California and western mountain towns b Rural plains c Large east coast cities d Southern cities such as Atlanta

c Large east coast cities

Which of the following early 20th-century cultural conflicts most directly contradicted the scene portrayed in the image above? a Fundamentalism versus modernism b Idealism versus disillusionment c Native-born versus new immigrants d Urban versus rural

c Native-born versus new immigrants

*Can't find chart, but it shows the economic status of both the North and the South during the Civil War* Which of the following was a direct effect of the trend shown in the chart? a The South had significant financial advantage relative to the North b the North was able to use its additional resources to overwhelm Confederate forces at the start of the Civil War c The North benefited from an industrial economy that could produce needed manufactured goods for the war effort d The South was able to effectively leverage its reliance on cotton as a cash crop

c The North benefited from an industrial economy that could produce needed manufactured goods for the war effort

The image most strongly supports the argument that Reconstruction a led to the unfair punishment of White Southerners by the North b involved unconstitutional abuses of government power c temporarily altered race relations in the South d encouraged large-scale rebellions by former slaves

c temporarily altered race relations in the South

A typical sentiment of a nativist living in a U.S. northeastern city in the mid- to late- 1800s would most likely be a that Protestants should take literacy tests before being allowed to vote. b that German immigrants were spreading slums and selling their votes to Whigs. c that Irish immigrants were stealing jobs and were selling their votes to Democrats. d that Catholics should reserve new jobs for fellow Catholics.

c that Irish immigrants were stealing jobs and were selling their votes to Democrats.

The advertisement for Social Security most directly reflects which of the following developments during the New Deal? a the unpopularity of Francis Townsend's program b the need to provide individuals with money for consumer goods c the idea that government could provide citizens with some aid to deal with life's vicissitudes d the continuing decline in most Americans' standard of living despite the plethora of New Deal agencies

c the idea that government could provide citizens with some aid to deal with life's vicissitudes

*Picture displaying a town (Sudsbery, Massachussetts) where a meeting house is surrounded by homes and farms* What Puritan goal is best represented by the town structure shown in the map? a. To allow an ethnically and religiously diverse community to develop b. To allow division of property based on the English system of primogeniture c. To create a community of like minded religious believers centered around a church meeting house d. To encourage expansion from the community center as it's population increased

c. To create a community of like minded religious believers centered around a church meeting house

"Young ladies in town, and those that live round, Let a friend at this season advise you: Since money's so scarce, and times growing worse, Strange things may soon hap and surprise you; First then, throw aside your high top knots of pride, Wear none but your own country linen, Of Economy boast, let your pride be the most To show clothes of your own make and spinning." The Daughters of Liberty Urge Americans to Boycott British Goods The excerpt most clearly demonstrates that colonial women participated in the struggle against unfair British measures as a. spies gathering information about the Loyalists in their areas. b. rioters protesting in the streets against royal officials. c. consumers refusing to purchase imported British goods. d. laundresses accompanying the colonial troops fighting the British.

c. consumers refusing to purchase imported British goods.

"[T]he condition of the African race throughout all the States where the ancient relation between the two [races] has been retained enjoys a degree of health and comfort which may well compare with that of the laboring population of any country in Christendom; and, it may be added that in no other condition, or in any other age or country, has the Negro race ever attained so high an elevation in morals, intelligence, or civilization." - John C. Calhoun, political leader, 1844 In the 1840s and 1850s, the views expressed by Calhoun most directly contributed to a. the United States acquisition of new territory in the West b. the development of sharecropping and tenant farming in the South c. the rise of voluntary organizations to promote religious reform d. increased sectional divisions between the North and the South

d

(could not find image) We Owe Allegiance to No Crown, John Woodside, c. 1814 The painting above is best understood in the context of a. U.S. dominance over the North American continent. b. federal efforts to assert authority over the states. c. increased migration from Europe to the United States. d. the emergence of a new national culture.

d

"Various are the reports and conjectures of the causes of the present Indian war. Some impute it to an imprudent zeal in the magistrates of Boston to christianize those heathen before they were civilized and enjoining them the strict observation of their laws... Some believe there have been vagrant and Jesuitical priests, who have made it their business, for some years past, to go from Sachem to Sachem, to exasperate the Indians against the English and to bring them into a confederacy, and that they were promised supplies from France and other parts to extirpate [eradicate] the English nation out of the continent of America." Edward Randolph, report of King Philip's War (Metacom's War) in New England, 1676 Which of the following best characterizes relations between the English and American Indians in New England following Metacom's War? a. Recognition of American Indian property and land rights b. Peaceful accommodation between both groups c. Religious freedom for the English and American Indians d. Dramatic decline and dispersion of the American Indian population

d. Dramatic decline and dispersion of the American Indian population

"On the subject of slavery . . . I will be as harsh as truth, and as uncompromising as justice. . . . On this subject, I do not wish to think, or speak, or write, with moderation. . . . I am in earnest—I will not equivocate—I will not excuse—I will not retreat a single inch—AND I WILL BE HEARD.' — William Lloyd Garrison, first issue of abolitionist newspaper The Liberator, January 1831 The type of views expressed in the excerpt emerged most directly from which of the following trends? a. Growing nativism b. Opposition to industrialization c. The Second Great Awakening d. Support for the idea of Manifest Destiny

c

*picture of "Mexicans entering the United States, United States immigration station, El Paso, Texas, 1938"* Which common experience did Mexican Americans share with other "non-white" groups during the Great Depression? a A sharp increase in employment as domestic servants, due to lack of industrial jobs b Mass deportations by the federal government c Loss of jobs to white Americans d Migration from large urban areas to the countryside for jobs

c Loss of jobs to white Americans

Which of the following is an example of FDR touting America as being an "Arsenal of Democracy" a The Tennessee Valley authority b The Civilian Conservation Corps c The Lend-Lease Program d The Works Progress Administration

c The Lend-Lease Program

*nine blind men and a tiger cartoon* Which of the following developments is the most direct effect of the situation portrayed in the image? a. Increased public opposition to the use of nuclear weapons b. Reduced congressional oversight of United States military interventions c. Reduced public trust in the federal government and the President d. Expanded use of military force to achieve foreign policy goals in Eastern Europe

c. Reduced public trust in the federal government and the President

*Picture of Hamilton's Financial Plan* Which of the following was a significant political development which can be most attributed to the enactment of the full financial program partly represented in the above graphic? a. Speculators in U.S. bonds lost money. b. The Constitution was amended to allow for political parties. c. Formal political parties organized in opposition emerged. d. The Supreme Court's power of judicial review was instituted.

c. Formal political parties organized in opposition emerged.

*Picture of New England town layout* How was the general community structure in the southern English colonies different from that of the Puritan communities? a. Large plantations shared by various family owners vs. individual farms b. Large metropolitan areas vs. towns c. Isolated plantations vs. town communities d. Plantations physically connected to cities vs. small farm plots within town communities

c. Isolated plantations vs. town communities

*Picture of Hamilton's Financial Plan* Which of the following best describes the reasoning behind the financial construct adopted by the early U.S. government shown in the above graphic? a. Congress did not consider long-term economic investment to be in the nation's best interests. b. Early Republican lawmakers were reluctant to enact any kind of direct or indirect taxes. c. States were eager to turn over the management of their affairs to the central federal government. d. The wealthy lenders who would fund the national debt would have a direct interest in a strong and successful national government.

d. The wealthy lenders who would fund the national debt would have a direct interest in a strong and successful national government.

*Picture of pie graph displaying the percentage of slave populations. 5 percent in New England, 36 in Upper South, 58 in Lower South* The difference in slave populations depicted in the graphs most directly resulted from differences in a. colonial relationships with American Indians b. the degree of participation in the Atlantic economy c. structures of British imperial control d. climate and geographic conditions for cash crop agriculture

d. climate and geographic conditions for cash crop agriculture

"Young ladies in town, and those that live round, Let a friend at this season advise you: Since money's so scarce, and times growing worse, Strange things may soon hap and surprise you; First then, throw aside your high top knots of pride, Wear none but your own country linen, Of Economy boast, let your pride be the most To show clothes of your own make and spinning." The Daughters of Liberty Urge Americans to Boycott British Goods One significant long term result of the women's efforts described in the excerpt was the a. development of the homespun textile and cotton industry in the North. b. repeal of the Stamp Act c. increase in British hat manufacturers' profit and the extension of hat duties. d. independence of the American states

d. independence of the American states

"We hold these truths to be self-evident, that all men are created equal; that they are endowed by their Creator with certain unalienable rights, and that among these are, life, liberty, and the pursuit of happiness.'' Here was a time, in which your tender feelings for yourselves had engaged you thus to declare, you were then impressed with proper ideas of the great violation of liberty, and the free possession of those blessings, to which you were entitled by nature; but, Sir, how pitiable is it to reflect, that although you were so fully convinced of the benevolence of the Father of Mankind, and of his equal and impartial distribution of these rights and privileges, ... in detaining by fraud and violence so numerous a part of my [enslaved] brethren, under groaning captivity and cruel oppression, that you should at the same time be found guilty of that most criminal act, which you professedly detested in others, with respect to yourselves." Letter of Benjamin Banneker to Thomas Jefferson, 1791 The view of many Southern plantation owners that the Declaration of Independence did not invalidate their property rights in slaves led to a. their ban on slavery in the Northwest Territory. b. the first amendment in the Bill of Rights. c. their support of a South Carolinian abolition society. d. the 3/5 compromise in the Constitution.

d. the 3/5 compromise in the Constitution

This political cartoon was intended to a highlight the influence of money in the late 19th century on urban populations. b demonstrate that people can get wealthy by using their brain c criticize the work of Progressive reformers in promoting greater political participation in the direct primary elections and conventions d expose corruption in New York City's government.

d expose corruption in New York City's government.

In an effort to gain more northern support, the Democratic Party adopted which issue by the 1854 election? a Non-expansion of slavery b Gradual abolition c Enforcement of the Fugitive Slave Law d Popular sovereignty

d popular sovereignty

Which of the following best explains a reaction to the changes in energy production depicted on the graph? A Political leaders argued that excessive power consumption was economically wasteful. B Manufacturers began moving factories to countries with sufficient power supplies. C Americans voluntarily reduced their power consumption to save resources. D Environmental groups were formed to lobby against increased pollution.

D Environmental groups were formed to lobby against increased pollution.

*Can't find chart, but it shows the economic status of both the North and the South during the Civil War* The information provided in the chart would be most useful to historians as a source of information about which of the following regarding the Civil War? a The advantages enjoyed by the North over the course of the entire Civil War b The assets that the South had that helped them resist Union forces c The diplomatic disadvantages faced by the North d The economic resources both the South and North had to mobilize at the start of the war effort

a The advantages enjoyed by the North over the course of the entire Civil War

"Now, we can see a new world coming into view. . . . In the words of Winston Churchill, a world order in which 'the principles of justice and fair play protect the weak against the strong. . . .' Aworld where the United Nations . . . is poised to fulfill the historic vision of its founders. A world in which freedom and respect for human rights find a home among all nations. The Gulf War put this new world to its first test. And my fellow Americans, we passed that test." - President George H. W. Bush, address to Congress, March 6, 1991 Which of the following events most directly led to President Bush believing that a new "world order" was emerging? a The end of the Cold War with the Soviet Union b United States military forces withdrawing from Vietnam c The spread of computers and global information networks d Terrorist attacks on the World Trade Center and the Pentagon

a The end of the Cold War with the Soviet Union

The growing rift between conservative and liberal political perspectives in the 1970s was most likely in response to which of the following? A Political scandals B Declining public confidence C Social and cultural issues D Containment foreign policy

C Social and cultural issues

*Keep Within Compass picture* The creator of the illustration above would most likely have supported which of the following? a. The ideal of "republican motherhood" b. The rise of Gilded Age women's clubs c. The antebellum women's movement d. The efforts of women to gain the right to vote

a. The ideal of "republican motherhood"

One significant effect of the program referred to in the picture above was that it: a. Allowed for the Soviet Union to rebuild after the war. b. Prevented Western Europe from falling to communism. c. Prevented Iran from gaining nuclear weapons. d. Sparked a period of Isolationism in the United States

b. Prevented Western Europe from falling to communism.

The artistic movement of the era was founded by artists from a. California. b. the South. c. upstate New York d. the Appalachian Mountains.

c


Related study sets

Biochemistry: Amino Acids and Urea Cycle, Glycolysis, metabolic path..., Membrane Transport, Citric Acid Cycle, Lipids and Membranes

View Set

Chapter 1: Fire protective system

View Set

Exam Study Guide 3- Spinal Cord Injuries (PrepU)

View Set

History of Architecture - Egyptian Architecture

View Set